Vous êtes sur la page 1sur 91

Test Information

Test Name Grand Test 122 Total Questions 300

Test Type Examination Difficulty Level Difficult

Total Marks 1200 Duration 180minutes

Test Question Language:- ENGLISH


Q.1 All of the following are medial rotators of the arm except
a. Latissimus dorsi
b. Teres major
c. Subscapularis
d. Infraspinatus
Your Ans. d Correct Ans. d

Solution. Ans-1: (d) Infraspinatus


Ref: Read the text below
Sol :
· The latissimus dorsi, teres major, and subscapularis medially rotate the arm.
· The infraspinatus and teres minor rotate the arm laterally.
· The deltoid is unique in that its anterior part rotates the arm medially, and its posterior part rotates the arm laterally.
Q.2 Multiple spontaneous idiopathic and symmetrical fractures are characteristic of :
a. Stress fracture
b. Milkman’s syndrome
c. Rickets
d. Osteoporosis
Your Ans. b Correct Ans. b

Solution. Ans-2: (2) Milkman’s syndrome


Ref: Read the text below
Sol :
· Decreased tubular reabsorption of phosphate, resulting in osteomalacia which gives a peculiar striped appearance (multiple
pseudofractures) to the bones in roentgenograms.
Also known as Looser-Milkman syndrome.
Q.3 Chondrocalcinosis is seen in :
a. Ochronosis
b. Hyperparathyroidism
c. Rickets
d. Hypervitaminosis D
Your Ans. d Correct Ans. a

Solution. Ans-3: (a) Ochronosis


Ref: CMDT -2002, 841
Sol :
§ Chondrocalcinosis is calcification ofarticular cartilage.
§ It is seen in calcium pyrophosphate dehydrate deposition disease, hyperparathyroidism, haemochromatosis,
alkaptonuria, Acromegaly, gout, Wilson’s disease.
Q.4 Commonest position of appendix :
a. Pelvic
b. Retrocaecal
c. Subcaecal
d. Promontoric
Your Ans. b Correct Ans. b
Solution. Ans-4: (b) Retrocaecal
Ref: BDC - 256
Sol :
§ The vermiform appendix is located in the right lower quadrant of abdomen.
§ It is a narrow, worm shaped tube, arising from the posteromedial caecal wall, 2 cms or less below the end of the
ileum.
§ Its opening is occasionally guarded by a semicircular fold of mucous membrane known as the valve of gerlach.
§ The appendix is usually located at the junction of the taeniae, found on the surface of the caecum.
§ Its length varies from 2-20 cms, with an average length of 9 cms.
§ The attachment of the base of the appendix to the caecum remains constant, whereas the tip can be found in a retrocaecal
(65%)-commonest, pelvic (30%)-second most common, subcaecal, preileal, post-ileal or promontoric positions.
Q.5 All of the following structures are located within the rectus sheath except
a. Pyramidalis
b. Rectus abdominis
c. Inferior epigastric arteries and veins
d. Deep inguinal ring
Your Ans. d Correct Ans. d

Solution. Ans-5: (d) Deep inguinal ring


Ref: Read the text below
Sol :
· The contents of the rectus sheath include the pyramidalis and rectus abdominis muscles, the superior and inferior
epigastric arteries and veins, the lymphatics, and ventral primary rami of T7–T12 nerves (Moore, p 180).
Q.6 Inferiorly, the inferior margin of the external oblique aponeurosis thickens and folds back on itself to form which
of the following structures?
a. Rectus sheath
b. Inguinal ligament
c. Arcuate line
d. Deep inguinal ring
Your Ans. b Correct Ans. b

Solution. Ans-6: (b) Inguinal ligament


Ref: Read the text below
Sol :
· Inferiorly, the inferior margin of the external oblique aponeurosis thickens and folds back on itself to form the inguinal
ligament, a fibrous band extending between the anterior superior iliac spine and the pubic tubercle.
Q.7 All of the following structures forms the stomach bed except :
a. Hepatic flexure of colon
b. The left kidney
c. The pancreas
d. Transverse mesocolon
Your Ans. a Correct Ans. a

Solution. Ans-7: (a) Hepatic flexure of colon


Ref: BDC -240, 258
Sol :
§ Anteriorly :The anterior abdominal wall, the left costal margin, the left pleura and lung, the diaphragm, and the left lobe of
the liver.
§ Posteriorly : The lesser sac, the diaphragm, the spleen, the left suprarenal gland, the upperpart of the left kidney,
the splenic artery, the pancreas, the transverse mesocolon, and the transverse colon.
§ Hepatic flexureof colon is related to the colic impression on the inferior surface of the right lobe of the liver.
Q.8 All of the following are thought to modulate gastrointestinal blood flow except :
a. Adenosine and oxygen
b. Parasympathetic nervous system
c. Arteriolar and precapillary sphincters
d. Hormones
Your Ans. b Correct Ans. b
Solution. Ans-8: (b) Parasympathetic nervous system
Ref:Read the text below
Sol :
§ The sympathetic rather than the parasympathetic nervous system contributes to the modulation of gastrointestinal
blood flow.
Q.9 Histology of myositis ossificans mimics :
a. Hemophilic arthropathy
b. Osteosarcoma
c. Osteoclastoma
d. Fibrosarcoma
Your Ans. b Correct Ans. b

Solution. Ans-9: (2) Osteosarcoma


Ref: Read the text below
Sol :
§ Myositis ossificans differshistological from extraosseous osteosarcoma by zone phenomenon characterized by well
oriented bone speckles separated by benign appearing stroma near outer margin of lesion
Q.10 A patient is asked to place the hands posteriorly on the hips and to push the elbows posteriorly against
resistance. Which muscle is being tested?
a. Levator scapulae
b. Rhomboid
c. Trapezius
d. Latissimus dorsi
Your Ans. d Correct Ans. b

Solution. Ans-10: (b) Rhomboid


Ref: Read the text below
Sol :
To test the rhomboids, the patient is asked to place the hands posteriorly on the hips and to push the elbows
posteriorly against resistance
Q.11 Which of the following statements concerning the sacral sympathetic trunks is correct?
a. Usually has four sympathetic ganglia.
b. Descends on the pelvic surface of the ischium.
c. Ends as the dorsal nerve of the penis or clitoris.
d. Pierces the sacrotuberous ligament.
Your Ans. b Correct Ans. a

Solution. Ans-11: (a) Usually has four sympathetic ganglia.


Ref: Read the text below
Sol :
· The sacral sympathetic trunks descend posterior to the rectum in the extraperitoneal connective tissue and send gray
rami communicantes to each of the ventral rami of the sacral and coccygeal nerves.
· Each of the sacral trunks is smaller than the lumbar trunks and usually has four sympathetic ganglia (Moore, p 350).
Q.12 The inferior hypogastric plexus receives fibers from which of the following nerves?
a. Pudendal and obturator
b. Sciatic and superior gluteal
c. Inferior gluteal and lumbosacral
d. Pelvic splanchnic and hypogastric
Your Ans. d Correct Ans. d

Solution. Ans-12: (d) Pelvic splanchnic and hypogastric


Ref: Read the text below
Sol :
· The pelvic splanchnic nerves merge with the hypogastric nerves to form the inferior hypogastric (and pelvic)
plexuses. (Moore,p 350).
Q.13 “Trident hand” seen in :
a. Achondroplasia
b. Mucopolysaccharidosis
c. Diaphyseal achlasia
d. Cleido-cranial dystosis
Your Ans. a Correct Ans. a

Solution. Ans-13: 1) Achondroplasia


Ref: Read the text below
Sol :
§ Achondroplasia is rhizomelic micromelia associated with frontal bossing and low nasal bridge.
§ It is due to Defective cartilaginous molding of the bone precursor. The micromelia is the most obviousfinding with
limbs shorter than the 5th percentile after 20 weeks.
§ The frontal bossing and depressed nasal bridge can also be recognized occasionally
§ More subtle anomalies such as the trident hand (an increased interspace between the 3rd and 4th digit) or the lack of
widening of the lumbar canal can also be identified.
Q.14 Which rotator cuff muscle does not rotate the humerus?
a. Supraspinatus
b. Infraspinatus
c. Teres minor
d. Subscapularis
Your Ans. d Correct Ans. a

Solution. Ans-14: (a) Supraspinatus


Ref: Read the text below
Sol :
· The supraspinatus is the only rotator cuff muscle that does not rotate the humerus.
Q.15 The deep artery of the arm accompanies which of the following before passing around the body of the humerus?
a. Radial nerve
b. Musculocutaneous nerve
c. Median nerve
d. Ulnar nerve
Your Ans. a Correct Ans. a

Solution. Ans-15: (a) Radial nerve


Ref: Read the text below
Sol :
· The deep artery of the arm accompanies the radial nerve through the radial groove and passes around the body of
the humerus
Q.16 Less O2 saturation in blood is seen in :
a. R-L shunt
b. LV obstruction
c. RV obstruction
d. L-V shunt
Your Ans. a Correct Ans. a

Solution. Ans-16: (a) R-L shunt


Ref: Ganong’s Physiology, 23rd ed., p.-620
Sol :
§ Shunting of blood from the right sideto the left side of the circulation (right-to-left shunt) is powerful cause of hypoxemia.
§ The shunt may be intracardiac or may be intrapulmonary.
§ It has been traditionally thought that this cause could be readily distinguished from the others as the only cause that
cannot be corrected by the administration of 100% oxygen.
Q.17 All of the following statements about the protein kinase C signal transduction pathway are true except
a. After activation, protein kinase C is degraded to protein kinase M
b. Protein kinase C phosphorylates tyrosines on proteins
c. Protein kinase C requires Ca2+ for full activation
d. Protein kinase C requires lipids for full activation
Your Ans. c Correct Ans. b
Solution. Ans-17: (b) Protein kinase C phosphorylates tyrosines on proteins
Ref:Read the text below
Sol :
§ Protein kinase C isa member of a class of kinases that phosphorylates only serine and threonine, not tyrosines.
§ Protein kinase C is activated bylipids and Ca2+ ; when activation occurs, protein kinase C moves from the cytoplasm to the
plasma membrane via the process called translocation.
§ It is degraded by a calcium-activated protease to form the lipid- and Ca2+ -independent protein kinase M.
Q.18 Each statement below concerning cyclic adenosine monophosphate (cAMP) is true except
a. cAMP levels may be increased or decreased by hormone stimulation
b. It is the second messenger for the action of parathyroid hormone (PTH) on the kidney
c. It activates protein kinase C by binding to the regulatory subunit and causing dissociation of the catalytic subunit
d. It is degraded intracellularly by a family of phosphodiesterase isoenzymes
Your Ans. c Correct Ans. c

Solution. Ans-18: (c) It activates protein kinase C by binding to the regulatory subunit and causing dissociation of
the catalytic subunit
Ref:Read the text below
Sol :
§ Protein kinase C is activated by Ca2+ and diacylglycerol, not by cyclic adenosine monophosphate (cAMP). Cyclic
AMPis synthesized from adenosine triphosphate (ATP) in a reaction that is catalyzed by adenylate cyclase.
§ The activity of adenylate cyclase may be either increased or decreased in response to hormone stimulation. Cyclic AMP is
the second messenger for the effect of parathyroid hormone (PTH) on the kidney; cAMP also activates protein
kinase A.
§ The binding of cAMP to the regulatory subunit results in the dissociation of the regulatory and catalytic subunits and a
concomitant increase in protein kinase activity.
§ The degradation of cAMP is mediated by a family of phosphodiesterases, which catalyze the hydrolysis to 5’ –
AMP.
Q.19 Which is responsible for respiratory drive:
a. O2
b. CO
c. CO2
d. Bicarbonate ions
Your Ans. c Correct Ans. c

Solution. Ans-19: (c) CO2


Ref: Ganong’s Physiology, 23rd ed., p.-516
Sol :
§ Although the body requires oxygen for metabolism, low oxygen levels do not stimulate breathing. Rather, respiratory
centre is directly stimulated by higher carbon dioxide levels or excess hydrogen ions in the blood.
§ As a result, breathing low-pressure air or a gas mixturewith no oxygen at all (such as pure nitrogen) can lead to loss of
consciousness without ever experiencing air hunger.
§ The respiratory centers try to maintain anarterial CO2 pressure of 40 mm Hg. With intentional hyperventilation, the CO2
content of arterial blood may be lowered to 10-20 mm Hg (the oxygen content of the blood is little affected), and the
respiratory drive is diminished.
Q.20 Regarding lung volumes, which of the following is true :
a. Functional residual capacity accounts for 75% of total lung capacity
b. Residual volume keeps alveoli inflated between breaths
c. The air inspired with a maximal inspiratory effort in excess of the tidal volume is 1500 ml
d. Residual volume is about 500 ml
Your Ans. c Correct Ans. b

Solution. Ans-20: (b) Residual volume keeps alveoli inflated between breaths
Ref: Ganong’s Physiology, 23rd ed., p.-474
Sol :
§ The amount of air that moves into the lungs with each inspiration (or the amount that moves out with each expiration) is
called the tidal volume (500 ml)
§ The air inspired with a maximal inspiratory effort in excess of the tidal volume is the inspiratory reserve volume.
(3000 ml)
§ The volume expelled by an active expiratory effort after passive expiration is the expiratory reserve volume (1200 ml), and
the air left in the lungs after a maximal expiratory effort is the residual volume (1200 ml).
Q.21 Which of the following would you least likely see in a person with longstanding type 2 diabetes?
a. Neuropathy
b. Nephropathy
c. Retinopathy
d. Ketoacidosis
Your Ans. d Correct Ans. d

Solution. Ans 21: (d) Ketoacidosis


Ref: Read the text below
Sol:
· Neuropathy, nephropathy, and retinopathy are chronic complications of type 2 diabetes.
· Ketoacidosis is an acute complication seen in type 1 diabetes.
Q.22 Major contribution to plasma osmolality is by which ion :
a. Sodium
b. Potassium
c. Glucose
d. Calcium
Your Ans. a Correct Ans. a

Solution. Ans-22: (a) Sodium


Ref: Ganong’s Physiology, 23rd ed., p.-6
Sol :
§ All but about 20 of the 290 mosm in each liter of normal plasma are contributed by Na+ and its accompanying anions,
principally Cl- and HCO3-. Other cations and anions make a relatively small contribution.
§ Although the concentration of the plasma proteins is large when expressed in grams per liter, they normally contribute
less than 2 mosm/L because of their very high molecular weights.
§ The major nonelectrolytes of plasma are glucose and urea, which in the steady state are in equilibrium with cells.
§ Their contributions to osmolalityare normally about 5 mosm/L each but can become quite large in hyperglycemia or
uremia.
§ The total plasma is important in assessing dehydration overhydration, and other fluid and electrolyte abnormalities.
§ Hyperosmolality can cause coma (hyperosmolar coma).
Q.23 Repolarization of a nerve is due to :
a. Hydrogen ions
b. Potassium ions
c. Sodium ions
d. Calcium ions
Your Ans. b Correct Ans. b

Solution. Ans-23: (b) Potassium ions


Ref: Ganong’s Physiology, 23rd ed., p.-85
Sol :
§ The Na+ channels rapidly enter the inactivated state and remain in this state for a few milliseconds before returning to
the resting state. In addition, the direction of the electrical gradient for Na+ is reversed during the overshoot because the
membrane potential is reversed, and this limits Na+ influx.
§ A third factor producing repolarization is the opening of voltage-gated K+ channels.
§ The net movement of positive charge out of the cell due to K+ efflux at this time helps complete the process of
repolarization.
§ The slow return of the K+ channels to the closed state also explains the after-hyperpolarization.
Q.24 With glucose which of the following is transported :
a. Hydrogen ions
b. Potassium ions
c. Calcium ions
d. Sodium ions
Your Ans. d Correct Ans. d

Solution. Ans-24: (d) Sodium ions


Ref: Ganong’s Physiology, 23rd ed., p.-318
Sol :
· Sodium-dependent glucose cotransporters(SGLT) are a family of glucose transporterfound in the intestinal
mucosa(enterocytes) of the small intestine(SGLT1) and the proximal tubuleof the nephron(SGLT2 in PCTand SGLT1 in PST).
· They contribute to renal glucose reabsorption. In the kidneys, 100% of the filtered glucose in the glomerulushas to be
reabsorbed along the nephron (98% in PCT, via SGLT2).
· In case of too high plasma glucose concentration (hyperglycemia), glucose is excreted in urine (glucosuria); because SGLT
are saturated with the filtered monosaccharide. One must know that glucose is never secreted by the nephron
Q.25 During pregnancy, deficiency of which vitamin in women causes neural tube defect in the newborn:
a. Thiamine
b. Pyridoxine
c. Folic acid
d. Cyanocobalamin
Your Ans. c Correct Ans. c

Solution. Ans-25: (c) Folic acid


Ref: Ganong’s Physiology, 23rd ed., p.-465
Sol :
Deficiency of :
§ Thiamine causes Beri Beri, neuritis
§ Pyridoxine causes Convulsion, hyperirritability
§ Folic acid causes sprue, anemia and neural tube defect in the newborn to folate-deficient mothers
§ Cyanocolbalamin causes pernicious anemia, loss of vibration and position sense, dementia, abnormal gait.
Q.26 Main content of bilayer cell membrane is :
a. Glycerol
b. Cholesterol
c. Cholesterol ester
d. Triacyl glycerol
Your Ans. d Correct Ans. b

Solution. Ans-26: (b) Cholesterol


Ref: Harper’s Biochemistry, 28th ed., p-408
Sol :
§ Biological membranes typically include several types of lipids other than phospholipids.
§ A particularly important example in animal cells is cholesterol, which helps strengthen the bilayer and decrease
its permeability.
§ Cholesterol also helps regulate the activity of certain integral membrane proteins.
§ Integral membrane proteins function when incorporated into a lipid bilayer.
§ Because bilayers define the boundaries of the cell and its compartments, these membrane proteins are involved in
many intra-and inter-cellular signaling processes.
Q.27 Oily substance secreting gland is :
a. Sweat gland
b. Meibomian gland
c. Salivary gland
d. Lacrimal gland
Your Ans. b Correct Ans. b

Solution. Ans-27: (b) Meibomian gland


Ref: Internet resources
Sol :
§ The meibomian glands (or tarsal glands) are a special kind of sebaceous glands at the rim of the eyelids inside the tarsal
plate, responsible for the supply of meibum, an oily substance that prevents evaporation of the eye’s tear film, prevents
tear spillage onto the cheek, makes the closed lids airtight and acts as a blockade for tear fluid, trapping tears between
the oiled edge and eyeball.
Q.28 Beta waves has a frequency:
a. 8-12 Hz
b. 12 and 30 Hz
c. 2 and 8 Hz
d. 30 and 40 Hz
Your Ans. b Correct Ans. b

Solution. Ans-28: (b) 12 and 30 Hz


Ref: Ganong’s Physiology, 23rd ed., p.-233
Sol :
Beta wave, or beta rhythm, is the term used to designate the frequencyrange of human brainactivity between 12 and 30 Hz(12
to 30 transitions or cycles per second). Beta waves are split into three sections: High Beta Waves (19 Hz+); Beta Waves
(15–18 Hz); and Low Beta Waves (12–15 Hz). Beta states are the states associated with normal waking consciousness
Q.29 Metabolic alkalosis will be observed in a patient with
a. Hyperaldosteronism
b. Hyperventilation
c. Persistent diarrhea
d. Renal failure
Your Ans. a Correct Ans. a

Solution. Ans-29: (a) Hyperaldosteronism


Ref: Read the text below
Sol :
· Metabolic alkalosis is causedby the loss of nonvolatile, or fixed, acid from the body. An increased aldosterone secretion
will cause an increase in H+ secretion from the distal nephron, resulting in metabolic alkalosis.
· Persistent diarrhea will cause the loss of bicarbonate fromthe body resulting in metabolic acidosis. Renal failure is
often accompanied by metabolic acidosis because of the inability to excrete H+.
· Diabetes also causes metabolic acidosis because of the accumulation of keto acids. Hyperventilation results in a
respiratory alkalosis, which is compensated for by a decreased excretion of acid by the kidneys, resulting in a
metabolic acidosis.
Q.30 Which one of the following substances causes renal blood flow to decrease?
a. Nitric oxide
b. Bradykinin
c. Prostaglandins
d. Adenosine
Your Ans. b Correct Ans. d

Solution. Ans-30: (d) Adenosine


Ref: Read the text below
Sol :
· Blood flow through the kidney is controlled by a myriad of humoral agents. Adenosine acts via A2 receptors to
increase afferent arteriolar resistance and thereby decrease renal blood flow.
· Nitric oxide dilates the afferent arteriole and constricts the efferent arteriole, producing a rise in glomerular
capillary pressure (and glomerular filtration) without having much of an effect on renal blood flow.Prostaglandins,
bradykinin, and dopamine all increase renal blood flow.
· Cyclooxygenase inhibitors, such as aspirin, that decrease prostaglandin synthesis may impair renal blood flow
sufficiently to exacerbate the effects of renal failure.
Q.31 What is the end product of anaerobic glycolysis :
a. Pyruvate
b. Lactate
c. Fats
d. cholesterol
Your Ans. b Correct Ans. b
Solution. Ans-31: (b) Lactate
Ref: Harper’s Biochemistry, 28th ed., page-149
Sol :

Q.32 Enzyme deficient in Marfan’s syndrome is :


a. Cystathione B synthetase
b. Lysyl oxidase
c. Collagenase
d. None of the above
Your Ans. a Correct Ans. d

Solution. Ans-32: (d) None of the above


Ref: Harper’s Biochemistry, 28th ed., page-531
Sol :
§ Marfan syndrome is caused by mutations in the FBN1 gene on chromosome 15, which encodes a glycoprotein called
fibrillin-1, which is essential for the proper formation of the extracellular matrix including the biogenesis and maintenance of
elastic fibers.
§ Elastin fibers are found throughout the body but are particularly abundant in the aorta, ligaments and the ciliary
zonules of the eyes, consequently these areas are among the worst affected.
Q.33 Which of the following does not occur in starvation :
a. Hypoglycemia
b. Hypercholesterolemia
c. Lipolysis
d. Ketoacidosis
Your Ans. b Correct Ans. b

Solution. Ans-33: (b) Hypercholesterolemia


Ref: Harper’s Biochemistry, 28th ed., page-141
Sol :
§ When food intake ceases, the body’s glycogen stores are used up in about 24 hours.
§ The level of insulin in circulation is low and the level of glucagon is very high.
§ The main means of energy production is lipolysis.
§ Gluconeogenesis converts glycerol into glucose and the Cori cycle converts lactate into usable glucose.
§ The high demand for glucose by the fetus, and for lactose synthesis in lactation, can lead to ketosis. This may be seen as mild
ketosis with hypoglycemia in human beings.
Q.34 Which of the following is a debranching enzyme ?
a. Glycogen synthetase
b. Glucose-6-phosphatase
c. Amylo 1, 6-glucosidase
d. Amylo 1, 4-1-1, 6 transglycosylase
Your Ans. d Correct Ans. c
Solution. Ans-34: (c) Amylo 1, 6-glucosidase
Ref.:Read the text below
Sol :
§ Debranching enzyme acts during glycogenolysis once phosphorylase and glucan 1,4 – 1, 4 transferase has been acted
upon glycogen.
§ The last residue of glucose attached to glycogen in 1, 6 linkage is removed via action of debranching enzyme (1, 6
glucosidase.)
Q.35 A 54 yr old male presents with weakness, fatigue, shortness of breath and dizziness. Hb was found to be 6.4 g/dl.
RBC shows low lactate production. Which enzyme deficiency is most likely in this patient ?
a. Phosphoglucoisomerase
b. Phosphofructokinase
c. Pyruvate kinase
d. Lactate dehydrogenase
Your Ans. c Correct Ans. c

Solution. Ans-35: (c) Pyruvate kinase


Ref.:Read the text below
Sol :
§ Out of all the cases showing deficiency of glycolytic enzymes, about 95% of the patients show the deficiency of
pyruvate kinase and 4% exhibit deficiency of phophoguco isomerase.
§ Most common enzyme to be deficient in haemolytic anemia is the glucose 6 phosphate dehydrogenase, second most common
enzyme to be deficient in a case of haemolytic anemia is the pyruvate kinase.
Q.36 Reaction unique to gluconeogenesis ?
a. Lactate – pyruvate
b. Oxaloacetate – phosphoenolpyruvate
c. Phosphoenolpyruvate – pyruvate
d. Glucose 6 phosphate – fructose 6 phosphate
Your Ans. b Correct Ans. b

Solution. Ans-36: (b) Oxaloacetate – phosphoenolpyruvate


Ref.:Read the text below
Sol :
· All other reactions are common to both glycolysis and gluconeogenesis.
Q.37 Which of the following is not a phospholipids?
a. Cerebroside
b. Plasmalogen
c. Sphingomyelin
d. Cephalin
Your Ans. b Correct Ans. b

Solution. Ans-37: (b) Plasmalogen


Ref.:Read the text below
Sol :
§ Plasmalogens are compoundswhich are similar to phospholipids with certain differences.
§ First difference is that in plasmalogens first fatty acid is attached at sn-1 with the help of ether bond and not like ester
bond found in phospholipids.
§ Second differences is plasmalogens, first fatty acid is mostly unsaturated alcohol.
Q.38 Anticoagulant added to blood for estimation of prothrombin time is :
a. Heparin
b. Oxalate
c. Sodium citrate
d. EDTA
Your Ans. c Correct Ans. c
Solution. Ans-38: (c) Sodium citrate
Ref: Internet resources
Sol :
§ There are many variables that can affect the PT/INR result outcome.
§ In fact, preanalytical variables account for up to 64% of all errors in PT/INR testing.
§ One of the most important factors is the anticoagulant used in drawing the blood specimen.
§ The World Health Organization guidelines recommend the use of 3.2% buffered citrate.
§ The evacuated tube must be completely filled (at least 90% full) to maintain the proper anticoagulant-to-blood
ratio.
§ It is best to avoid traumatic venipunctures to minimize the release of tissue factor, which can initiate coagulation.
Q.39 Which of the following is the radiosensitive stage of cell cycle :
a. G0
b. G1
c. G2
d. S
Your Ans. c Correct Ans. c

Solution. Ans-39: (c) G2


Ref: Harper’s Biochemistry, 28th ed., page-516
Sol :
§ Irradiation of cells may not only lead to cell death but to other changes as well.
§ Radiation has been shown to alter a cell’s progression through the cell cycle.
§ The delay occurs at particular points in the cell cycle, such as early G2 and the G1/s interface.
§ Multiple factors affect this kinetic alteration, including the radiation dose rate, and cell type. The cell’s position in the
cell cycle is a factor also.
Q.40 How many molecules of pyruvate are formed from complete metabolism of 1 molecule of glucose:
a. 1
b. 2
c. 3
d. 4
Your Ans. b Correct Ans. b
Solution. Ans-40: (b) 2
Ref: Harper’s Biochemistry, 28th ed., page-154-55
Sol :

Q.41 Compared with a healthy individual, lymph nodes from a person with a deficiency in B lymphocytes would have
a. Few or no primary follicles
b. Enlarged germinal centers
c. Few Howell-Jolly bodies
d. No paracortex
Your Ans. d Correct Ans. a

Solution. Ans 41 : (a) Few or no primary follicles.


Ref– Read the text below
Sol:
· The major cell type within follicles is the B cell; a germinal center is a follicle where cells are undergoing active
proliferation.
· A deficiency in B cells would result in decreased size and number of follicles.
· The paracortex is predominately a T cell area. Heinz bodies (red cell inclusion body of denatured hemoglobin) and
Howell-Jolly bodies (red cell inclusion body of parasites) are found in the spleen.
Q.42 The multistep theory of carcinogenesis can be applied to what form of cancer?
a. Head and neck cancer
b. Breast cancer
c. Lung cancer
d. Colorectal cancer
Your Ans. d Correct Ans. d
Solution. Ans 42 : (d) Colorectal cancer.
Ref– Read the text below
Sol:
· Colon canceris one cancer in which the multistep theory of carcinogenesis has been studied.
· In the multistep theory, the cancer develops from multiple somatic mutational steps that change normal epithelium to
adenoma to carcinoma.
· An accumulation of somatic mutations is necessary for cancer to develop.
Q.43 Acute infectious purpura fulminans is caused by
a. Neisseria meningitis and Varicella
b. Gonococci
c. E. Coli
d. Proteus
Your Ans. a Correct Ans. a

Solution. Ans-43: (a) Neisseria meningitis and Varicella


Ref: Read the text below
Sol :
It is a life-threatening disorder of acute onset. It is characterized by cutaneous haemorrhageand necrosis(tissue death), low blood
pressure, feverand disseminated intravascular coagulation.[citation needed]
Causes
Common causes are severe infection (especially with meningococcus, and Capnocytophaga canimorsus, and other Gram-
negative organisms), and deficiency of the natural anticoagulants protein Cor protein Sin the blood. In some cases, a cause is
never found.
Treatment
Treatment is mainly by removing the underlying cause and degree of clotting abnormalities and with supportive treatment
(antibiotics, volume expansion, tissue oxygenation, etc.). Thus, treatment includes aggressive management of the septic state.
Surgical debridement, escharotomies, fasciotomies, and even amputations. In many cases, digits may need to be amputated
when their blood supply has ceased completelyThe use of full dose heparinor other anticoagulantis controversial.
Q.44 What is the most common extra-articular manifestation of ankylosing spondylosis ?
a. Anterior uveitis
b. Aortic regurgitation
c. Cataract
d. Inflammatory bowel disease
Your Ans. a Correct Ans. a

Solution. Ans-44: (a) Anterior uveitis


Ref: Harrison’s- 2111.
Sol :
Ankylosing Spondylitis (AS)
§ Synonyms : Marie Strumpell disease, Bechetrew disease
§ HLA B27 associated seronegative spondyloarthropathy
§ Bony anklyosis of spine – “bamboo spine”
§ Affects young adults, low back pain and limitation of flexion and extension of lumbar spine is prominent.
§ Schober test is useful measure of lumbar spine flexion.
§ Extra-articular manifestations :
a. Acute anterior uveitis (most common, occurs in 40%, can antedate the spondylitis)
b. Inflammatory bowel disease, cauda equine syndrome, upper pulmonary lobe fibrosis, aortic insufficiency, retroperitoneal
fibrosis, prostatitis, amyloidosis.

Q.45 The most specific autoantibody associated with lupus erythematous is


a. dsDNA
b. PCNA
c. Cardiolipin
d. Clq
Your Ans. a Correct Ans. a
Solution. Ans-45: (a) dsDNA
Ref: Read the text below
Sol :
· Antinuclear antibody(ANA) testing and anti-extractable nuclear antigen (anti-ENA) form the mainstay of
serologictesting for SLE. Several techniques are used to detect ANAs. Clinically the most widely used method is indirect
immunofluorescence. The pattern of fluorescence suggests the type of antibody present in the patient's serum.
· ANA screening yields positive results in many connective tissue disorders and other autoimmune diseases, and may
occur in normal individuals. Subtypes of antinuclear antibodies include anti-Smithand anti-double stranded DNA(dsDNA)
antibodies (which are linked to SLE) and anti-histone antibodies(which are linked to drug-induced lupus). Anti-dsDNA
antibodies are highly specific for SLE; they are present in 70% of cases, whereas they appear in only 0.5% of
people without SLE.[2]The anti-dsDNA antibody titersalso tend to reflect disease activity, although not in all
cases.[2]Other ANA that may occur in SLE sufferers are anti-U1 RNP(which also appears in systemic sclerosis),
SS-A(or anti-Ro) and SS-B(or anti-La; both of which are more common in Sjögren's syndrome). SS-A and SS-B confer a specific
risk for heart conduction block in neonatal lupus.[46]
· Other tests routinely performed in suspected SLEare complement systemlevels (low levels suggest consumption by
the immune system), electrolytesand renal function(disturbed if the kidney is involved), liver enzymes, and complete blood
count.
· The lupus erythematosus (LE) cell test was commonly used for diagnosis, but it is no longer used because the LE cellsare
only found in 50–75% of SLE cases, and they are also found in some people with rheumatoid arthritis, scleroderma, and drug
sensitivities. Because of this, the LE cell test is now performed only rarely and is mostly of historical significance
Q.46 Henoch-Schonlein purpura is characterized by the deposition of the following immunoglobulin around the vessels
a. IgM
b. IgG
c. IgA
d. IgE
Your Ans. c Correct Ans. c

Solution. Ans-46: (c) IgA


Ref: Robbin’s 8th/e p. 934
Sol :
Henoch-Schonlein purpura
§ Henoch-Schonlein purpura is vasculitis of small vessels (capillaries, venule or arterioles) and characterized by deposition of
IgA in the wall of involved vessels.
§ H.S. purpura is characterized by tetrad of purpura, arthritis, glomerulonephritis, and abdominal pain.
§ All changes result from the deposition of circulating immune complexes, containing IgA and complement on the
endothelium of the blood vessels throughout the body and within the glomerular mesangial regions.
Q.47 Kaposi sarcoma associated with gut may be seen in
a. Nonhodgkin’s lymphoma
b. HIV – infection
c. Fungal infection
d. Keratoacanthosis
Your Ans. b Correct Ans. b

Solution. Ans-47: (b) HIV – infection


Ref: Harrison 17th/e p. 1186
Sol :
§ There are at least four distinct epidemiological form of Kaposi sarcoma.
o Classic form seen in older men of Mediterranean of Eastern European origin.
o No recognized contributing factor (not associated with AIDS)
§ Equatorial African form that occurs in all ages
o Without any recognized precipitating factor
§ Kaposi sarcoma associated with organ transplantation and its attendant iatrogenic immunosuppressed state
§ Form associated with HIV – Infection
§ GI involvement (as asked in this question) is seen in 50% of patients, and usually takes one of these 2 form :
o Mucosal involvement : which may lead to severe bleeding.
o Biliary tract involvement : Lesions may infiltrate the GB leading to obstructive jaundice.
§ Kaposi sarcoma is the most common neoplasm in AIDS Patients.
Q.48 The following are predisposing factors for esophageal Ca except
a. Plummer-Vinson syndrome
b. Tylosis Palmaris
c. Chronic Achalasia
d. Benzene therapy
Your Ans. d Correct Ans. d
Solution. Ans-48: (d) Benzene therapy
Ref: Harrison 17th/e p. 807
Sol :
Predisposing factors for ca Esophagus
§ For adenocarchinoma
o Chronic gastric reflux (i.e. Barrett’s esophagus)
§ For Squamous cell Ca
o Alcohol
o Cigarette
o Mucosal damage from physical agents → Hot tea, Lye ingestion, Radiation-induced strictures, chronic achlasia
§ Other ingested carcinogens → Nitrates, smoked opiates, Fungal toxins in pickled vegetables
o Plummer – Vinson – Paterson Kelly syndrome (Esophageal Web + glossitis + Iron deficiency)
o Tylosis plamaris et plantaris (Congenital hyperkeratosis and pitting of palms and soles)
o Dietary deficiencies of molybdenum, Zinc, Vitamin A.
§ Celiac sprue
o Chronic Achalasia
Q.49 Common sites for cushing ulcers include all of the following except
a. Esophagus
b. Stomach
c. 1st part of duodenum
d. Distal duodenum
Your Ans. a Correct Ans. d

Solution. Ans-49: (d) Distal duodenum


Ref: Robbin’s 7th/e P. 819
Sol :
§ Cushings ulcers are stress ulcers that areassociated with intracranial injury or an increase in intracranial pressure.
§ About the site of involvement in Cushing ulcer the maximum information I could get was from Robbin’s. it states ‘Gastric,
duodenal and esophageal ulcers arising in patients with interacranial injury, operation or tumors are designated
Cushing ulcer”
§ So the best option to rule out is distal duodenum, as almost all duodenal ulcers are located in the proximal part.
§ Also know
§ Cushing ‘s ulcers are more prone to perforate than other stress ulcers.
Q.50 MHC restriction is a part of all except
a. Antiviral cytotoxic T cell
b. Antibacterial helper T cell/cytotoxic cells
c. Allograft rejection
d. Autoimmune disorder
Your Ans. d Correct Ans. d

Solution. Ans-50: (d) Autoimmune disorder


Ref: Harrison - 207
Sol :
MHC – Class I restriction
1. Graft rejection
2. Cytotoxic cell mediated cytolysis of viral infected or tumor cells.
MHC – Class II restriction
1. Graft versis host response
2. Mixed leukocyte reaction

Q.51 Cardiomyopathy may be seen in all except


a. Duchenne muscular dystrophy
b. Friedrich’s ataxia
c. Type II glycogen storage disease
d. Alkaptonuria
Your Ans. d Correct Ans. d
Solution. Ans-51: (d) Alkaptonuria
Ref: Harrison 17th/e p. 1481
Sol :
§ Cardiomyopathy :
o Cardiomyopathy, which literally means “heart muscle disease”, is the deterioration of the function of the myocardium.
o Cardiomyopathies can generally be categorized into two groups
§ Extrinsic cardiomyopathies
o These are the cardiomyopathies where the primary pathology is outside the myocardium.
o Most cardiomyopathies are extrinsic, because by for the most common cause of cardiomyopathy is ischemia, i.e. ischemic
cardiomyopathy.
o Other important extrinsic cardiomyopathies are alcoholic cardiomyopathy and diabetic cardiomyopathy.
§ Intrinsic cardiomyopathies
o An intrinsic cardiomyopathy is weakness in the muscle of heart that is not due to an identifiable external cause.
o Intrinsic cardiomyopathies are generally classified into three types.
o Dilated cardiomyopathy → Most common intrinsic cardiomyopathy.
o Hypertrophic cardiomyopathy
o Restrictive cardiomyopathy →Least common intrinsic cardiomyopathy
Q.52 Crew haircut appearance in X-ray skull and Gandy gamma bodies are seen in
a. G-6-PD deficiency
b. Hodgkin’s lymphoma
c. Hereditary spherocytosis
d. Sickle cell anaemia
Your Ans. d Correct Ans. d

Solution. Ans-52: (d) Sickle cell anaemia


Ref: Chapman 3rd/e p. 392
Sol :
§ Hair on end appearance is seen in :
o Sickle cell anemia
o Hereditary sperocytosis
o G-6-PD deficiency
§ Among these disorders Gama Gandy bodies are seen in sickle cell anemia.
§ Gamma Gandy bodies are found in spleen.
§ These are nodules that develop through organization of splenic microhemorrhages followed by hemosiderin and calcium
deposition.
§ These are seen in congestive splenomegaly caused by sickle cell anemia, CML, Liver cirrhosis.
Q.53 The peripheral blood smear of a patient shows features of thalasemia, also presented with anaemia. Family
history is also +ve. The investigation done to establish the diagnosis is
a. ESR estimation
b. Blood spherocyte estimation
c. Bone marrow aspiration
d. Hb-electrophoresis
Your Ans. d Correct Ans. d

Solution. Ans-53: (d) Hb-electrophoresis


Ref: Nelson 17th/e p. 1630
Sol :
§ Hemoglobin electrophoresis should always be the first investigation to include/exclude the diagnosis of thalasemia.
§ The level of normal adult hemoglobin HbA is markedly decreased with proportionate increase in HbA2 and HbF.
§ X–ray findings in thalassemia
o Crew cut appearance
Hair on end appearance
Q.54 Sign of shock is
a. Hypotension
b. Bradycardia
c. Polyuria
d. Chest pain
Your Ans. a Correct Ans. a
Solution. Ans-54: (a) Hypotension
Ref: Davidson’s- 177
Sol :
General features of shock common to most types of shock
§ Hypotension (systolic BP < 100 mmHg)
§ Tachycardia with low volume pulse (pulse > 100/min)
§ Sweating
§ Cold, clammy skin (cyanoses peripheries)
§ Rapid, shallow respiration
§ Drowsiness, confusion, irritability
§ Oliguria (urine output < 30 ml/hr)
§ Elevated or reduced CVP
§ Multi organ failure

Q.55 Common cause of chronic renal failure is


a. Hypotension
b. Hypertension
c. Diabetes insipidus
d. Malaria
Your Ans. b Correct Ans. b

Solution. Ans-55: (b) Hypertension


Ref: Davidson’s- 486
Sol :
Common causes of chronic renal failure
Diabetes mellitus 20-40%
Hypertension 5-25%
Glomerular diseases 10-20%
Interstitial disease 5-15%
Renal artery stenosis 5%
Systemic inflammatory diseases 5%
Congenital and inherited (e.g. polycystic kidneys, Alports syndrome) 5%

Q.56 Platelet production (thrombopoiesis) is affected by more than one cytokine. Which of the following sets seems to
be the most important in platelet development?
a. IL-3, granulocyte colony-stimulating factor (G-CSF), and granulocytemacrophage colony-stimulating factor (GM-CSF)
b. IL-4, IL-6, and thrombopoietin
c. Erythropoietin, thrombopoietin, and IL-6
d. IL-6 and thrombopoietin
Your Ans. c Correct Ans. d

Solution. Ans 56 : (d) IL-6 and thrombopoietin.


Ref– Read the text below
Sol:
· Platelet productionis stimulated by multiple cytokines, the most important being IL-6 and the peptide
thrombopoietin.
· IL-3, IL-6, and GM-CSFalso affect megakaryocytes,whereas erythropoietin and G-CSF are almost exclusively related to
erythropoiesis and granulocyte production, respectively.
· IL-4 is also predominantly granulocyte related.
Q.57 Which condition customarily terminates by transition to a blast cell phase with similarities to acute leukemia?
a. Chronic lymphocytic leukemia (B cell type)
b. Chronic myelocytic leukemia
c. Lymphoblastic leukemia
d. Hodgkin’s disease
Your Ans. b Correct Ans. b
Solution. Ans 57 : (b) Chronic myelocytic leukemia.
Ref– Read the text below
Sol:
· The myelocytic leukemia commonlym terminates following transition to an accelerated phase with transformation of the
principal malignant cell to a blastlike form.
· The lymphocytic leukemia involves impaired antibody production and other immune functions, resulting in susceptibility
to severe infections.
· Both forms of leukemialead to increasing accumulation of malignant cells, leading to organ enlargement, especially of the
spleen, liver, lymph nodes, and bone marrow.
· Hodgkin’s diseaseoften presents with superficial nodes found as a new mass.
Q.58 A patient has an X-linked disease. His three sisters do not have the disease.He most likely has
a. A mutant recessive gene on the X chromosome
b. A mutant dominant gene on the X chromosome
c. A mutant recessive gene on the Y chromosome
d. A mutant dominant gene on the Y chromosome
Your Ans. a Correct Ans. a

Solution. Ans 58 : (a) A mutant recessive gene on the X chromosome.


Ref– Read the text below
Sol:
· In males, only one copy of a mutant recessive gene on the X chromosome is sufficient to cause the genetic disease.
· Females, on the other hand, require two copies of the recessive gene.
· If this had been a dominantly inherited condition, chances are that at least one of his sisters would be affected.
Q.59 The fact that type IV osteogenesis imperfecta can be caused by defects on COLIA 1 and COLIA 2 is an example of
a. Gonadal mosaicism
b. Genetic heterogeneity
c. Allelic heterogeneity
d. Polymorphism
Your Ans. c Correct Ans. b

Solution. Ans 59 : (b) Genetic heterogeneity.


Ref– Read the text below
Sol:
· Genetic heterogeneityis defined as a situation in which mutations of different genes produce similar or identical
phenotypes.
· Gonadal mosaicismrefers to mutation affecting some of the germ cells (sperm or eggs).
· Allelic heterogeneityrefers to the state in which multiple alleles at a single locus can produce a disease phenotype or
phenotypes.
Q.60 The FMR1 protein
a. Is expressed in fragile X-associated mental retardation
b. Is normally found in brain and ovaries
c. When defective, is not sufficient to cause the fragile X-associated mental retardation syndrome
d. Is coded by the FMR1 gene which has the (5′-CGG-3′)n repeat segments
Your Ans. a Correct Ans. d

Solution. Ans 60 : (d) Is coded by the FMR1 gene which has the (5′-CGG-3′)n repeat segments.
Ref– Read the text below
Sol:
· The family mental retardation (FMR1) protein is normally expressed in brain and testes.
· It is encoded by the FMR1 gene. Amplification of the CGG region to a repeat number greater than 200 causes
methylation of the CpG island and prevents the FMR1 protein from being expressed.
· This defect is sufficient to cause the fragile X-associated mental retardation syndrome.
Q.61 Which toxicity do all of the first-line TB drugs have in common?
a. Neuropathy
b. Optic neuritis
c. Hepatotoxicity
d. Serious drug-drug interactions
Your Ans. c Correct Ans. c
Solution. Ans 61: (c) Hepatotoxicity
Ref– Read the text below
Sol:
· All the first-line drugs can cause hepatotoxicity, the major toxicity of TB therapy. INH can cause neuropathy (which
can be prevented by co-administration of vitamin B 6 ), ethambutol can cause optic neuritis, rifampin can cause serious
drug-drug interactions (particularly with HIV protease inhibitors), and pyrazinimide causes hyperuricemia (although
this is usually asymptomatic).
Q.62 A medication that is associated with gingivitis
a. Phenytoin
b. Carbamazepine
c. Propranolol
d. Diazepam
Your Ans. a Correct Ans. a

Solution. Ans 62: (a) Phenytoin


Ref– Read the text below
Sol:
· Gingivitis is a form of periodontal disease.
· Periodontal diseaseis when inflammation and infection destroy the tissues that support the teeth, including the gingiva
(gums), the periodontal ligaments, and the tooth sockets (alveolar bone )
Q.63 Vascular abnormalities are a known side effect of several cytotoxic drugs. Which of the following cytotoxic drugs
is most likely to lead to thrombotic micro-angiopathy ?
a. Mitomycin
b. 5-fluorouracil
c. Taxoids
d. Cyclopentenyl cytosine
Your Ans. b Correct Ans. a

Solution. Ans 63: (a) Mitomycin


Ref– Read the text below
Sol:
· The term 'thrombotic microangiopathy' (TMA) describes syndromes of microangiopathic hemolytic anemia,
thrombocytopenia, and variable signs of organ impairment, due to platelet aggregation in the microcirculation.
· Mitomycin-C is, etiologically, the most common causative agent inducing 'thrombotic microangiopathy'in a dose
dependent manner.
Q.64 A fifty-five year old with chronic cardiac failure is on treatment with digoxin and a loop diuretic.This combination
is likely to cause digoxin toxicity by the following mechanism:
a. This combination increases the half-life of digoxin
b. Diuretics decrease potassium levels
c. Frusemide and digoxin interact to form a poisonous compound
d. Digoxin is a competitive inhibitor of frusemide
Your Ans. b Correct Ans. b

Solution. Ans 64: (b) Diuretics decrease potassium levels


Ref– Read the text below
Sol:
· Loop and thiazide diureticsdecrease potassium and magnesium levels, predisposing patients taking both a diuretic and
digoxin to an increased risk of digoxin toxicity.
· Also, amphotericin B (Fungizone), an antifungal, has an additive potassium-lowering effect when given with a thiazide
or loop diuretic.
· Thiazides may increase the blood levels of lithium. Bile acid sequestrants cholestyramine (Questran) and colestipol
(Colestid) decrease the absorption of thiazide diuretics when given concomitantly, while nonsteroidal anti-inflammatory
drugs such as indomethacin (Indocin) may decrease the therapeutic effects of both the loop and thiazide diuretics.
Q.65 Diplopia and ataxia are the most common dose-related adverse effects of :
a. Vigabatrin
b. Carbamazepine
c. Gabapentin
d. Topiramate
Your Ans. b Correct Ans. b
Solution. Ans 65: (b) Carbamazepine
Ref– Read the text below
Sol:
Carbamazepine
· The most common side effects are dizziness, drowsiness, ataxia, diplopia, nystagmus (around 50%), heachache,
tremor (often responds to propranolol), cognitive blunting, decreased libido in men (through lowering of free testosterone levels
secondary to enzyme induction), hyponatremia & water intoxication, sinus node dysfunction in combination with
lithium, lowered response of TSH to TRH.
Q.66 Which drug is currently approved for administration once a week as prophylaxis for MAC in HIV infection?
a. Rifabutin
b. Azithromycin
c. Clarithromycin
d. Ethambutol
Your Ans. c Correct Ans. b

Solution. Ans 66: (b) Azithromycin


Ref– Read the text below
Sol:
The various prophylactic regimens for MAC in HIV infection (usually initiated at CD4 counts :5 50 cells/fLL) include
azithromycin 1200 mg orally every week, clarithromycin 500 mg orally twice a day, and rifabutin 300 mg orally
every day.
Q.67 Cardiotoxicity is a well-known side effect of several cytotoxic drugs. Which of the following cytotoxic drugs is most
likely to lead to long term morbidity due to cardiotoxicity ?
a. Doxorubicin
b. 5-fluorouracil
c. Taxoids
d. Cyclopentenyl cytosine
Your Ans. a Correct Ans. a

Solution. Ans 67: (a) Doxorubicin


Ref– Read the text below
Sol:
· Cardiotoxicityis a well-known side effect of several cytotoxic drugs, especially of the anthracyclines and can lead to
long term morbidity.
· The mechanism of anthracycline induced cardiotoxicity seems to involve the formation of free radicals leading to
oxidative stress.
· This may cause apoptosis of cardiac cells or immunologic reactions.
Q.68 Selegiline :
a. Selectively inhibits MAO-B
b. The most common adverse effect is postural hypotension
c. Cannot be prescribed concurrently with levodopa
d. Cannot be prescribed concurrent with amantadine
Your Ans. a Correct Ans. a

Solution. Ans 68: (a) Selectively inhibits MAO-B


Reference – Read the text below
Sol:
· Selegeline, an MAO-B inhibitorsmay slow disease progression in idiopathic Parkinson’s disease.
· It usually allows dose reduction and prolongs the duration of action of levodopa.
· Oral selegelineis well absorbed (100%) and extensively metabolized in the liver. Rarely hypertension has been reported.
· Amantidine (which stimulates release of endogenous dopamine) potentates its anti-Parkinson effects.
Q.69 An 18-year-old man is admitted to casually in status epilepticus. There is a history of three previous unexplained
blackouts in the last year. The following are appropriate:
a. Intravenous diazepam
b. Intramuscular phenytoin
c. Administration of 24% oxygen
d. Once the acute episode is over oral gabapentin should be commenced.
Your Ans. a Correct Ans. a
Solution. Ans 69: (a) Intravenous diazepam
Reference – Read the text below
Sol:
· Status epilepticus is a medical emergency with a mortially of about 10%.
· Rapid suppression of seizure activitycan usually be achieved with intravenous diazepam (usually formulated as an
emulsion).
· False teeth should be removed, on airway established and oxygen (60%) administered as soon as possible.
Q.70 A 60-year-old diabetic male on an oral hypoglycemic agent develops abnormal liver function tests. Which of the
following agents can cause this finding?
a. Glyburide
b. Metformin
c. Troglitazone
d. Acarbose
Your Ans. b Correct Ans. c

Solution. Ans-70: (c)Troglitazone


Reference – Read the text below
Sol:
· Hepatic dysfunctionhas occurred particularly with the use of troglitazone, necessitating its removal from the market.
· Patients treated with other thiazolidinediones should be monitored for this possibility.
Q.71 Which of the following enzymes can polymerize deoxyribonucleotides into DNA?
a. DNA ligase
b. DNA gyrase
c. RNA polymerase III
d. Reverse transcriptase
Your Ans. d Correct Ans. d

Solution. Ans-71: (d)Reverse transcriptase


Reference – Read the text below
Sol:
· Reverse transcriptase is an RNA-dependent DNA polymerase that can synthesize first a single strand and then
a double-stranded DNA from a single-strand RNA template.
· It was originally found in animal retroviruses. Primase is a DNA-dependent RNA polymerase enzyme that
synthesizes an RNA molecule 10 to 200 nucleotides in length that initiates or "primes" DNA synthesis. DNA ligase
joins DNA fragments and DNA gyrase winds or unwinds DNA.
· Transfer RNA, 5SRNA, and other small RNAs are synthesized by RNA polymerase III (RNA polymerase I synthesizes
ribosomal RNA and RNA polymerase II synthesizes messenger RNA).
Q.72 Which of the following cardiovascular agents is classified chemically as a glycoside?
a. Nifedipine
b. Digoxin
c. Flecainide
d. Cholestyramin
Your Ans. b Correct Ans. b

Solution. Ans-72: (b)Digoxin


Reference – Read the text below
Sol:
· Most glycosides are natural products obtained from plant material. Although there are very few medicinal agents
that are glycosides, the group known as the cardiac glycosides are extremely important and they are widely used for
treating congestive heart failure.
· Digoxin is a cardiac glycoside obtained from Digitalis lanata.
· Other cardiac glycosides include digitoxin, which is obtained from Digitalis purpurea, and ouabain, which is
obtained from Strophanthus gratus.
Q.73 A 16 year old female with Addison’s disease is intolerant of her hydrocortisone treatment which she takes at a
dose of 20mg in the morning and 10mg in the evening. Which of the following doses of prednisolone would
provide an equivalent daily dose as her hydrocortisone ?
a. 5mg
b. 7.5mg
c. 10mg
d. 12.5mg
Your Ans. a Correct Ans. b
Solution. Ans 73: (b) 7.5mg
Reference – Read the text below
Sol:
· The equivalent ratio of prednisolone to hydrocortisone is 1:4. For Dexamethasone to hydrocortisone the ratio is
roughly 1:40.
Q.74 A drug-drug interaction is possible with sildenafil and which of the following agents?
a. Dobutamide
b. Alprostadil
c. Digoxin
d. Nitrates
Your Ans. d Correct Ans. d

Solution. Ans-74: (d)Nitrates


Reference – Read the text below
Sol:
· Sildenafil is effective in many patients with erectile dysfunction. It increases cGMP by inhibiting
phosphodiesterase isoform 5.
· The potentiation of nitrate action by sildenafil is thought to result from both agents increasing the concentration of nitric
oxide, which leads to an increase in cGMP, thereby resulting in hypotension.
Q.75 A 22-year-old female who requests a postcoital contraceptive after being raped would best be treated with which
of the following?
a. Mifepristone
b. Spironolactone
c. Aminoglutethimide
d. Leuprolide
Your Ans. a Correct Ans. a

Solution. Ans-75: (a)Mifepristone


Reference – Read the text below
Sol:
· Mifepristone is structurally related to norethindrone. This compound is classified as a progesterone antagonist
with weak agonistic properties.
· A single dose can function as an emergency postcoital contraceptive. It also can induce an abortion by causing
contraction of the myometrium, which leads to detachment of the embryo.
· The drug is used in single or multiple doses followed by the administration of a prostaglandin to cause the abortion.
· Estrogens used alone or in combination with progestins have also proven effective in postcoital contraception.
Q.76 A 65-year-old diabetic male with erectile dysfunction would be best treated with which of the following?
a. Sildenafil
b. Gossypol
c. Androstenedione
d. Finasteride
Your Ans. a Correct Ans. a

Solution. Ans-76: (a)Sildenafil


Reference – Read the text below
Sol:
· Sildenafil inhibits the action of phosphodiesterase isoform 5, the major phosphodiesterase in the corpus
cavernosum that degrades cGMP. Nitric oxide is released on sexual stimulation.
· This leads to an increase in cGMP. By inhibiting the enzyme, sildenafil causes the accumulation of cGMP.
· This results in vascular smooth-muscle dilation, allowing increased blood flow into the penis followed by erection.
Q.77 What are the most common adverse effects of anticonvulsive drugs?
a. Headache and dizziness
b. Gastrointestinal symptoms
c. Alternation of cognition and mentation
d. All of the above
Your Ans. d Correct Ans. d

Solution. Ans-77: (d)All of the above


Reference – Read the text below
Sol:
· Alternation in cognition and mentation, gastrointestinal symptoms, appetite and body weight, and headache or
dizziness are all common adverse effects of anticonvulsive drugs.
Q.78 Phenoxybenzamine is best described as :
a. Alpha-receptor antagonist
b. Anticholinergic
c. Muscarinic agonist
d. Beta blocker
Your Ans. a Correct Ans. a

Solution. Ans 78: (a) Alpha-receptor antagonist


Ref– Read the text below
Sol:
· An example of a selective alpha-receptor antagonist is phenoxybenzamine
· Phenoxybenzamine belongsto a class of medicines called the alpha adrenergic blockers. It works by blocking alpha
receptors in certain parts of the body. Alpha receptors are present in the muscle that lines the walls of blood vessels.
· When the receptors are blocked by phenoxybenzamine, the muscle relaxes and the blood vessels widen. This
widening of the blood vessels results in a lowering of blood pressure. This medicine can be used to treat high blood pressure.
· This medicine is used in the short term treatment of severe hypertensive episodes associated with
phaeochromocytoma
Q.79 The most useful medication that may be used in absence seizures
a. Phenytoin
b. Primidone
c. Carbamazepine
d. Valproic acid
Your Ans. d Correct Ans. d

Solution. Ans 79: (d) Valproic acid


Ref– Read the text below
Sol:
Valproic acid is a drug of choice for:
· Generalized epilepsy of unknowncause that produces more than one type of seizure.
· Absence seizures.Ethosuximide is the other drug of choice. If a person has absence seizures as well as other types of
seizures, however, valproic acid usually works better because it can control several types of seizures.
· Myoclonic seizures, such as those caused by juvenile myoclonic epilepsy.
· Clonazepam may be used when valproic acid does not work. Valproic acid is also considered a first-line drug for
treating partial seizures.
Q.80 An epileptic on hypertensive treatment develops acute hemorrhagic pancreatitis. Which medication is the most
likely cause ?
a. Propranolol
b. Nifedipine
c. Verapamil
d. Sodium Valproate
Your Ans. d Correct Ans. d

Solution. Ans 80: (d) Sodium Valproate


Ref– Read the text below
Sol:
· Valproate (VPA)-associated pancreatitisappears to be more frequent in young persons (mean age 16.4 years) but may
occur at any age.
· The highest risk appears to exist during the first months of treatment: 43.8% of the cases developed during the first 3
months, and 68.8% developed during the first year. In most patients, the reaction was rapidly reversible when valproate was
discontinued. It could be severe with deaths reported.
· Asymptomatic elevation of serum amylaseisseen in 10%. Awareness of the problem and early discontinuation of VPA
may be effective in preventing serious reactions.
Q.81 The eclipse period of a one-step viral multiplication curve is defined as the period of time between the
a. Uncoating and assembly of the virus.
b. Start of the infection and the first appearance of extracellular virus.
c. Start of the infection and the first appearance of intracellular virus.
d. Start of the infection and uncoating of the virus.
Your Ans. a Correct Ans. c
Solution. Ans 81 : (c) Start of the infection and the first appearance of intracellular virus..
Ref– Read the text below
Sol:
· The period of time between the adsorption and penetration of the virus until the first appearance of
intracellular virus is the eclipse phase.
· For most viruses, nucleic acid and structural protein synthesis is accompanied by the assembly of protein and nucleic
acid complexes.
· The assembly and egress of mature infectious virus mark the end of the eclipse phase of infection, during which
infectious virus cannot be recovered from the infected cell. Nucleic acids from RNA viruses and poxviruses assemble into
nucleocapsids in the cytoplasm.
· For all DNA viruses except poxviruses, viral DNA assembles into nucleocapsids in the nucleus.
Q.82 Viral-induced heart disease is most frequently associated with
a. Arenavirus infections
b. Coxsackievirus infections
c. Echovirus infections
d. Enterovirus infections
Your Ans. b Correct Ans. b

Solution. Ans 82: (b) Coxsackievirus infections.


Ref– Read the text below
Sol:
· Acute inflammation of the heart muscle(myocarditis), its covering membranes (pericarditis), or both can be caused by a
variety of viral agents.
· Group B coxsackievirusesare the most commonly implicated enteroviruses.
· Such infections are usually self-limiting but may be fatal in the acute phase (arrhythmia or heart failure) or progress
to chronic dilated myocardiopathy
Q.83 In most normal persons, what percentage of the total serum immunoglobulin (Ig) is IgG?
a. 25%
b. 50%
c. 60%
d. Over 70%
Your Ans. c Correct Ans. d

Solution. Ans 83 : (d) Over 70%.


Ref– Read the text below
Sol:
· Immunoglobulin (Ig) Gis the predominant immunoglobulin in serum.
· A normal person has approximately 73% IgG, 19% IgA, 7% IgM, 1% IgD, and 0.01% IgE.
Q.84 M. tuberculosis can be found in the sputum of patients with tuberculosis. After digestion of the sputum, isolation
is best accomplished using
a. Löffler’s medium
b. Thayer-Martin agar
c. Thiosulfate citrate bile salts sucrose medium
d. Löwenstein-Jensen medium
Your Ans. d Correct Ans. d

Solution. Ans 84 : (d) Löwenstein-Jensen medium


Ref– Read the text below
Sol:
· Löwenstein-Jensen slants or plates, which are composed of a nutrient base and egg yolk, are used routinely for the
initial isolation of mycobacteria.
· Small inocula of M. tuberculosiscan also be grown in oleic acid albumin media; large inocula can be cultured on simple
synthetic media.
Q.85 Which of the following components determines the class-specific antigenicity of immunoglobulins?
a. J chain
b. T chain
c. Light chain
d. Heavy chain
Your Ans. d Correct Ans. d
Solution. Ans 85 : (d) Heavy chain
Ref– Read the text below
Sol:
· The heavy chain defines the immunoglobulin (Ig) class.
· Gamma chains specify IgG, mu (µ) chains specify IgM, and so on.
· Each of the classescan contain either a kappa or lambda set of light chains. IgA and IgM can contain a J chain.
Q.86 Susceptibility to the yeast Candida can occur in
a. Congenital agammaglobulinemia
b. Congenital thymic aplasia (digeorge syndrome).
c. Common variable hypogammaglobulinemia.
d. Systemic lupus erythematosus (SLE).
Your Ans. b Correct Ans. b

Solution. Ans 86 : (b) Congenital thymic aplasia (digeorge syndrome).


Ref– Read the text below
Sol:
Immunity to Candidais predominantly cell mediated; thus, susceptibility is increased in instances of depressed T-cell
function, as in congenital thymic aplasia (DiGeorge syndrome).
Q.87 Which of the following statements about immunoglobulin M (IgM) is true?
a. It is the reaginic antibody
b. It is important in the first few days of the primary immune response
c. It increases in serum concentration after IgG has reached its peak serum concentration
d. It is the smallest of the immunoglobulin molecules
Your Ans. b Correct Ans. b

Solution. Ans 87 : (b) It is important in the first few days of the primary immune response..
Ref– Read the text below
Sol:
· Immunoglobulin M (IgM),the first antibody to be produced after antigenic stimulation, has a very important role in the
first few days of a primary immune response.
· The largest of the immunoglobulin molecules, it is not involved in allergic (type I hypersensitivity) reactions.
Q.88 Which of the following is the most frequent cause of blood transfusion associated hepatitis?
a. Hepatitis A virus
b. Hepatitis B virus
c. Hepatitis C virus
d. Hepatitis D virus
Your Ans. b Correct Ans. c

Solution. Ans 88 : (c) Hepatitis C virus.


Ref– Read the text below
Sol:
· Hepatitis C virus is an RNA virus in the flavivirus (eg, yellow fever, dengue) family.It has a very simple genome,
consisting of just three structural and five nonstructural genes.
· There are at least six major genotypes, with multiple subtypes. The genotypes have different geographic distributions
and may be associated with differing severity of disease as well as response to therapy.
· Hepatitis C is an insidious disease in that it does not usually cause a clinically evident acute illness. Instead, its
first manifestation (in 25% of those infected) may be the presence of smoldering chronic hepatitis that may ultimately lead to
liver failure.
· Its transmission is less well understood than for hepatitis A, B, and D.
· Hepatitis C was the major cause of posttransfusion hepatitis until a serologic testfor screening blood donors was
developed.
Q.89 A patient presents with severe colitis associated with an overgrowth of Clostridium difficile in the lower bowel.
The most likely cause of this condition was
a. Botulinum food poisoning
b. A stomach ulcer
c. Compromised immune system
d. Antibiotic therapy
Your Ans. d Correct Ans. d
Solution. Ans 89 : (d) Antibiotic therapy. .
Ref– Read the text below
Sol:
· Antibiotic therapycan reduce the normal flora in the bowel thereby allowing pathogenic organisms normally present in low
numbers to overgrow.
· None of the other answers explains the overgrowth of C. difficile.
Q.90 Exotoxin A most closely resembles the action of which of the following?
a. Heat-labile (LT) Escherichia coli
b. Shiga toxin
c. Diphtheria toxin
d. Vibrio cholerae toxin
Your Ans. b Correct Ans. c

Solution. Ans 90 : (c) Diphtheria toxin.


Ref– Read the text below
Sol:
· Both Pseudomonasexotoxin A and Corynebacterium diphtheriae toxin inhibit protein synthesis through the inhibition
of elongating factor (EF-2).
· Diphtheria toxin, produced by toxigenic strains of C. diphtheriae, is the primary virulence factor in clinical disease.
· The toxin is synthesized in precursor form; is released as a 535-amino-acid, single-chain protein; and has an LD50 of
~100 ng/kg of body weight.
· The toxin is produced in the pseudomembranous lesion and is taken up into the bloodstream, through which it is
distributed to all organ systems. Once bound to its cell surface receptor (a heparin-binding, epidermal growth factor–like
precursor), the toxin is internalized by receptor-mediated endocytosis and enters the cytosol from an acidified early endosomal
compartment.
· In vitro, the toxin may be separated into two chains after digestion with serine proteases: the N-terminal A
fragment and the C-terminal B fragment.
Q.91 Which of the following staphylococcal organisms causes subacute bacterial endocarditis that occurs 2 months or
more after heart surgery?
a. Staphylococcus aureus
b. Staphylococcus epidermidis
c. Staphylococcus haemolyticus
d. Staphylococcus saprophyticus
Your Ans. b Correct Ans. b

Solution. Ans 91 : (b) Staphylococcus epidermidis.


Ref– Read the text below
Sol:
· Staphylococcus epidermidisis ubiquitous as part of the normal flora. Organisms are introduced into the host during
invasive procedures.
Q.92 Which one of the following best describes the components of vaccines against Haemophilus influenzae disease?
a. Live, attenuated Haemophilus influenzae
b. Toxoid derived from Haemophilus influenzae
c. Polysaccharide derived from Haemophilus influenzae
d. Polysaccharide derived from Haemophilus influenzae conjugated to a protein antigen
Your Ans. d Correct Ans. d

Solution. Ans 92 : (d) Polysaccharide derived from Haemophilus influenzae conjugated to a protein antigen
Ref– Read the text below
Sol:
· Covalent conjugatesof capsular polysaccharide with diphtheria protein have been developed for Haemophilus
influenzae.
· Unconjugated polysaccharideis weakly immunogenic in children under 2 year of age.
· However, the conjugated vaccine produces higher of titers of antibody, superior responsiveness in
childrenunder 2 years of age, and enhanced efficacy of booster administrations.
Q.93 Which of the following organisms does not produce a toxin as a mechanism of its pathogenesis?
a. Vibrio cho/erae
b. Clostridium tetani
c. Clostridium botulinum
d. Plasmodium falciparum
Your Ans. d Correct Ans. d
Solution. Ans 93: (d) Plasmodium falciparum
Ref– Read the text below
Sol:
· Tetanospasmin, produced by C. tetani,is the cause of the clinical syndrome of tetanus.
· C. botulinum produces botulinum toxinin its pathogenesis. E. coli 0157:H7 produces a shiga-like toxin that causes
diarrhea.
· The major pathogenic factor of V. choerae is a potent exotoxin.
· The causative organisms of malaria, of which Plasmodium falciparum is one, do not produce toxins
Q.94 Which of the following zoonotic gram-negative rods can be commonly acquired by ingestion?
a. Brucella melitensis
b. Pasteurella multocida
c. Capnocytophaga canimorsus
d. Yersinia pestis
Your Ans. d Correct Ans. a

Solution. Ans 94: (a) Brucella melitensis


Ref– Read the text below
Sol:
· Although gastrointestinal tularemia can be acquired by ingestion of F. tularensis organisms, Brucella melitensis
is commonly acquired through the ingestion of unpasteurized goat's milk cheese
Q.95 Which of the following conditions is a major risk factor for sepsis and death with exposure to Capnocytophaga?
a. HIV
b. Splenectomy
c. Eczema
d. Iron overload
Your Ans. b Correct Ans. b

Solution. Ans 95: (b) Splenectomy


Ref– Read the text below
Sol:
Splenectomized patientsare at high risk for disseminated infection with this organism and often present with OIC and sepsis
after a dog.
Q.96 Which one of the following best describes the Sabin polio vaccine?
a. Provides little gastrointestinal immunity
b. Prepared with inactive virus
c. Administered by injection
d. Carries a small risk of causing disease
Your Ans. d Correct Ans. d

Solution. Ans 96 : (d) Carries a small risk of causing disease


Ref– Read the text below
Sol:
The main disadvantage of attenuated live virus is the small risk of infection, estimated to be 1 per 2.4 million
doses.
Q.97 A commercial vaccine consisting of virion subunits prepared by recombinant technology exists for
a. Hepatitis B virus
b. Rabies virus
c. Rotavirus
d. Varicella-zoster virus
Your Ans. a Correct Ans. a

Solution. Ans 97 : (a) Hepatitis B virus.


Ref– Read the text below
Sol:
· Both the Recombivax-HB and Engerix-B vaccines for protection from hepatitis B virus contain the virus surface
antigen prepared from yeast using recombinant DNA technology.
Q.98 Which of the following causes a nosocomial infection with the potential to cause serious respiratory disease in an
infant pediatric ward?
a. Adenovirus
b. Picornavirus
c. Coxsackie A virus
d. Respiratory syncytial virus
Your Ans. d Correct Ans. d

Solution. Ans 98 : (d) Respiratory syncytial virus


Ref– Read the text below
Sol:
· Respiratory syncytial virusis a serious respiratory disease pathogen for infants and has been associated with hospital-
acquired (nosocomial) infections.
Q.99 Which of the following is true of Clostridium tetani, the causative organism of tetanus?
a. It is a gram-negative rod
b. It forms spores that are heat resistant
c. It is an obligate intracellular organism
d. It is best cultured on Thayer-Martin agar
Your Ans. b Correct Ans. b

Solution. Ans 99: (b) It forms spores that are heat resistant
Ref– Read the text below
Sol:
· C. tetani is an anaerobic, motile, gram-positive spore-forming rod. The spores are resistant to both heat and
many detergents
Q.100 Which pathogen is the agent of "woolsorter's disease"?
a. Coxiella burnetii
b. Brucella ovis
c. Brucella melitensis
d. Bacillus anthracis
Your Ans. d Correct Ans. d

Solution. Ans 100: (d) Bacillus anthracis


Ref– Read the text below
Sol:
· Pulmonary anthraxis known as "woolsorter's disease," as this condition often occurred in farm workers around massive
amounts of sheep wool containing B. anthracis spores.
· Brucella melitensis can cause disease (brucellosis) in humans after exposure to goats or sheep. B. ovis
primarily causes genital diseases in rams.
Q.101 The main law which deals with procedures to be followed in a criminal case is
a. Cr. P.C.
b. Indian contract Act
c. Indian evidence Act
d. I.P.C.
Your Ans. a Correct Ans. a

Solution. Ans 101: (a)Cr. P.C.


Ref:Read the text below
Sol:
· All law can broadly be divided in two- substantive law and procedural law. While substantive law deals with rights
and duties, procedural law relates to procedures.
Indian Penal Code and Indian Contract Act are mainly substantive laws, while criminal procedure code (Cr. P.C.) and Indian
Evidence Act are mainly procedural laws.
Ans 101: (a)Cr. P.C.
Ref:Read the text below
Sol:
· All law can broadly be divided in two- substantive law and procedural law. While substantive law deals with rights
and duties, procedural law relates to procedures.
Indian Penal Code and Indian Contract Act are mainly substantive laws, while criminal procedure code (Cr. P.C.) and Indian
Evidence Act are mainly procedural laws.
Q.102 All of the following are other names of hypostasis except :-
a. Algor mortis
b. Postmortem staining
c. Livor mortis
d. Suggilation
Your Ans. a Correct Ans. a

Solution. Ans 102: (a)Algor mortis


Ref:Read the text below
Sol:
Hypostasis is also known as postmortem staining, cadaveric lividity, suggilations, livor mortis, vibices,& darkening of death.
Ans 102: (a)Algor mortis
Ref:Read the text below
Sol:
Hypostasis is also known as postmortem staining, cadaveric lividity, suggilations, livor mortis, vibices,& darkening of death.
Q.103 A 20-year-old young female comes to a male gynecologist for pelvic examination. The consent for examination is :
a. Implied consent
b. Written consent
c. Verbal consent
d. Informed consent
Your Ans. a Correct Ans. a

Solution. Ans 103: (a)Implied consent


Ref:Read the text below
Sol:
· Consent that is inferred from signs, actions, or facts, or by inaction or silence.
· Implied consent differs from express consent, which is communicated by the spoken or written word.
· Implied consent is a broadly based legal concept. Whether it is as valid as express consent depends on the situation and
the applicable law. For example, the owner of a car generally is liable for an accident caused by someone who drove that car
with his or her consent. In many states, that consent can be express or implied, and implied consent may arise from
seemingly innocuous actions.
Q.104 Which of the following statement about the chilotic line are correct ?
a. It is present on the femur and is useful in the determination of race.
b. It is present on the femur and is useful in the determination of sex.
c. It is present on the hipbone and is useful in the determination of race.
d. It is present on the hipbone and is useful in the determination of sex.
Your Ans. d Correct Ans. d

Solution. Ans 104: (d)It is present on the hipbone and is useful in the determination of sex.
Ref:Read the text below
Sol:
CHILOTIC LINE
Anthropometric line extendingfrom the posterior aspect of the iliopectineal eminence to the closest point on the anterior
auricular margin (the pelvic segment) and then to the iliac crest (the sacral portion); because the pelvic segment is predominant
in females and the sacral segement is predominant in males, the chilotic index may be used in forensic practice to identify the
sex of human skeletal remains, although studies suggest its reliabliity is limited.
Q.105 A district consumer redressal forum can entertain claims for compensation up to maximum of
a. Rs. 50,000
b. Rs. 1 Lakh
c. 5 Lakh
d. Rs. 20 Lakh
Your Ans. d Correct Ans. d

Solution. Ans 105: (d)Rs. 20 Lakh


Ref:Read the text below
Sol:
Previously, the district forum could entertain claims for only upto 5 lakh, but by an amendment of 2002, which came into
force with effect from 15 March 2003, the jurisdiction is now 20 lakh [Section 11(1) of Consumer Protection Act 1986]
Q.106 Ossificatin center for lunate appears at :
a. Birth
b. 2 years
c. 4 years
d. 8 years.
Your Ans. c Correct Ans. d

Solution. Ans 106: (d)8 years.


Ref:Read the text below
Sol:
· The appearance of ossification centres in >50% of children for Lunate is at the age of 6-7 years in Males and at 5-
6 years in Females.
· 100% appearance is seen at 9-10 years in Males and at 7-8 years in Females
Q.107 Anterior fontanelle closes :
a. Just after birth
b. At 2 months
c. By 8 months
d. By 18 months
Your Ans. d Correct Ans. d

Solution. Ans 107: (d)By 18 months


Ref:Read the text below
Sol:
· There are normally two fontanels, both in the midline of the skull, one (the anterior fontanel) well in front of
the other (the posterior fontanel).
· The posterior fontanel closes first, at latest bythe age of 8 weeks in a full-term baby.
· The anterior fontanel closes at 18 months of age on the average but it can close normally as early as 9 months.
If fontanels close too early or too late, that may be a sign of a problem.
Q.108 Testing for the presence of air in the middle ear is
a. Fodere’s test
b. Ploucquet’s test
c. Breslau’s second life test
d. Wreden’s test
Your Ans. d Correct Ans. d

Solution. Ans 108: (d)Wreden’s test


Ref:Read the text below
Sol:
· During embryonic life, the middle ear contains gelationous tissue.
· During afforts of breathing, some air enters the middle ear through the Eustachian tube.
Thus if the air is found in the middle ear, it is quite reasonable to suppose that the child had been born alive.
Ans 108: (d)Wreden’s test
Ref:Read the text below
Sol:
· During embryonic life, the middle ear contains gelationous tissue.
· During afforts of breathing, some air enters the middle ear through the Eustachian tube.
Thus if the air is found in the middle ear, it is quite reasonable to suppose that the child had been born alive.
Q.109 The age of criminal liability is
a. 12 years
b. 14 years
c. 16 years
d. 17 years
Your Ans. a Correct Ans. a

Solution. Ans 109: (a)12 years


Ref:Read the text below
Sol:
· The age of criminal liability is 12 years.
· In the words of section 3 of the Juveniles Act, "It shall be conclusively presumed that no child under the age of 12 years
can be guilty of any offence."
Q.110 Exposure and abandonment of child under twelve years by his parents may lead to their prosecution under section
a. 301 IPC
b. 317 IPC
c. 318 IPC
d. 363 IPC
Your Ans. c Correct Ans. b

Solution. Ans 110: (b)317 IPC


Ref:Read the text below
Sol:
SECTION 317
Neglignece on the part of the father towards the son can be punished under section 317.
Exposure and abandonment of child under twelve years, by parent or person having care of it.
Whoever being the father or mother of a child under the age of twelve years, having the care of such child, shall expose or leave
such child in any place with the intention of wholly abandoning such child, shall be punished with imprisonment of either
description for a term which may extend to seven years; or with fine, or with both.
Explanation.- this section is note intended to prevent the trial of the offender for murder or culpable homicide, as the case may
be, if the child die in consequence of exposure.
Ans 110: (b)317 IPC
Ref:Read the text below
Sol:
SECTION 317
Neglignece on the part of the father towards the son can be punished under section 317.
Exposure and abandonment of child under twelve years, by parent or person having care of it.
Whoever being the father or mother of a child under the age of twelve years, having the care of such child, shall expose or leave
such child in any place with the intention of wholly abandoning such child, shall be punished with imprisonment of either
description for a term which may extend to seven years; or with fine, or with both.
Explanation.- this section is note intended to prevent the trial of the offender for murder or culpable homicide, as the case may
be, if the child die in consequence of exposure.
Q.111 “Propaganda” is characterized by
a. Knowledge is actively acquired.
b. Reflective behaviour.
c. Behaviour centered.
d. Stimulate primitive desires.
Your Ans. c Correct Ans. d

Solution. Ans 111: (d) Stimulate primitive desires.


Ref– Read the text below
Sol:

Q.112 ‘Yuzpe and Lance’ method is used for :-


a. Menstrual regulation
b. Non-surgical abortion in the first 12 weeks of pregnancy
c. Emergency contraception using OCPs
d. Mortality rates.
Your Ans. c Correct Ans. c
Solution. Ans 112: (c) Emergency contraception using OCPs
Ref– Read the text below
Sol:
· The Yuzpe Regimen is a method of emergency contraception using a combination of estrogen and progestogen hormones
and started within 72 hours of sexual intercourse. It has been superseded by a progestogen-only hormonal regimen.
· This regimen allows a woman who has had unprotected sex to avoid pregnancy by taking 12 hours apart two sufficient
doses of estrogen and progestogen hormones. The sooner this is started, the more effective it is and the
effectiveness more than 72 hours after sexual intercourse is greatly reduced.
The licensing for emergency Intrauterine device (IUD) insertion allows for up to 7 days and it is highly effective for up to 7 days
after the time of ovulation, which may extend more than five days after the relevant risky intercourse.
Q.113 Denominator of positive predictive value is
a. True positive + False positive
b. True positive + False negative
c. False positive + True negative
d. False positive + False negative
Your Ans. a Correct Ans. a

Solution. Ans 113: (a) True positive + False positive


Ref– Read the text below
Sol:
Sensitivity and specificity are terms used to describe the value of tests. This calculator will determine these
values based on true positives, false positives, true negatives, and false negatives.

Disease Present No Disease


Positive test a b
Negative test c d
Sensitivity= a/(a+c)
Specificity = d/(b+d)
Positive Predictive Value = a/(a+b)
Negative Predictive Value = d/(c+d)
Q.114 In the context of epidemiology, the following are important criteria for making casual inferences except.
a. Strength of association
b. Consistency of association
c. Coherence of association
d. Predictive value
Your Ans. d Correct Ans. d

Solution. Ans 114: (d) Predictive value


Ref– Read the text below
Sol:
PREDICTIVE VALUE:
The predictive value of a test is a measure (%) of the times that the value (positive or negative) is the true value, i.e. the percent
of all positive tests that are true positives is the Positive Predictive Value.

__TP___ X 100 = Predictive Value of a Positive Result (%)


TP + FP

__TN___ X 100 = Predictive Value Negative Result (%)


FN + TN

Q.115 The WHO has set the SARS containment period at:
a. 10 days
b. 20 days
c. 28 days
d. 30 days
Your Ans. a Correct Ans. b
Solution. Ans 115: (b) 20 days
Ref– Read the text below
Sol:
· The mortality rate of SARS is higher than that of non-H5N1 influenza strains or other common respiratory tract
infections.
· The overall mortality rate of SARS has been approximately 10%. According to the CDC and the WHO, the death rate
among individuals older than 65 years exceeds 50%.
The WHO has set the SARS containment period at 20 days. If no new cases of SARS are reported in a given area over a 20-day
period, given the relatively short incubation period of the disease, the WHO considers SARS infections in that area to be
contained.
Q.116 The period between exposure and the onset of infectiousness is:
a. Serial interval.
b. Infectious (communicable) period
c. Infectious (communicable) period
d. Latent period
Your Ans. d Correct Ans. d

Solution. Ans 116: (d) Latent period


Ref– Read the text below
Sol:
· Incubation period: time from exposure to development of disease. In other words, the time interval between invasion by
an infectious agent and the appearance of the first sign or symptom of the disease in question.
· Latent period: the period between exposure and the onset of infectiousness (this may be shorter or longer than the
incubation period).
Ans 116: (d) Latent period
Ref– Read the text below
Sol:
· Incubation period: time from exposure to development of disease. In other words, the time interval between invasion by
an infectious agent and the appearance of the first sign or symptom of the disease in question.
· Latent period: the period between exposure and the onset of infectiousness (this may be shorter or longer than the
incubation period).
Q.117 As per the ‘National Socio-Demographic Goals for 2010’, Make school education up to age 14 free and compulsory,
and reduce drop outs at primary and secondary school levels to below:
a. 10 percent for both boys and girls.
b. 20 percent for both boys and girls.
c. 30 percent for both boys and girls.
d. 90 percent for both boys and girls.
Your Ans. a Correct Ans. b

Solution. Ans 117: (b) 20 percent for both boys and girls.
Ref– Read the text below
Sol:
National Socio-Demographic Goals for 2010
· Address the unmet needs for basic reproductive and child health services, supplies and infrastructure.
· Make school education up to age 14 free and compulsory, and reduce drop outs at primary and secondary school levels to
below 20 percent for both boys and girls.
· Reduce infant mortality rate to below 30 per 1000 live births.
· Reduce maternal mortality ratio to below 100 per 100,000 live births.
· Achieve universal immunization of children against all vaccine preventable diseases.
· Promote delayed marriage for girls, not earlier than age 18 and preferably after 20 years of age.
· Achieve 80 percent institutional deliveries and 100 percent deliveriesby trained persons.
· Achieve universal access to information/counseling, and services for fertility regulation and contraception with
a wide basket of choices.
· Achieve 100 per cent registration of births, deaths, marriage and pregnancy.
· Contain the spread of Acquired Immunodeficiency Syndrome (AIDS), and promote greater integration between the
management of reproductive tract infections (RTI) and sexually transmitted infections (STI) and the National AIDS Control
Organisation.
· Prevent and control communicable diseases.
· Integrate Indian Systems of Medicine (ISM) in the provision of reproductive and child health services, and in reaching out to
households.
· Promote vigorously the small family norm to achieve replacement levels of TFR.
Bring about convergence in implementation of related social sector programs so that family welfare becomes a
people centred programme.
Q.118 ASHA must primarily be a woman resident of the village – married/ widowed/ divorced, preferably in the age:
a. 20 to 45 years
b. 25 to 35 years
c. 25 to 45 years
d. 20 to 35 years
Your Ans. b Correct Ans. c

Solution. Ans 118: (c) 25 to 45 years


Ref– Read the text below
Sol:
Following are the key components of ASHA:
· ASHA must primarily be a woman resident of the village – married/ widowed/ divorced, preferably in the age group of 25 to
45 years.
· She should be a literate woman with formal education up to class eight. This may be relaxed only if no suitable person
with this qualification is available.
· ASHA will be chosen through a rigorous process of selection involving various community groups, self-help groups,
Anganwadi Institutions, the Block Nodal officer, District Nodal officer, the village Health Committee and the Gram
Sabha.
· Capacity building of ASHA is being seen as a continuous process. ASHA will have t undergo series of training episodes to
acquire the necessary knowledge, skills and confidence for performing her spelled out roles.
· The ASHAs will receive performance-based incentivesfor promoting universal immunization, referral and escort
services for Reproductive & Child Health (RCH) and other healthcare programmes, and construction of household
Q.119 Suspected cause preceding the observed effect is an example for
a. Temporal association
b. Specificity of association
c. Strength of association
d. Biological plausibility
Your Ans. a Correct Ans. a

Solution. Ans 119: (a) Temporal association


Ref– Read the text below
Sol:
Epidemiologists judge the causality of an epidemiological association based on the following guidelines:
· Temporal Association. Exposure must precede the disease, and in most epidemiological studies this can be inferred. In
studies where exposure and disease are measured simultaneously or exposure is measured after the occurrence of disease, the
temporal association should be evaluated.
· Specificity.
· Consistency.
· Biological Plausibility.
· Strength of Association.
· Dose-Response Relationship.
Reversibility.
Q.120 Typhoid oral vaccine is given
a. 1,3,5 days
b. 1,2,3 days
c. 1,2,4 days
d. 1,7,14 days
Your Ans. d Correct Ans. a

Solution. Ans 120: (a) 1,3,5 days


Ref– Read the text below
Sol:
· The typhoid oral vaccine is given in a series of 4 capsules that are taken 1 per day on alternating days (days 1, 3, 5,
and 7).
· On this alternating-day schedule, you will take 1 capsule every 48 hours for 7 days.
· You must take each capsule according to the recommended schedule for this vaccine to be effective.
Q.121 Which one of the following tests is not used for checking quality of pasteurization of milk
a. phosphatase test
b. Standard plate count
c. Coliform count
d. Orthotoludine test
Your Ans. d Correct Ans. d
Solution. Ans 121: (d) Orthotoludine test.
Ref– Read the text below
Sol:
To check the efficiency of pasteurization
Phosphatase test
To check the efficiency of pasteurization. The test is used to detect inadequate pasteurization.
Standard plate count
Bacteriological quality of pasteurized milk is determined by the standard plate count
Coliform count
Their presence in pasteurized milk is an indication either of improper pasteurization or post-
pasteurization contamination.
Q.122 The size of sand particles in rapid filters is
a. 0.6-2.0m
b. 2.5-0.45mm
c. 4.1-5.2mm
d. 0.4 – 0.7 mm
Your Ans. d Correct Ans. d

Solution. Ans 122: (d) 0.4 – 0.7 mm


Ref– Read the text below
Sol:
· The Rapid Sand Filter (RSF) differs from the Slow Sand Filterin a variety of ways, the most important of which are the
much greater filtration rate and the ability to clean automatically using backwashing.
· The mechanism of particle removal also differs.
· The filter media is the important component of the filter which actually removes the particles from the water being treated.
Filter media is most commonly sand, though other types of media can be used, usually in combination with sand.
· The sand used in rapid sand filters is coarser than the sand used in slow sand filters. This larger sand has larger pores
which do not fill as quickly with particles removed from the water.
· Coarse sand also costs less and is more readily available than the finer sand used in slow sand filtration. The filter
sand used in rapid sand filters is prepared from stock sand specifically for the purpose.
Most rapid sand filters contain 60 to 75 cm thickness of sand, but some newer filtersare deeper. The sand used as
filter media in RSF is generally of effective size of 0.4 to 0.7 mm and uniformity coefficient of 1.3 to 1.7. The standing water
depth over filter varies between 1.0 and 2.0 m.
Q.123 Numerator for calculating Perinatal mortality rate is:
a. All infant deaths less than or equal to 7 days of age
b. All infant deaths less than or equal to 28 days of age
c. All infant deaths less than or equal to 1 year of age
d. All infant deaths between 28 days to 1 yearof age
Your Ans. a Correct Ans. a

Solution. Ans 123: (a) All infant deaths less than or equal to 7 days of age
Ref– Read the text below
Sol:
· Perinatal mortality (PNM), also perinatal death, refers to the death of a fetus or neonate and is the basis to calculate
the perinatal mortality rate.
· Variations in the precise definition of the perinatal mortality exist specifically concerning the issue of inclusion or
exclusion of early fetal and late neonatal fatalities.
· The WHO's definition is "number of stillbirths and deaths in the first week of life per 1,000 live births",but other
definitions have been use
Q.124 Human development index includes all of the following except :
a. Longevity
b. Disability
c. Literacy
d. Income
Your Ans. b Correct Ans. b

Solution. Ans 124: (b) Disability


Ref– Read the text below
Sol:
HUMAN DEVELOPMENT INDEX
· Human development index (HDI) is defined as "a composite index combining indicators representing three dimensions -
longevity (life expectancy at birth); knowledge (adult literacy rate and mean years of schooling); and income (real GDP per capita
in purchasing power parity in US dollars)"
Q.125 Isolation period for mumps is
a. Until fever subsides
b. Until pain subsides
c. Until swelling subsides
d. Until headache subsides
Your Ans. a Correct Ans. c

Solution. Ans 125: (c) Until swelling subsides


Ref– Read the text below
Sol:
Mode of transmission of mumps
· Transmissionoccurs through via respiratory aerosols and respiratory droplet spread or by direct contact with contaminated
saliva.
· Mumps is communicable from six to seven days before to nine days after the onset of parotitis.
· Asymptomatic and inapparent cases can also be infectious.
Q.126 Standardized mortality rate is
a. Standardized for age
b. Standardized for disease
c. Standardized for regions
d. Standardized for a particular time period
Your Ans. c Correct Ans. a

Solution. Ans 126: (a) Standardized for age


Ref– Read the text below
Sol:
· The standardized mortality ratio or SMR in epidemiologyis the ratio of observed deaths to expected deaths according to
a specific health outcomein a populationand serves as an indirect means of adjusting a rate.
· The figure for observed deaths is usually obtained for a particular sampleof a population. The figure for expected
deaths reflects the number of deaths for the larger population from which the study sample has been taken e.g. national level of
mortality attributed to a particular health outcome.
· The calculation used to determine the SMR is simply: number of observed deaths/number of expected deaths.
Q.127 The electrostatic precipitator are used in the prevention of
a. Radiation risks
b. Air pollution emissions
c. Mosquito nuisance
d. Diffuse vibrations
Your Ans. b Correct Ans. b

Solution. Ans 127: (b) Air pollution emissions


Ref– Read the text below
Sol:
· An electrostatic precipitator (ESP), or electrostatic air cleaner is a particulate collection device that removes particles
from a flowing gas (such as air) using the force of an induced electrostatic charge.
· Electrostatic precipitatorsare highly efficient filtration devices that minimally impede the flow of gases through the
device, and can easily remove fine particulate matter such as dust and smoke from the air stream.
· In contrast to wet scrubbers which apply energy directly to the flowing fluid medium, an ESP applies energy only to the
particulate matter being collected and therefore is very efficient in its consumption of energy (in the form of electricity).
Q.128 World “No tobacco” day is
a. May 31st
b. April 5th
c. December 11th
d. November 14th
Your Ans. a Correct Ans. a

Solution. Ans 128: (a) May 31st


Ref– Read the text below
Sol:
· World No Tobacco Day is observed around the world every year on May 31. It is meant to encourage a 24-hour period
of abstinence from all forms of tobacco consumption across the globe.
· The day is further intended to draw global attention to the widespread prevalence of tobacco use and to negative
health effects, which currently lead to 5.4 million deaths worldwide annually.
Q.129 For radical treatment of malaria in adult in low risk area, primaquine is given in dosage of
a. 15mg per day for 3 days
b. 10mg per day for 7 days
c. 15 mg per day for 5 days
d. 5 mg per day for 10 days
Your Ans. c Correct Ans. c

Solution. Ans 129: (c) 15 mg per day for 5 days


Ref– Read the text below
Sol:
Radical Treatment:
Low Risk Area
Plasmodium vivax

Chloqouine Phosphate Primaquine


Age in year 150 mg base 2.5 mg base
Single dose Daily dose for 5 days

<1 Mg base Mg base


75 Nil
1-4 150 2.5
5-8 300 5.0
450 10.0
9-14 600 15.0
15 & above

Q.130 Purified chick embryo cell vaccine is prepared from


a. Wild virus
b. Live attenuated virus
c. Street Virus
d. Fixed Virus
Your Ans. d Correct Ans. d

Solution. Ans 130: (d) Fixed Virus


Ref– Read the text below
Sol:
Purified chick embryo cell vaccine (PCECV- RabipurTm)
· PCECVis prepared from fixed rabies virus strain FLURY LEP grown in primary cultures of chicken fibroblasts.
· The vaccine was first marketed in 1984 and is now available in more than 70 countries and more than 30 million doses of
PCECV have been administered worldwide.
· It is available as a single dose vial containing lyophilised vaccine that is reconstituted with diluent to a final volume of 1
ml.
· Various studies have shown that PCECV is atleast as effective as the HDCV16-18. It is cheaper than HDCV.
Q.131 Features of moderately retracted tympanic membrane are all except
a. Handle of malleus appearance foreshortened
b. Cone of light is absent or interrupted
c. Lateral process of malleus becomes more prominent
d. Lateral process of malleus becomes less prominent
Your Ans. c Correct Ans. d

Solution. Ans-131: (d) Lateral process of malleus becomes less prominent


Ref: Dhingra -61-62
Sol :
Retracted Tympanic membrane –
§ It is the result of negative intratympanic pressure when Eustachian tube is blocked
Characteristics
§ It appears dull and lusterless
§ Cone of light is absent or interrupted
§ Handel of malleus appears foreshortened
§ Lateral process of malleus becomes more prominent
§ Anterior and posterior malleal folds become sickle shaped.
§ It is immobile or has limited mobility when tested with pneumatic otoscope or siegle’s speculum.
Q.132 A 5 year old boy has been diagnosed to have posterior superior retraction pocket. All would constitute part of the
management except :
a. Audiometry
b. Mastoid exploration
c. Tympanoplasty
d. Myringoplasty
Your Ans. d Correct Ans. b

Solution. Ans-132: (b) Mastoid exploration


Ref: Scott’s Brown -3430
Sol :

Q.133 An old man who is endentulous squamous cell CA in bucal mucosa that has developed infiltrated to the alveolus.
Following is not indicated in treatment :
a. Radiotherapy
b. Segment mandibulectomy
c. Marginal mandibulectomy involving removal of outer table only
d. Marginal mandibulectomy involving removal of upper half of mandible
Your Ans. a Correct Ans. c

Solution. Ans-133: (c) Marginal mandibulectomy involving removal of outer table only
Ref: Schwartz 9/e, P. 494
Sol :
§ “In the edentulous patient, the alveolar process is often reabsorbed & tumours arising from the alveolar crest often
invade into the cortical bone (of mandible) by direct extension”.
§ Mandibulectomy should be done.
§ “Treatment of lesion of the alveolar mucosa frequently require reaction of the underlying bone”.
Q.134 All are true about Laryngomalacia except :
a. Most common congenital anomaly of larynx
b. Stridor disappear on supine position
c. Manifest many weak after birth
d. It needs not treatment in most of the cases
Your Ans. b Correct Ans. b
Solution. Ans-134: (b) Stridor disappear on supine position
Ref: Dhingra 5/e Pg-314
Sol :
o Laryngomalacia is most common congenital anomaly of larynx
o As stridor is exacerbated by lying the child in supine postion
o Manifests many weeks after birth
o “Infants with laryngo malacia usually have no sign of respiratory abnormality at birth – Inspiratory stridor typically
develops after a few days or weeks”.
Q.135 A 2 years male boy presenting with sudden severe dyspnoea, most common cause is :
a. Foreign body
b. Bronchiolits
c. Asthmatic Attack
d. None
Your Ans. a Correct Ans. a

Solution. Ans-135: (a) Foreign body


Ref: Scotts Brown - 1117
Sol :
§ In case of stridor with acute airway obstruction (i.e. dyspnsea) always history of any foreign body ingestion should
be taken.
Q.136 Metastasis of carcinoma buccal mucosa goes to :
a. Regional lymph node
b. Liver
c. Heart
d. Brain
Your Ans. a Correct Ans. a

Solution. Ans-136: (a) Regional lymph node


Ref: Dhingra 5/e Pg – 682
Sol :
§ Patterns of spread of buccal mucosal carcinoma
§ Early lesion is discrete and exophytic.
§ As it enlarges, it penetrates the underlying muscle and eventually extends to skin.
§ Peripheral growth occurs into gingivobuccal gutters and into gingiva and underlying bone.
§ Lymphatic spread is first to Submandibular and subdigastric nodes.
§ Distant metastasis is rare.
§ Tumours of buccal mucosa
§ “Tumors in this area have a propensity to spread locally and to metastasize to regional lymphatic’s”.
Q.137 Ludwig’s angina is characterized by all the following except :
a. Cellulitis of the floor of the mouth
b. Caused by anaerobic organisms
c. Aphthous ulcers in the pharynx
d. Infection spreads to retropharyngeal space
Your Ans. c Correct Ans. c

Solution. Ans-137: (c) Aphthous ulcers in the pharynx


Ref: Read the text below
Sol :
Ludwig’s angina
· Cellulitis of the floor of the mouth
· Caused by anaerobic organisms
· Infection spreads to retropharyngeal space
Q.138 After laparoscopic appendectomy, patient had fall from bed on her nose after which she had swelling in nose and
slight difficulty in breathing. Next step in management :
a. IV antibiotics for 7-10 days
b. Observation in hospital
c. Surgical drainage
d. Discharge after 2 days and follow up of the patient after 8 weeks
Your Ans. c Correct Ans. c
Solution. Ans-138: (c) Surgical drainage
Ref: Tuli 1/e, p 148
Sol :
§ The patient in the question had fall from bed following which there is a swelling in nose and slight difficulty in
breathing.
§ This patient has probably had septal haematoma which should be drained immediately under LA.
Q.139 Most difficult site to remove cholesteatoma in sinus tympani is related with
a. Anterior facial ridge
b. Posterior facial ridge
c. Epitympanum
d. Hypotympanum
Your Ans. c Correct Ans. b

Solution. Ans-139 : (b) Posterior facial ridge


Ref: Scott’s Brown -3113
Sol :
§ The sinus tympani (Posterior facial ridge) is the posterior extension of the mesotympanum and lies deep to both
the promontory and facial nerve
§ The medial wall of sinus tympani becomes continuous with the posterior portion of the medial wall of the tympanic cavity.
§ This is the worst region for access because it is above pyramid, posterior to intact stapes and medial to facial
nerve.
Q.140 A lady has B/L hearing loss since 4 years which worsened during pregnancy. Type of impedance audiometry graph
will be :
a. Ad
b. As
c. B
d. C
Your Ans. b Correct Ans. b

Solution. Ans-140: (b) As


Ref: Read the text below
Sol :
Curves of impedence audiometry :
Type of curve Condition
A curve Normal
(Normal peak height and pressure. Eustachian tube obstruction
Otosclerosis
As curve(It is also a variant of normal tympanogram but may be Tumors of middle ear
shallow) Fixed malleus syndrome
Tympanosclerosis
Ossicular discontinuity
Ad curve
Post Stapedectomy
(Variant of normal with high peak)
Monometric ear drum
Fluid in middle ear
B curve
Secretory otitis media
(Flat or dome shaped curve)
Tympanic membrane perforation
Indicating lack of compliance
Grommet in ear
Retracted tympanic membrane
C curve
Faulty function of Eustachian tube/Eustachian tube
(negative peak pressure)
obstruction

Q.141 Band shaped keratopathy is associated with


a. Amiodarone
b. Hyperparathyroidism
c. Amyloidosis
d. Hypothyroidism
Your Ans. b Correct Ans. b
Solution. Ans-141: (b) Hyperparathyroidism
Ref: Parsons Disease of the Eye, 20th ed, Page-201
Sol :
§ It occurs due to the deposition of hyaline infiltration of superficial parts of stroma followed by deposition of calcareous
salts.
§ A whitish band appears in the interpalpebral area commencing at the inner and outer sides, progressing until it forms a
continuous band across the cornea, interspersed with round holes or cleaves within the band itself.
§ It occurs in :
o Chronic uveitis especially in blind shrunken eyes and Still’s disease.
o Aphakic eyes following vitrectomy with silicone oil.
o Hyperparathyroidism
o Vitamin D toxicity
Sarcoidosis
Q.142 Laser iridotomy is indicated for
a. Open angle glaucoma
b. Closed angle glaucoma
c. Congenital glaucoma
d. Neovascular glaucoma
Your Ans. a Correct Ans. b

Solution. Ans-142: (b) Closed angle glaucoma


Ref: Kanski 22
Sol :
§ PACG is a conditionin which the peripheral iris moves forwards to block the openings of trabecular meshwork at the
angle causing a rise of intraocular pressure.
§ Treatment of choice : Nd YAG laser iridotomy-It has been advocated as initial treatment but may be difficult until the
corneal thickening and iris congestion have settled (usually 48 hrs.)
Q.143 A 8 year male boy suffered injury to left eye with suspected foreign body from chisel hammer, all of the following
tests can help localize a metallic intraocular foreign body except :-
a. X-ray orbit
b. B-scan of eye
c. Visual field testing
d. CT scan
Your Ans. c Correct Ans. c

Solution. Ans-143: (c) Visual field testing


Ref: Read the text below
Sol :
· Visual field does not help in localizing foreign body, rather detects neuronal loss leading to field defects.
Xray orbit, PNS and lateral views are initial imaging procedures used for localizing intraocular metallicforeign bodies.
Q.144 The wave length of Nd: YAG laser in Nanometer is
a. 1064 Nanometer
b. 532 Nano meter
c. 514 Nanometer
d. 800 Nano meter
Your Ans. a Correct Ans. a

Solution. Ans-144: (a) 1064 Nanometer


Ref:Parson’s Diseases of the Eye, 20th edn. Pg. 568
Sol :
§ Wavelength of Nd: YAG laser = 1064 nm.
Q.145 Angular conjunctivitis caused by the following except
a. Streptococcous hemolyticus
b. Staphylococcus aureus
c. Neisseria gonorrhea
d. Moraxella lacunata
Your Ans. b Correct Ans. c
Solution. Ans-145: (c) Neisseria gonorrhea
Ref:Parson’s Diseases of the Eye. 20th edn. Pg. 162
Sol :
§ Angular conjunctivitis (diplobacillaryconjunctivitis) is typically caused by Moraxella lacunata.
§ Also caused by staphylococci.
§ Responds to tetracycline eye ointment and eyedrops containing zinc.
Q.146 Which of the following subtypes of cataract is the most common cause of visual morbidity :
a. Senile
b. Congenital
c. Traumatic
d. Drug toxicity
Your Ans. d Correct Ans. a

Solution. Ans-146: (a) Senile


Ref: Parsons’ -248
Sol :
§ Age-related/senile cataract is responsiblefor 48% of world blindness, according to the World Health Organization
(WHO).
Q.147 In Lowe syndrome, all of the following are seen except :
a. Undescended testes in males
b. Cataract
c. Hypertonia
d. Low IQ
Your Ans. c Correct Ans. c

Solution. Ans-147: (c) Hypertonia


Ref: Internet resources
Sol :
§ There are three main symptoms which are present in all individuals with Lowe syndrome :
ØCataracts in one or both eyes at birth
ØLow muscle toneand weakness (hypotonial)
ØKidney dysfunction– the kidneys may functions normally at birth, but abnormal is present often by 1 year old.
ØMental retardation
ØSeizures – usually in young children with the syndrome
ØBehavior problems
ØGlaucoma – often in both eyes
ØBone weakness – Many individuals with Lowe syndrome have bone fractures, especially in the leg bones when learning to walk.
About one–third of individuals have more than one fracture.
Undescended testicles in boys.
Q.148 Initial treatment for perforated corneal ulcer is :
a. Conjuctival flap grafting
b. Tissue adhesive glue
c. Contact lens
d. Local anesthetics drops instillation
Your Ans. b Correct Ans. b

Solution. Ans-148: (b) Tissue adhesive glue


Ref: Parsons’ -191
Sol :
§ Drugs that are usually contraindicated in corneal ulcer are topical corticosteroids and anesthetics – these should not be
used on any type of corneal ulcer because they prevent healing, may lead to superinfection with fungi and other
bacteria.
§ Further option is the use of a tissue adhesive such as N-butyl 2-ethyl cyanoacrylate monomer which is applied to the area
of perforation after careful debridement.
Drying of the adhesive may take few minutes after which anterior chamber may reform.
Q.149 Ashok, 34 year male is diagnosed with granulomatous uveitis of right eye. All of the following features are
consistent with diagnosis except -
a. Insidious onset
b. Mutton fat keratic precipitates
c. Marked inflammation
d. Iris nodules
Your Ans. c Correct Ans. c
Solution. Ans-149: (c) Marked inflammation
Ref: Read the text below
Sol :
Diagnosis of granulomatous Vs. Nongranulomatous is basically pathological. But few clinical features help in
characterising these conditions - following features help to differentiate -

Conditions which cause granulomatous uveitis one -


· Tuberculosis, leprosy, syphilis, brucellosis
· Sarcoidosis
· HSV, HZV, Varicella
Phacoanaphylactic uveitis
Q.150 Treatment for absolute glaucoma is:
a. Acetazolamide
b. Trabeculectomy
c. Timolol
d. Cyclocryotherapy
Your Ans. d Correct Ans. d

Solution. Ans-150: (d) Cyclocryotherapy


Ref: Indian Journal of Ophthalmology.
Sol :
Absolute glaucoma
Absolute glaucoma is the end stage of all types of glaucoma. The eye has no vision, absence of pupillary light reflexand
pupillary response, and has a stony appearance. Severe pain is present in the eye. The treatment of absolute glaucoma is a
destructive procedure like cyclocryoapplication, cyclophotocoagulation, or injection of 100% alcohol.
Q.151 Most common cause for community acquired pneumonia :
a. Staphylococcus
b. H influenza
c. Pseudomonas
d. Streptococcus
Your Ans. d Correct Ans. d

Solution. Ans-151: (d) Streptococcus


Ref:Read the text below
Sol :
§ Microbila Causes of Community – Acquired Pneumonia by Site of Care

Hospitalized Patients
Non – ICU
S. pneumonia ICU
Outpatients
S. pneumonia
Streptococcus pneumoniae
M. pneumoniae
Mycoplasma pneumoniae
Staphylococcus aureus
Haemophilus influenzae
Chlamydophila pneumoniae Legionella spp.
C. pneumoniae
H. influenzae Gram – negative bacilli
Respiratory viruses
Legionella spp. H. influenzae
Respiratory viruses”
Q.152 Which of the following drugs is useful in treatment of PSVT?
a. Adenosine
b. Amiodarone
c. Verapamil
d. Digitoxin
Your Ans. a Correct Ans. a

Solution. Ans-152: (a) Adenosine


Ref: Read the text below
Sol :
· Adenosine, an ultra short acting AV nodal blocking agent, is indicated if vagal maneuvers are not effective.
· If successful, followup therapy with diltiazem, verapamilor metoprololmay be indicated. Adenosinemay be safely used
during pregnancy.
· SVT that does not involve the AV node may respond to other anti-arrhythmic drugs such as sotalolor
amiodarone.
Q.153 Which of the following test of peritoneal fluid has the greatest sensitivity for detecting spontaneous bacterial
peritonitis?
a. White blood cell count
b. Gram’s stain
c. pH
d. Lactate
Your Ans. a Correct Ans. a

Solution. Ans-153: (a) White blood cell count


Ref.:Read the text below
Sol :
§ There is not true gold standard in the diagnosis of spontaneous bacterial peritonitis.
§ If the white blood cell count is more than 250 calls/mm3, there is a greater than 95% chance that bacterial peritonitis is
present.
§ The sensitivities of ascetic fluid lactate and pH are less than 50%, whereas that of the Gram stain is less than 10%.
Q.154 Which of the following represents a contraindication to abdominal paracentesis?
a. Obesity
b. Extensive prior abdominal surgery
c. Abdominal aortic aneurysm
d. Disseminated intravascular coagulation
Your Ans. d Correct Ans. d

Solution. Ans-154: (d) Italy


Ref.:Read the text below
Sol :
§ There are few contraindications to abdominal paracentesis. The most important is the presence of severe
uncountrollable coagulopathy.
§ If there are concerns about localizing site for aspiration of peritoneal fluid in obese patients or those with a history of extensive
abdominal surgery, ascetic fluid collections can be localized by ultrasonography.
Q.155 Which of the following least likely to present with a red blood cell profile of microcytosis?
a. Iron-deficiency anemia
b. Thalassemia major
c. Alcoholism
d. Lead poisoning
Your Ans. c Correct Ans. c

Solution. Ans-155: (c) Alcoholism


Ref.:Read the text below
Sol :
§ An early sign of alcoholism is macrocytosis.
§ Thalassemias, iron-deficiency anemia, and lead poisoning all result in microcytosis.
Q.156 Drug of choice for thromboembolism is :
a. Streptokinase
b. Low molecular weight heparin
c. Urokinase
d. Warfarin
Your Ans. b Correct Ans. b
Solution. Ans-156: (b) Low molecular weight heparin
Ref: Read the text below
Sol :
Management
Anticoagulant medications include the following:
· Heparin or a low-molecular-weight heparin (LMWH)
· Subsequent administration of an oral coumarin derivative (typically, warfarin sodium)
· Oral factor Xa inhibitors (eg, rivaroxaban)
Thrombolytic options (for initial treatment of patients with acute, massive PE causing hemodynamic instability)
include the following:
· Tissue plasminogen activator (t-PA; first-choice thrombolytic agent), including the recombinant agents alteplase, reteplase,
and tenecteplase
· Streptokinase (risk of antibody development)
· Urokinase (of limited availability)
Surgical interventions include the following:
· Thrombectomy
· Embolectomy (limited to massive PE when thrombolysis is contraindicated or other treatments have failed)
· Venous interruption (currently rare)
Q.157 Neurological and cardiac manifestation of syphilis are seen in :
a. Primary syphilis
b. Secondary syphilis
c. Tertiary syphilis
d. Quaternary syphilis
Your Ans. c Correct Ans. c

Solution. Ans-157: (c) Tertiary syphilis


Ref: Read the text below
Sol :
o Classification and clinical features of syphilis
Hard chancere
Primary :
Paintless, regional lymphadenopathy
General : Fever, malaise, arthralgia, sore throat and generalized lymphadenopathy.
Secondary : Skin : Red/brown maculopapular non-itchy, sometimes scaly rash ; condylomata lata.
Mucous membranes : Mucous patches, ‘snail-track’ ulcers in oropharynx and on genitalia.
Late benign : Gummas (bone and viscera).
Tertiary : Cardiovascular : Aortitis and aortic regurgitation.
Neurosyphilis : Meningovascular involvement, general paralysis of the insane (GPI) and tabes dorsalis.

Q.158 A 45 yr old female for pre-surgery hemostatic evaluation has PT 13.5 sec(control 11-15s) and APTT 48s(control 26-
39s).she does not give history of excessive bleeding following laparoscopic cholecystectomy done 2 yrs ago.
Which of the following should be done?
a. Factor VIII assay
b. Platelet aggregation test
c. Dilute russel viper venom time
d. Ristocetin co-factor assay
Your Ans. c Correct Ans. c

Solution. Ans-158: (c) Dilute russel viper venom time


Ref: Harrison principle of internal medicine 17th ed pg 367
Sol:
In this question APTT is prolonged and PT is normal
· In isolated factor VIII deficiency both PT and APTT is prolonged so factor VIII assay will not help
· Platelet aggregation test is donefor Glanzmann thrombasthenia which is a platelet aggregation defect where BT is
prolonged, so it will not be helpful(Robbins 670)
· Ristocetin co-factor assay is done for Bernard soulier disease which is a platelet adhesion defect where BT is
prolonged, so it will not be helpful(Robbins 670)
Coming to Dilute russel viper venom time, it is done for antiphopholipid antibody syndrome
· Antibodies to phospholipids (cardiolipin) or phospholipid-binding proteins are detected by ELISA.
· When these antibodies interfere with phospholipid-dependent coagulation tests, they are termed lupus anticoagulants.
· The aPTT has variable sensitivity to lupus anticoagulants, depending in part on the aPTT reagents used.
· An assay utilizing a sensitive reagent has been termed an LA-PTT.
· The dilute Russell Viper Venom test (dRVVT) and the tissue thromboplastin time (TTI) are modifications of standard tests
with the phospholipid reagent decreased, thus increasing the sensitivity to antibodies that interfere with the phospholipid
component.
Q.159 60 yr old man with complains of progressive dementia for the past 6 months. Suddenly he complained jitteriness
of whole body. EEG showed sharp periodic bipolar spikes. Diagnosis is
a. Alzheimer's disease
b. Creutzfeld Jackob disease
c. Lewy body dementia
d. Herpes encephalitis
Your Ans. c Correct Ans. b

Solution. Ans-159: (b) Creutzfeld Jackob disease


Ref: Harrison internal medicine - 2646
Sol:
· The EEG is often useful in the diagnosis of CJD, although only about 60% of individuals show the typical pattern.
· During the early phase of CJD, the EEG is usually normal or shows only scattered theta activity. In most advanced cases,
repetitive, high-voltage, triphasic, and polyphasic sharp discharges are seen, but in many cases their presence is
transient.
· The presence of these stereotyped periodic bursts of <200 ms duration, occurring every 1–2 s, makes the diagnosis
of CJD very likely.
· These discharges are frequently but not always symmetric; there may be a one-sided predominance in amplitude. As CJD
progresses, normal background rhythms become fragmentary and slower.
Q.160 Which is Vitamin K dependent clotting factor
a. Factor VII
b. Factor I
c. Factor XI
d. Factor XII
Your Ans. a Correct Ans. a

Solution. Ans-160: (a) Factor VII


Ref: Harper's Illustrated Biochemistry, 28th ed pg 587
Sol:
· Vitamin K–dependent carboxylation of Glu to Glaresidues occurs in the amino terminal regions of factors II, VII, IX, and
X and also proteins C and S
Q.161 9 year old boy with elevated both PT and APTT. What is the diagnosis
a. Defect in extrinsic pathway
b. Defect in Intrinsic pathway
c. Platelet function defect
d. Defect in common pathway
Your Ans. d Correct Ans. d
Solution. Ans-161: (d) Defect in common pathway
Ref: Harrison principle of internal medicine 17th ed pg 367
Sol:
Hemostatic Disorders and Coagulation Test Abnormalities

Prolonged activated partial thromboplastin time (aPTT)


No clinical bleeding – decreased factors XII, high-molecular-weight kininogen, protein kinase
Variable, but usually mild, bleeding – decreases factor XI, mild decrease FVIII and FIX
Frequent, severe bleeding – severe deficiencies of FVIII and FIX
Heparin
Prolonged prothrombin time (PT)
Factor VII deficiency(extrinsic factors)
Vitamin K deficiency – early
Warfarin anticoagulation
Prolonged aPTT and PT
Factor II, V or X deficiency(common pathway factors)
Vitamin K deficiency – late
Direct thrombin inhibitors
Prolonged thrombin time
Heparin or heparin-like inhibitors
Mild or no bleeding – dysfibrinogenemia
Frequent, severe bleeding – afibrinogenemia
Prolonged PT and/or aPTT not correct with mixing with normal plasma
Bleeding – specific factor inhibitor
No symptoms, or clotting and/or pregnancy loss – lupus anticoagulant
Disseminated intravascular coagulation
Heparin or direct thrombin inhibitor
Abnormal clot solubility
Factor XIII deficiency
Inhibitors or defective cross-linking
Rapid clot lysis
Deficiency of alpha2-antiplasmin or plasminogen activator inhibitor 1
Treatment with fibrinolytic therapy

Q.162 Which of the vitamin deficiency lead to lactic acidosis


a. Riboflavin
b. Thiamine
c. Niacin
d. Panthotheic acid
Your Ans. b Correct Ans. b

Solution. Ans-162: (b) Thiamine


Ref: Harrison - 290
Sol:
· An increase in plasma L-lactate may be
· Type A secondaryto poor tissue perfusion circulatory insufficiency (shock, cardiac failure), severe anemia, mitochondrial
enzyme defects, and inhibitors (carbon monoxide, cyanide)—or
· Type B— due to aerobic disorders malignancies, nucleoside analogue reverse transcriptase inhibitors in HIV, diabetes
mellitus, renal or hepatic failure, thiamine deficiency, severe infections (cholera, malaria), seizures, or drugs/toxins (biguanides,
ethanol, methanol, propylene glycol, isoniazid, and fructose).
· Propylene glycolmay be used as a vehicle for IV medications including lorazepam, and toxicity has been reported in
several settings.
· Unrecognized bowel ischemiaor infarction in a patient with severe atherosclerosis or cardiac decompensation receiving
vasopressors is a common cause of lactic acidosis.
· Pyroglutamic acidemiahas been reported in critically ill patients receiving acetaminophen, which is associated with
depletion of glutathione. D-Lactic acid acidosis, which may be associated with jejunoileal bypass, short bowel syndrome, or
intestinal obstruction, is due to formation of D-lactate by gut bacteria.
Q.163 Which one of the pair not correct
a. Supportive Care - chronic viral hepatitis B
b. Antiviral Drugs - acute viral hepatitis B
c. Supportive Care -acute viral hepatitis B
d. Antiviral Drugs – chronic viral hepatitis B
Your Ans. b Correct Ans. b

Solution. Ans-163: (b) Antiviral Drugs - acute viral hepatitis B


Ref: Harrison 17th ed pg 1946
Sol:
· In hepatitis B, among previously healthy adults who present with clinically apparent acute hepatitis, recovery occurs in
~99%; therefore, antiviral therapy is not likely to improve the rate of recovery and is not required.
· In rare instances of severe acute hepatitis B, treatment with a nucleoside analogue, such as lamivudine, at the 100-
mg/d oral dose used to treat chronic hepatitis B , has been attempted successfully
Q.164 “Morning dip” in peak expiratory flows is a feature of :
a. Nocturnal asthma
b. Emphysema
c. Cardiac asthma
d. Bronchiectasis
Your Ans. a Correct Ans. a

Solution. Ans-164: (a) Nocturnal asthma


Ref: Read the text below
Sol :
§ Validation of morning dip of peak expiratory flow as an indicator of the severity of nocturnal asthma.
§ Overnight falls in peak expiratory flow (PEF) (with the morning dip of the index) may be considered the hallmark of
nocturnal asthma.
Q.165 Pulmonary embolism may present as :
a. Pulmonary embolism may present as :
b. Acute breathlessness
c. Chest pain
d. All the above
Your Ans. d Correct Ans. d

Solution. Ans-165: (d) All the above


Ref: Read the text below
Sol :
Acute Pulmonary Embolism
§ History : A characteristic clinical setting, such as prolonged immobilization, recent surgery, congestive heart failure, or
recent trauma will be present.
§ Oral contraceptives, sickle– cell anaemia and polycythaemia are associated with increased incidence of pulmonary
embolism.
§ The usual history is acute onset breathlessness and chest pain, occasionally associated with blood – tinged sputum.
Q.166 Treatment for childhood hypothyroidism is with :
a. T4
b. T3
c. Levothyroxine
d. TSH
Your Ans. c Correct Ans. c

Solution. Ans-166: (c) Levothyroxine


Ref: Read the text below
Sol :
Management of childhood hypothyroidism
· The aim of treatment is early detection and early thyroid hormone replacement to ensure that infants do not develop
irreversible neurological disability.
· Thyroxine hormone replacement with L-thyroxine is given once daily and titrated to TFTs.
· There is no evidence at present to suggest that higher starting doses of thyroxine have more beneficial effect on
outcome compared with standard doses.
· TFTs need to be monitored on a regular basis. The frequency of blood tests can be reduced after the first 2 years of life
once adequate replacement is achieved.
· T4 should ideally be kept in the upper half of the normal range.
Transient hypothyroidism need not be treated unless the low T4 and raised TSH persist beyond 2 weeks. Treatment is
usually terminated after 3 to 5 months
Q.167 Relative Bradycardia is seen in all except :
a. Malaria
b. Brucellosis
c. Malignant lymphoma
d. Bacterial pneumonias
Your Ans. a Correct Ans. b

Solution. Ans-167: (b) Brucellosis


Ref: Read the text below
Sol :
When a person has a fever, the pulse usually increases along with the temperature. Relative bradycardia refers to a
pulse temperature deficit, with the pulse less than that expected for a given temperature.
Criteria for using relative bradycardia in clinical diagnosis :
o Age of patient > = 13 years.
o Temperature > = 102° F and < = 106° F.
o The pulse is taken simultaneously with the temperature.
o The patient has normal sinus rhythm with no arrhythmias, second or third degree heart block, or pacemaker.
Infectious causes of relative bradycardia:
o Legionnaires’ disease
o Psittacosis
o Q fever
o Typhoid fever
o Typhus
o Malaria
o Babesiosis
o Leptospirosis
o Yellow fever
o Dengue fever
o Rocky Mountain spotted fever.
Noninfectious causes of relative bradycardia :
o Beta – blockers
o CNS lesions
o Malignant lymphoma
o Factitious fever
o Drug – related fever.
Q.168 Ideal investigation for adrenal insufficiency is :
a. ACTH stimulation
b. Cortisol level estimation
c. Dexamethasone test
d. Urinary steroid estimation
Your Ans. a Correct Ans. a

Solution. Ans-168: (a) ACTH stimulation


Ref: Read the text below
Sol :
§ ACTH Stimulation test : This is the most specific test for diagnosing adrenal insufficiency. Blood cortisol levels are
measured before and after a synthetic form of adrenocorticotrophic hormone (ACTH), a hormone secreted from the
anterior pituitary, is given by injection.
Q.169 Pulsus paradoxus is found in all of following except:
a. A severe asthmatic attack
b. Severe left ventricular failure
c. Myocardial disease
d. Constrictive pericarditis
Your Ans. d Correct Ans. b

Solution. Ans-169: (b) Severe left ventricular failure


Ref: Read the text below
Sol :
• Pulsus paradoxus is the exaggerated fall of pressure associated with inspiration due to disease process like asthma,
constrictive pericarditis, tamponade and left ventricular disease (cardiomyopathy).
• It is not a feature of left ventricular failure.
Q.170 About treatment of phenylketonuria first step to be done is
a. Stopping the substrate of defective enzyme
b. Supplement of defective enzyme
c. Reducing the substrate for the enzyme
d. Provide the deficient proteins
Your Ans. a Correct Ans. c

Solution. Ans-170: (c) Reducing the substrate for the enzyme


Ref: Harrison - 2470
Sol:
· To prevent mental retardation, diagnosis and initiation of dietary treatment of classic phenylketonuria must occur
before the child is 3 weeks of age.
· Dietary phenylalanine restrictionis usually instituted if blood phenylalanine levels are >250 mmol/L (4 mg/dL).
· Treatment consists of a specialdiet low in phenylalanine and supplemented with tyrosine, since tyrosine becomes an
essential amino acid in phenylalanine hydroxylase deficiency. With therapy, plasma phenylalanine concentrations should
be maintained between 120 and 360 mmol/L (2 and 6 mg/dL). Dietary restriction should be continued and
monitored indefinitely.
Q.171 A patient presented with thunder clap headache. Followed by unconsciousness with progressive III cranial nerve
plasy. Etiology
a. Extradural hemorrhage
b. Aneurysmal subarachnoid hemorrhage
c. Basilar migraine
d. Cluster headache
Your Ans. b Correct Ans. b

Solution. Ans-171: (b) Aneurysmal subarachnoid hemorrhage


Ref: Harrison 17th ed pg 1727
Sol:
· Most unruptured intracranial aneurysms are completely asymptomatic. Symptoms are usually due to rupture and
resultant SAH, although some present with mass effect on cranial nerves or brain parenchyma.
· At the moment of aneurysmal rupture with major SAH, the ICP suddenly rises.
· This may account for the sudden transient loss of consciousness that occurs in nearly half of patients.
· Sudden loss of consciousness may be preceded by a brief moment of excruciating headache, but most patients first
complain of headache upon regaining consciousness.
· In 10% of cases, aneurysmal bleeding is severe enough to cause loss of consciousness for several days.
· The patient often calls the headache "the worst headache of my life"; however, the most important characteristic
is sudden onset.
· Occasionally these ruptures may present as headache of only moderate intensity or as a change in the patient's usual
headache pattern. The headache is usually generalized, often with neck stiffness, and vomiting is common.
· Occasionally, prodromal symptoms suggest the location of a progressively enlarging unruptured aneurysm.
· A third cranial nerve palsy, particularly when associated with pupillary dilatation, loss of ipsilateral (but retained
contralateral) light reflex, and focal pain above or behind the eye, may occur with an expanding aneurysm at the junction of the
posterior communicating artery and the internal carotid artery.
· A sixth nerve palsy may indicate an aneurysm in the cavernous sinus, and visual field defects can occur with an expanding
supraclinoid carotid or anterior cerebral artery aneurysm.
· Thunderclap headache is a variant of migraine that simulates a SAH.
· Before concluding that a patient with sudden, severe headache has thunderclap migraine, a definitive workup for aneurysm
or other intracranial pathology is required.
Q.172 12 year old child having microcytic hypochromic anemia his blood picture showing Hb >40 gm/l, serum iron levels
20,transferring saturation 40%,ferritin >800 the diagnosis is
a. Iron deficiency anemia
b. Hemochromatosis
c. Atransferrinemia
d. DMT1 gene mutation
Your Ans. d Correct Ans. c

Solution. Ans-172: (c) Atransferrinemia


Ref: Read the text below
Sol:
· Atransferrinemia is characterisedby microcytic hypochromic picture, low serum iron and low TIBC.
· Tranferrin is low leading to high ferritin.
· Hemochromatosis typically doesnt have MCHC or low iron.
Q.173 A 16-year old girl presented with abdominal pain after administration of a sulfa drug. She was incoherent and
suffered a seizure. Which of the following is the most likely possibility?
a. Acute intermittent porphyria
b. Congenital erythropoetic porphyria
c. Adenosine deaminase deficiency
d. HGPRT deficiency
Your Ans. a Correct Ans. a

Solution. Ans-173: (a) Acute intermittent porphyria


Ref:Current Medical Diagnosis and Treatment 2013; Chapter 40, pg 1664-1665
Sol :
Essentials for Diagnosis of AIP:
· Unexplained abdominal crisis, generally in young women.
· Acute peripheral or central nervous system dysfunction.
· Recurrent psychiatric illnesses.
· Hyponatremia.
· Porphobilinogen in the urine during an attack.
· Patients show intermittent abdominal pain of varying severity, and in some instances it may so simulate acute
abdomen as to lead to exploratory laparotomy.
· Because the origin of the abdominal pain is neurologic, there is absence of fever and leukocytosis.
· Complete recovery between attacks is usual.
· Any part of the nervous system may be involved, with evidence for autonomic and peripheral neuropathy.
· Peripheral neuropathy may be symmetric or asymmetric and mild or profound.
· In the latter instance, it can even lead to quadriplegia with respiratory paralysis.
· Other central nervous system manifestations include seizures, psychosis, and abnormalities of the basal ganglia.
· Hyponatremia may further cause or exacerbate central nervous system manifestations.
· The diagnosis can be confirmed by demonstrating an increased amount of
· porphobilinogen in the urine during an acute attack.
· Freshly voided urine is of normal color but may turn dark upon standing in light and air.
· Avoidance of factors known to precipitate attacks of AIP— especially drugs—can reduce morbidity.
· Sulfonamides and barbiturates are the most common culprits.
Q.174 An adult hypertensive male presented with sudden onset severe headache and vomiting. On examination, there is
marked neck rigidity and no focal neurological deficit was found. The symptoms are most likely due to –
a. Intracerebral parenchymal haemorrhage
b. Subarachnoid haemorrhage
c. Meningitis
d. Ischemic stroke
Your Ans. b Correct Ans. d

Solution. Ans-174: (d) Ischemic stroke


Ref:Harrison’s ‘Principles of Internal Medicine’; 18/e, pg 2262
Sol :
· The patient often calls the headache "the worst headache of my life";
· However, the most important characteristic is sudden onset.
· The headache is usually generalized, often with neck stiffness, and vomiting is common.
· “Sudden headache in the absence of focal neurologic symptoms is the hallmark of aneurysmal rupture”

SUBARACHNOID HEMORRHAGE –
· Patient feels thunder clap like sensation and complains of worst head-ache of his life.
Most common causes Most common locations
· Terminal ICA
· Trauma · MCA Bifurcation
· Berry Aneurysm (Spontaneous subarachnoid hemorrhage) · Top of Basilar artery

Pathophysiology –
· 85% are seen in anterior circulation
· A neck with dome is seen.
· Arterial internal elastic lamina disappears at the base of neck.
· Media becomes thin – connective tissue replaced by smooth muscle.

Risk of Rupture -
· 7 mm
· Top of Basilar artery
· Origin of PCA

Clinical features –
· Due to compression of adjacent structures by aneurysm
· Sentinel bleed – small Rupture causes leak of blood in Subarachnoid space.

Prognosis assessed by –
· Hunt & Hess scale
· WFNS (world Fed. of Neurological scale).

Delayed Neurological deficit –


· Rerupture
· Hydrocephalus
· Vasospasm – M.C. Cause of Delayed morbidity and death with in 4 to 14 days after SAH
· Hyponatremia

Lab Investigation –
· Blood in CSF is the hallmark of SAH.
· ECG – ST Segment and T wave changes Similar to Cardiac ischemia.

Treatment –
1. Surgical – Clipping of neck
2. Medical –
· Maintain airway, BP,
· Manage Vasospasm, Hydrocephalus, Hyponatremia,
· To Decrease ICP – Nicardipine, Labetolol, Esmolol
· Stool softener to prevent straining.
· Seizures are Uncommon but prophylactic phenytoin is given.
· Glucocorticoids are given to reduce head and neck pain.
· To treat vasospasm – Calcium channel blocker eg. Nimodipine 60 mg. PO every 4 hrly.
· For Chronic hydrocephalus –shunting is done.
· To treat hyponatremia – no free water restriction is done as hyponatremia and hypovolemia is due to BNP and ANP. so oral
salt coupled with NS or hypertonic saline.
· Hyponatremia should not be corrected very fast as it can cause central pontine myelinolysis syndrome
Q.175 Which of the following features helps in distinguishing seizures from syncope?
a. Loss of consciousness
b. Injury due to fall
c. Urinary incontinence
d. Physical weakness with clear sensorium
Your Ans. c Correct Ans. d

Solution. Ans-175: (d) Physical weakness with clear sensorium


Ref:Harrison’s ‘Principles of Internal Medicine’; 18/e, pg 3261; Braunwald’s Heart Disease; 9/e
Sol :
· Syncope is a transient loss of consciousness due to transient global cerebral hypoperfusioncharacterized by
rapid onset, short duration, and spontaneous recovery.
· Loss of consciousnessresults from a reduction of blood flow to the reticular activating system located in the brainstem and
does not require electrical or chemical therapy for reversal.
· The metabolism of the brain, in contrast to that of many other organs, is exquisitely dependent on perfusion.
· Consequently, cessation of cerebral blood flow leads to loss of consciousness within approximately 10 seconds.
· Restoration of appropriate behavior and orientation after a syncopal episode is usually immediate.
· Retrograde amnesia, although uncommon, can be present in the elderly.
· Syncope, as defined here, represents a subset of a much wider spectrum of conditions that can result in transient loss of
consciousness, including conditions such as stroke and epileptic seizures.
· Nonsyncopal causes of transient loss of consciousness differ in their mechanism and duration.
· Characteristics of a seizure include the presence of an aura, cyanosis, unconsciousness, motor manifestations lasting >15
seconds, postictal disorientation, muscle soreness, and sleepiness.
· In contrast, a syncopal episode is more likely if the event was provoked by acute pain or anxiety or occurred immediately
after arising from the lying or sitting position.
· Patients with syncope often describe a stereotyped transition from consciousness to unconsciousness that includes
tiredness, sweating, nausea, and tunneling of vision, and they experience a relatively brief loss of consciousness.
· Headache or incontinence usually suggests a seizure but may on occasion also occur with syncope.
· A brief period (i.e., 1–10 seconds) of convulsive motor activity is frequently seen immediately at the onset of a syncopal
episode, especially if the patient remains in an upright posture after fainting (e.g., in a dentist's chair) and therefore has a
sustained decrease in cerebral perfusion.
· Rarely, a syncopal episode can induce a full tonic-clonic seizure. In such cases the evaluation must focus on both the cause
of the syncopal event as well as the possibility that the patient has a propensity for recurrent seizures.
Q.176 Damage control surgery is –
a. Done to control damage done during surgery
b. Done during the triage procedure
c. Minimal intervention to stabilize the patient and do a definitive surgery later
d. Maximum possible surgical intervention done considering that it is the last chance for intervention
Your Ans. c Correct Ans. c

Solution. Ans-176: (c) Minimal intervention to stabilize the patient and do a definitive surgery later
Ref:http://en.wikipedia.org/wiki/Damage_control_surgery
Sol :
· Damage control surgery (DCS) is a form of surgery utilized in severe unstable injuries typically by trauma surgeons.
· This form of surgery puts more emphasis on preventing the trauma triad of death, rather than correcting the anatomy.
· The first recorded instance of damage control surgery was in 1983 by Stone.
· In 1993, Rotondo was the first to show definitive proof that damage control surgery yielded better outcomes than
alternatives, and coined the term
· This procedure is generally indicated when a person sustains a severe injury that impairs their ability to maintain
homeostasis due to severe hemorrhage leading to metabolic acidosis, hypothermia, and increased coagulopathy.
· A major component of the surgery is early recognition of a person who could benefit from it, which often means bypassing
the emergency department except for attempts of immediate stabilization techniques, such as gaining airway
access.
· Typically the operating room is heated higher than normal to help deal with the associated hypothermia.
· The procedure comprises three different steps that are needed for full effect.
· In the first procedure a laparotomy is performed to control hemorrhage.
· Generally this procedure will last no longer than one hour.
· After immediate life threats have been surgically managed, the area is then covered temporarily and the person sent to an
intensive care unit for the second phase.
· In the second phase the patient is given a combination of various medications and treatments to help restore
a physiologic balance, especially with regards to their temperature, oxygenation, and pH level.
· An important element of treatment at this stage is passive rewarming, as generally it will reverse most of the ill effects of
the trauma triad.
· This phase generally lasts no longer than two days but is dependent on the person’s condition.
· When the person does not have their condition improved within the first 24 hours it could mean there was missed
hemorrhage which could require immediate surgery, regardless of the reversal of the trauma triad.
· In the third phase, the person is operated on again and more definitive procedures are performed.
Q.177 A elderly male presents with 4x5 cm lump in the right neck. FNAC assessment revealed it to be a squamous cell
carcinoma. Clinical assessment of the oral cavity, pharynx, hypopharynx and larynx did not yield any tumour.
Whole body PET scan did not show any increased uptake except for the neck mass. A diagnosis of unknown
primary was made. According to the AJCC system of classification, the TNM status of the tumour would be –
a. T1N2M0
b. T0N2aM0
c. T1N2aM0
d. TxN2aMx
Your Ans. d Correct Ans. d

Solution. Ans-177: (d) TxN2aMx


Ref:Reference: Sabiston’s ‘Textbook of Surgery’, 18/e; Chap 33; DeVita, Hellman and Rosenberg’s ‘Cancer:
Principles and Practice of Oncology’; 8/e, Chap 56.
Sol :
· Squamous carcinoma at a metastatic site represents approximately 5% of all patients with unknown primary.
· Effective treatment is available for patients with certain clinical syndromes (approximately 90% of patients), and appropriate
evaluation is important.

SQUAMOUS CARCINOMA INVOLVING CERVICAL AND SUPRACLAVICULAR LYMPH NODES


· The cervical lymph nodes are the most common metastatic site.
· Patients are usually middle-aged or elderly, and frequently they have abused tobacco and/or alcohol.
· When the upper or middle cervical lymph nodes are involved, a primary tumor in the head and neck region should be
suspected.
· Clinical evaluation should include an examination of the oropharynx, hypopharynx, nasopharynx, larynx, and
upper esophagusby direct endoscopy, with biopsy of any suspicious areas.
· CT of the neck better defines the disease in the neck and occasionally identifies a primary site.
· PET scanning is indicated,as it may also identify primary sites.

CLASSIFICATION DESCRIPTION
NX Regional lymph nodes cannot be assessed
N0 No regional lymph node metastasis
N1 Metastasis in a single ipsilateral lymph node, ≤3 cm in greatest dimension
Metastasis in a single ipsilateral lymph node, >3 cm but not >6 cm in greatest dimension; or in multiple
N2 ipsilateral lymph nodes, none >6 cm in greatest dimension; or in bilateral or contralateral lymph nodes,
none >6 cm in greatest dimension
N2a Metastasis in a single ipsilateral lymph node >3 cm but not >6 cm in greatest dimension
N2b Metastasis in multiple ipsilateral lymph nodes, none >6 cm in greatest dimension
N2c Metastasis in bilateral or contralateral lymph nodes, none >6 cm in greatest dimension
N3 Metastasis in a lymph node >6 cm in greatest dimension

In this case, as the primary is unknown, the T status cannot be assessed; hence denoted as Tx and Mx.
Q.178 An elderly male with a history 60-pack-year of smoking is now diagnosed to be suffering from carcinoma lung.
Histological evaluation of the tumour revealed small, highly mitotic cells with hyperchromatic nuclei. Which of the
following clinical presentation might occur in the patient during the course of his illness?
a. Psychosocial changes
b. Enlarged breasts, increased hair all over the body
c. Frequent need for blood transfusions
d. Thin extremities, central obesity
Your Ans. d Correct Ans. d
Solution. Ans-178: (d) Thin extremities, central obesity
Ref:Sabiston’s Textbook of Surgery, 18/e, Chap 59; Chapter 26, Current Medical Diagnosis and Treatment, 2013,
Chap 26.
Sol :
· Small cell lung cancer represents about 20% of all lung cancers; about 80% are centrally located.
· The disease is characterized by an aggressive tendency to metastasize.
· It often spreads early to mediastinal lymph nodes and distant sites, especially bone marrow and brain.
· Small cell lung cancer appears to arise in cells derived from the embryologic neural crest.
· Microscopically, these cells appear as sheets or clusters of cells with dark nuclei and little cytoplasm.
· This oatlike appearance under the microscope provides the term oat cell carcinoma to this disease.
· Neurosecretory granules are evident on electron microscopy.
· This tumor is staged as limited stage (disease restricted to an ipsilateral hemithorax within a single radiation port) and
extensive stage (obvious metastatic disease).
· These tumors manifest a prominent neuroendocrine phenotype in morphology, immunohistochemistry, and ultrastructural
features.
· The histology is characterized by organoid, ribbon, or festoon pseudorosette, and sometimes spindle patterns of cuboidal
cells with small and hyperchromatic nuclei.
· Small cell carcinoma of lung is also sometimes associated with a paraneoplastic syndrome in which ACTH is
secreted.
· Thus, it is one of the sources of ectopic ACTH secretion.
· This excessive ACTH secretion would lead to excessive cortiocstroid production from the adrenals leading to Cushing’s
syndrome.
· Thus, patient would present with fat redistribution characteristic of Cushing’s syndrome, i.e. central obesity with moon
face and buffalo hump; with thinning of extremities and a orange-on-skin appearance.

EXTRATHORACIC NON-METASTATIC SYMPTOMS (PARANEOPLASTIC SYNDROMES) OF LUNG CANCER


GENERAL
· Weight loss / Cachexia
· Fatigue
· General malaise
ENDOCRINE
· Cushing’s syndrome from adrenocorticotropic hormone secretion
· Inappropriate antidiuretic hormone causing hyponatremia
· Carcinoid syndrome
· Hypercalcemia due to excessive parathyroid hormone-like protein (PTHrP)
· Rarely, hypoglycemia or ectopic gonadotropins
SKELETAL
· Clubbing – 10 to 20%
· Hypertrophic pulmonary osteoarthropathy (painful periosteal proliferation at the ends of the long bones) – 5%
NEUROMUSCULAR (about 15% and most common with small cell carcinoma)
· Polymyositis
· Myasthenia-like syndrome (Lambert-Eaton)
· Peripheral neuropathy
· Subacute cerebellar degeneration
· Encephalopathy
VASCULAR THROMBOPHLEBITIS

Q.179 In a case of road traffic accident (RTA), pCO2 falls because –


a. Only respiratory centre is affected
b. Only respiratory apparatus is affected
c. Both respiratory centre and apparatus are affected
d. None of these are affected
Your Ans. d Correct Ans. d

Solution. Ans-179: (d) None of these are affected


Ref:Read the text below
Sol :
· If respiratory centre in medulla or respiratory apparatus are damaged, the respiration is affected.
· As a result, the patient will go in hypoventilation.
· This will lead to accumulation of CO2 in the body with a consequent rise in pCO2 (pCO2 concentration is inversely
proportional to alveolar ventilation).
· In the given case scenario, patient is showing a fall in pCO2 levels, which is suggestive of hyperventilation.
· Hence, neither respiratory centre nor apparatus can be affected in this case.
Q.180 A 49-year old male with a 35-pack-year history of smoking presents with a painless left scrotal sac mass.
Examination revealed microscopic hematuria, absence of AFP and LDH. The most probable cause of the scrotal
mass is –
a. Carcinoma lung
b. Seminoma
c. Epididymitis
d. Renal cell carcinoma
Your Ans. d Correct Ans. d

Solution. Ans-180: (d) Renal cell carcinoma


Ref:Read the text below
Sol :
DeVita, Hellman and Rosenberg’s ‘Cancer: Principles and Practice of Oncology’; 8/e, Chap 40
· In studies of risk of renal adenocarcinoma cigarette smoking has been found to be a risk factor.
· A statistically significant dose response has been observed in both sexes for pack-years of cigarette use.
· It has been estimated that 30% of renal carcinomas in men and 24% in women may be directly related to
smoking.
· Renal carcinoma may remain clinically occult for most of its course.
· The classic presentation of pain, hematuria, and flank mass occurs in a minority of patients and often is indicative of
advanced disease.
· A tumor in the kidney can progress unnoticed to a large size in the retroperitoneum until a metastasis appears.

AFP:
· The normal adult serum concentration is usually less than 15 ng/mL.
· Approximately 10% to 20% of clinical stage I, 20% to 40% of low-volume clinical stage II, and 40% to 60% of advanced
NSGCT will have increased AFP levels.
· Increased AFP levels are never seen in pure seminoma.

LDH:
· Increases in the serum concentration of LDH are a reflection of tumor burden, growth rate, and cellular proliferation.
· LDH comprises multiple isoenzymes, but in practice, the combined LDH value for all isoenzymes is used for clinical decision
making.
· Comparison of one laboratory to another is possible by using ratios of the detected level to the upper limit of normal for the
individual assay.
· Increased serum LDH concentrationsare observed in approximately 60% of NSGCT patients with advanced disease and
up to 80% of patients with advanced seminoma.

CLINICAL SYMPTOMS SEEN IN RENAL CELL CARCINOMA


Classic triad (gross hematuria, flank mass, pain) 9%
Hematuria 59%
Pain 41%
Abdominal mass 45%
Fever 7%
Weight loss 28%
Anemia 21%
Erythrocytosis 3%
Hypercalcemia 3%
Acute varicocele 2%
Tumor calcification on x-ray film 13%
Symptoms of metastases 10%
Cancer, incidental finding 7%

Q.181 The most serious complication of a pelvic fracture is –


a. Mal-union
b. Neurogenic shock
c. Hypovolemic shock
d. Rupture of urinary bladder
Your Ans. c Correct Ans. c
Solution. Ans-181: (c) Hypovolemic shock
Ref:Schwartz’ ‘Principles of Surgery’; 8/e, Chap 42.
Sol :
· Injuries to the chest, head, abdomen, and major vessels may require emergent treatment and take precedence over
most musculoskeletal injuries.
· Patients suffering multiple injuries often have suffered severe trauma and can be hemodynamically unstable from
skeletal injuries alone.
· Orthopaedic and trauma surgeons should be particularly alert to this when major pelvic injuries are present.
· In this scenario, the orthopaedist must work in tandem with the trauma surgeon to stabilize and treat the patient.
· The initial role of the orthopaedic surgeon is to help determine whether the patient has a mechanically unstable pelvic
injury.
· This type of injury can cause or contribute to hemodynamic instability.
· Patients in shock must be treated aggressively with volume replacement.
· Unstable pelvic fractures often require some type of stabilization to control hemorrhage.
· If hemodynamic instability occurs and other causes of blood loss have been ruled out, stabilization of the pelvis combined
with possible arteriography and embolization can be life saving.
· Persistent shock, despite fluid resuscitation and stabilization of the pelvic fracture, is an indicator of possible arterial
bleeding, and may require angiography and embolization.
Q.182 A 5-year old child spills boiling water accidentally over her face and trunk. Which of the following methods is the
most correct to estimate the body surface area involved in burns?
a. Rule of 9
b. Rule of palm
c. Lund and Browder chart
d. A growth chart
Your Ans. c Correct Ans. c

Solution. Ans-182: (c) Lund and Browder chart


Ref:http://en.wikipedia.org/wiki/Total_body_surface_area
Sol :
· Total body surface area (TBSA) is an assessment measure of burns of the skin.
· In adults, the "rule of nines" is usedto determine the total percentage of area burned for each major section of the
body.
· In some cases, the burns may cover more than one body part, or may not fully cover such a part; in these cases, burns
are measured by using the casualty's palm as a reference point for 1% of the body.
· For children and infants, the Lund-Browder chart is usedto assess the burned body surface area.
· Different percentages are used because the ratio of the combined surface area of the head and neck to the surface area of
the limbs is typically larger in children than that of an adult.

Lund-Browder chart (Due to rounding, values may not add to 100%)


Anatomic structure Adult (%) Child (%)
Anterior head 4.5 9
Posterior head 4.5 9
Anterior torso 18 18
Posterior torso 18 18
Anterior leg, each 9 6.75
Posterior leg, each 9 6.75
Anterior arm, each 4.75 4.75
Posterior arm, each 4.75 4.75
Genitalia / Perineum 1 1

Lund-Browder chart for special scenario:


Adult, obese (> 80 kg)
Anatomic structure Infant (< 10 kg) (%)
(%)
Head and neck 2 20
Anterior torso 25 16
Posterior torso 25 16
Leg, each 20 16
Arm, each 5 8
Genitalia / Perineum 0 1
Q.183 Which of the following regarding epigastric hernia is true?
a. Always above the umbilicus and in midline
b. May be above or below the umbilicus but always in midline
c. Always above the umbilicus on either side of the midline
d. Anywhere in the abdomen
Your Ans. a Correct Ans. a

Solution. Ans-183: (a) Always above the umbilicus and in midline


Ref: Sabiston’s ‘Textbook of Surgery’; 18/e,
Sol:
EPIGASTRIC HERNIA –
· About 3% to 5% of the population has epigastric hernias.
· Epigastric hernias are two to three times more common in men.
· These hernias are located between the xiphoid process and umbilicus and are usually within 5 to 6 cm of the
umbilicus.
· Like umbilical hernias, epigastric hernias are more common in individuals with a single aponeurotic decussation.
· The defects are small and often produce pain out of proportion to their size owing to incarceration of
preperitoneal fat.
· They are multiple in up to 20% of patients, and about 80% are just off the midline.
· Repair usually consists of excision of the incarcerated preperitoneal tissue and simple closure of the fascial defect similar to
umbilical hernias.
· Small defects can be repaired under local anesthesia.
· Uncommonly, these defects can be sizable and contain omentum or other intra-abdominal viscera and may require
mesh repairs.
Epigastric hernias are better repaired because the defect is small and fat that has herniated from within the peritoneal
cavity is difficult to reduce.
Q.184 Buerger’s disease affects all except :
a. Small arteries
b. Small veins
c. Medium size arteries
d. Multiparity
Your Ans. d Correct Ans. d

Solution. Ans-184: (d) Multiparity.


Ref:Bailey & Love 25/e 923
Sol:
§ Thromboangiitis obliterans(also known as Buerger's disease) is a recurring progressive inflammationand
thrombosis(clotting) of small and medium arteriesand veinsof the hands and feet. It is strongly associated with use of
tobaccoproducts, primarily from smoking, but also from smokeless tobacco
§ Buerger’s disease is not related with pregnancy or multiparity. (It is found in young males, smokers.)
Q.185 Nicoladoni Branham’s sign is seen in :
a. Buerger’ disease
b. Arteriovenous fistula
c. Raynaud’s disease
d. Paripheral aneurysm
Your Ans. b Correct Ans. b

Solution. Ans-185: (b) Arteriovenous fistula


Ref: Bailey and Love’s Surgery--923
Sol:
§ Nicoladoni sign/Branham sign/ Nicoladoni-Israel-Branhamsign, is the slowing of the heart rate in response to (manual)
compression of an arteriovenous fistula.
§ Arteriovenous fistula may be either congenital or acquired.
§ The structural effect of arterial blood flow on the veins is characteristic; they become dilated, tortuous and thick
walled (arterialized).
§ They physiological effect is that it can increase cardiac output, if big enough. Auscultation reveals a machinery
murmur.
Q.186 Which of the following is spared in lumbar sympathectomy :
a. L1
b. L2
c. L3
d. L4
Your Ans. a Correct Ans. a
Solution. Ans-186: (a) L1
Ref: Bailey and Love’s Surgery--924
Sol:
§ To preserve sexual functions, L1 is preserved.
Q.187 Seldinger needle is used for :
a. Suturing Muscles
b. Arteriography
c. Pulmonary biopsy
d. Lymphaography
Your Ans. c Correct Ans. b

Solution. Ans-187: (b) Arteriography


Ref: Bailey and Love’s Surgery--90
Sol:
§ Seldinger needleis used for cannulization or catheterization employing the seldinger technique. It has :
(i) Thin wall outer cannula ending in blunt tapered point.
(ii) Fitted stylet has sharp cutting tip with a short bevel.
(iii)Blunt, round ended stylet included for plugging of outer cannula after insertion.
(iv) Has kidney shaped flange for ease of handling.
§ Transfemoral arterial accessfor coronary angiography or angioplasty is the norm for most cardiologists.
§ Although this carries the advantage ofease of access as a result of the superficial location of the large caliber femoral
artery in the groin, it can also potentially cause rare vascular complications, such as pseudoaneurysms, arteriovenous
fistula, arterial occlusion, nerve injury and most seriously, retroperitoneal bleed.
Q.188 Pseudoarterial aneurysm in drug abuser’s seen in :
a. Radial
b. Brachial
c. Femoral
d. Carotid
e. Pedal
Your Ans. b Correct Ans. c

Solution. Ans-188: (c) Femoral


Ref: Schwartz 8/e p 732; Robbin’s 6/e p 524
Sol:
§ Pseudoarterial aneurysmare false aneurysm, usually secondary to trauma.
§ Such aneurysmis incomplete in arterial wall components (lined by adventitia only and sometimes by perivascular clot).
§ Femoral pseudoaneurysmsmay result from injury produced by intravenous drug abuse, femoral artery puncture for
angiography or femoral line insertion.
§ Pseudoaneurysm can also develop at the distal anastomosis of an aortofemoralbypass graft.
§ Uninfected small (< 5 cm) traumatic pseudoaneurysmcan often be treated by US guided compression of neck of
aneurysm or by thrombin injection.
Q.189 Inoperability of renal cell carcinoma is -
a. Stage I
b. Stage II
c. Stage III
d. Large (> 7 cm) growths
Your Ans. c Correct Ans. c

Solution. Ans-189: (c) Stage III


Ref:Read the text below
Sol:
· Surgical therapy is principally offered to patients with early renal cell carcinoma, in whom preoperative staging suggests
that the tumor is either size I or stage II, representing small (< 7 cm) or large (> 7 cm) growth limited to the kidney without
evidence of L.N. or metastatic disease.
Q.190 The test of choice for distinguishing chronic pancreatitis from carcinoma pancreas is:
a. Non- Contrast CT Scan
b. Dynamic contrast enhanced Spiral CT Scan
c. MRI
d. PET Scan
Your Ans. b Correct Ans. d
Solution. Ans-190: (d) PET Scan
Ref:Schwartz’s Textbook of Surgery 8th ed. page no: 1280
Sol:
· PET is a developing tool which is very fast growing and is becoming widely available and helps distinguishing between
chronic pancreatitis and carcinoma pancreas.
Q.191 One of the following statement go wrong with paraneoplastic syndrome. Resulting from renal cell carcinoma:
a. May present with hepatic dysfunction
b. RCC is sometimes referred as "Internist's tumor"
c. Paraneoplastic findings resolve following nephrectomy
d. Anaemia is never present
Your Ans. d Correct Ans. d

Solution. Ans-191: (d) Anaemia is never present


Ref:Schwartz 8th edt.-1531
Sol:
· Patients with renal cell carcinoma can also present with paraneoplstic manifestation such as anaemia,hepatic
dysfunction (Stanffer's syndrome), cachexia, polycythemia and hypercalcemia.
· Paraneoplastic findingsresulting from localized disease resolve following nephrectomy.
· Diagnosing a case of RCC may be easy by the presence of paraneoplastic findings, so referred as "Internist's
tumor".
Q.192 A46-year-old man is on a waiting list to secure a renal transplant. The genetic locus of transplant antigens in
humans is known as which?
a. Rhesus (Rh)
b. Ig (Immunoglobulin) A and IgM
c. Human leukocyte antigen (HLA)
d. Hepatitis B surface antigen (HBsAg)
Your Ans. c Correct Ans. c

Solution. Ans-192: (c) Human leukocyte antigen (HLA)


Ref:Read the text below
Sol:
· HLA was one of the first studied antigens.The transplant antigen is located on the surface. The strongest transplant
antigen is known as the major histocampatibility complex (MHC) and is found in humans on chronomosome 6.
· The higher the number of MHC matches, the better chance of survival for the allograft.
· However, zero MHC-matched grafts have been placed due to overwhelming demands.
· On the other hand, mismatch in ABO and Rh group results in hyperacute rejection are elevated.
Q.193 Which one is not a feature of hypospadias ?
a. Incontinence
b. Hooded prepuce
c. Chordee
d. Meatal stenosis
Your Ans. a Correct Ans. a

Solution. Ans-193: (a) Incontinence


Ref: Bailey and Love’s-1363
Sol:
§ Hypospadias : Hypospadiasis the MC congenital malformation of the urethra.
§ “Hooded prepuce” : The external meatus opens on the underside of the penis or the perineum and the ventral aspect of the
prepuce is poorly developed.
§ Types of hypospadias according to position of the meatus : Glanular, coronal, Penile and penoscrotal, Perineal
hypospadias.
§ “Chordee”.: The moresevere varieties of hypospadias represent and absence of the urethra and corpus spongiosum distal to
the ectopic opening.
Q.194 A 12-year-old boy experiences sudden severe pain in the right testis. On palpation there is exquisite tenderness.
The most likely diagnosis
a. Spontaneous hemorrhage
b. Torsion
c. Strangulated hernia
d. Epidydimitis
Your Ans. b Correct Ans. b
Solution. Ans-194: (b) Torsion
Ref: Bailey and Love’s, 25th-1380.
Sol:
§ Torsion of the testisis a surgical emergency because it causes strangulation of gonadal blood supply with subsequent
testicular necrosis and atrophy.
§ Clinically : Acute scrotal swelling in children indicates torsion of the testis until proven otherwise.
§ Tremendous amount of painis characteristic; dizziness and nausea maybe present.
§ Treatment : Prompt exploration, untwisting and fixation is the only way to save the torted testis. The patient should be
counseled and consented for orchidectomy before exploration. The anatomic abnormality is bilateral and the contralateral
testis should also be fixed.
Q.195 In lateral anal sphincterotomy, the following sphincter is divided
a. Subcutaneous part of external sphincter
b. Deep part of external sphincter
c. Internal sphincter
d. Puborectalis sphincter
Your Ans. a Correct Ans. c

Solution. Ans-195: (c) Internal sphincter


Ref: Bailey and Love’s, 25th dn. Pg 1253. ; Harrison’s, 17th -1909
Sol:
§ Lateral internal sphincterotomy is the procedure of choice for chronic anal fissure or an acute fissure that remains
severely symptomatic after a prolonged course of nonoperative measures.
§ The aim of surgery is to break the vicious cycle of internal sphincter spasm.
§ One third of the distal internal sphincter upto, but not above the dentate line is divided under anesthesia.
§ Lateral internal sphincterotomy MC leads to incontinence in women.
Q.196 In evaluating the role of the autonomic nervous system related to urinary incontinence that developed in a 67-
year-old man after prostatectomy, it is determined that the sympathetic nerves are injured. What is the natural
hormone in the catecholamine pathway?
a. Norepinephrine
b. Dopamine
c. Vasoactive intestinal peptide (VIP)
d. Isoproterenol
Your Ans. a Correct Ans. a

Solution. Ans-196: (a) Norepinephrine


Ref:Read the text below
Sol:
· The metabolic pathway of catecholamines is initiated by conversion of tyrosine to dopa, which in turn, forms dopamine.
· Dopamine forms norepinephrine, which is the precursor of epinephrine.
· Epinephrine is the main amine secreted during life and is concerned with the “fight or flight” reaction.
Q.197 True about external haemorrhoids below the dentate line is –
a. Painful
b. Ligation is done as the management
c. May turn malignant
d. Skin tags are not different from external haemorrhoids
Your Ans. a Correct Ans. a
Solution. Ans-197: (a) Painful
Ref: Bailey & Love’s ‘Short Practice of Surgery’; 24/e, pg 1262
Sol:
· Unlike internal haemorrhoids, external haemorrhoids consist of a conglomerate group of distinct clinical
entities.
· A thrombosed external haemorrhoid is commonly termed a perianal haematoma.
· It is a small clot occurring in the perianal subcutaneous connective tissue, usually superficial to the corrugator cutis
ani muscle.
· The condition is due to back pressure on an anal venule consequent upon straining at stool, coughing or lifting a
heavy weight.
· The condition appears suddenly and is very painful.
· On examination, a tense, tender swelling that resembles a semi-ripe black currant is seen.
· The haematoma is usually situated in a lateral region of the anal margin.
· Untreated, it may resolve, suppurate, fibrose and give rise to a cutaneous tag, or burst and exrude the clot, or continue
bleeding.
· In the majority of cases, resolution or fibrosis occurs, forming a skin tag.
· Provided it is seen within 36 hours of the onset, a perianal haematoma is best treated as an emergency.
· Under local anaesthesia, the haemorrhoid is bisected, and the two halves are excised together with 1.25 cm of
adjacent skin.
· This leaves a pear-shaped wound that is allowed to granulate.
· The relief of pain is immediate and a permanent cure is certain.
· On the rare occasions in which a perianal haematoma is situated anteriorly or posteriorly, it should be treated
conservatively because of the liability of a skin wound in these regions to become an anal fissure.
Q.198 In a deceleration accident which of the following fixed part of aorta is not ruptured?
a. At level of aortic valve
b. Where aorta goes behind the esophagus
c. Where it pierces the crura of diaphragm
d. At Ligamentum arteriosum
Your Ans. b Correct Ans. b

Solution. Ans-198: (b) Where aorta goes behind the esophagus


Ref:Schwartz’ ‘Principles of Surgery’; 8/e,
Sol:
· Perhaps the most feared occult injury in trauma surgery is a tear of the descending thoracic aorta.
· Widening of the mediastinum on AP chest X-ray strongly suggests this injury.
· The widening is caused by the formation of a hematoma around the injured aorta, which is temporarily contained
by the mediastinal pleura.
· Posterior rib fractures and laceration of small vesselsalso can produce similar hematomas.
· Should the hematoma ruptureinto the chest with an aortic injury, the patient will exsanguinate in seconds.
· However, it is well established that this injury can occur with an entirely normal chest x-ray, although the incidence is
approximately 2%.
· Because of this and the dire consequences of missing the diagnosis, CT and angiography are frequently performed
based on the mechanism of injury.
· Aortic tears occur when shearing forces are created in the chest.
· This is most often seen in high-energy transfer deceleration motor vehicle injuries with frontal or lateral impact.
· However, it may also occur following an ejection injury or fall.
· The tear usually occurs just distal to the left subclavian artery, where the aorta is tethered by the ligamentum
arteriosum.
· In 2 to 5% of cases the tear occurs in the ascending aorta, transverse arch, or at the diaphragm.
· Dynamic spiral CT is an excellent screening test.
· Positive findings are a hematoma around the aorta or injury of the aorta.
· This test appears to be highly sensitive, but its specificity is unknown.
· A clearly widened mediastinum on chest x-ray or abnormalities on CT are an absolute indication for emergent aortography.
Q.199 The most common early post-operative complication of ileostomy is –
a. Obstruction
b. Necrosis
c. Diarrhea
d. Prolapse
Your Ans. b Correct Ans. b
Solution. Ans-199: (b) Necrosis
Ref: Sabiston’s ‘Textbook of Surgery’; 18/e,
Sol:
Stomal complications:
Early Late
Poor location Prolapse
Retraction Stenosis
Ischemic necrosis Parastomal hernia
Stoma
Detachment Fistula formation
Abscess formation Gas
Opening wrong end Odour
Parastomal varices
Excoriation Dermatoses
Peristomal skin
Dermatitis Cancer
Skin manifestations of inflammatory bowel disease
Bowel obstruction
Systemic High output
Non-closure

Q.200 After doing a thoraco-abdominal graft patient he was unable to move his both lower limbs. The likely cause is –
a. Damage to thoracic splanchnic nerve
b. Damage to Lumbar spinal nerves
c. Ischaemia of lower limbs
d. Insufficiency of Arteria Magna Radicularis
Your Ans. d Correct Ans. d

Solution. Ans-200: (d) Insufficiency of Arteria Magna Radicularis


Ref:Read the text below
Sol:
The patient presents with an acute onset complete inability to move the limbs following a thoraco-abdominal graft.
Let us assess the options one by one-

Option (1):Damage to thoracic sphlancnic nerves.


Damage to thoracic sphlancnic nerves will cause involvement of the thoracic visceral organs. It might affect the functioning of
the lungs and heart. But in either case, it would not lead to inability to move both the lower limbs.

Option (2): Damage to Lumbar spinal nerves


The muscles of the limbs are supplied by segments ranging from L4 to S2. Hence, damage to lumbar spinal nerves might
partially affect some of the muscles of the lower limbs; it would not lead to complete inability to move the lower limbs

Option (3): Ischemia of lower limbs


Ischemia of lower limbs would usually present with the initial features of claudication. It does not lead to an inability to
completely move the lower limbs.

Option (4): Insufficiency of Arteria Magna Radicularis


A complete inability to move both the lower limbs is possible when the part of spinal cord supplying the lower limbs is affected.
This might occur when the blood supply to the part of the spinal cord is suddenly compromised, as might occur
due to injury to arteria magna radicularis during any surgery.

Hence, the most likely answer to this question is option ‘4’.


Q.201 Consider the following statements regarding endometrial cancer :

1. It is an adenocarcinoma.

2. Treatment of choice is radiotherapy.

3. It is associated with obesity, diabetes and hypertension.

Of these statements
a. 1, 2 and 3 are correct
b. 1 and 2 are correct
c. 2 and 3 are correct
d. 1 and 3 are correct
Your Ans. a Correct Ans. d
Solution. Ans-201: (d) 1 and 3 are correct
Ref.:BRS Gynecology- 324
Sol :
§ Risk factors for endometrial carcinoma –No pregnancies, prolonged reproductive time, unopposed estrogen and triad of
DM, hypertension and obesity.
§ 20% cases are nulliparus, and with late menopause in 75%.
§ Principal histologic subtypes – Adenocarcinoma (60-65%), adenoacanthoma (22%), the remaining subtypes – papillary
serous carcinoma, clear cell adenocarcinoma, adenosquamous carcinoma and secretory carcinoma. Papillary serous and clear
cell subtypes are associated with a poorer 5-year survival rate.
§ Management – of endometrial carcinoma is primarily surgical with other modalities as adjuvants, depending on
tumor grade and stage at diagnosis.
Q.202 Which of the following procedure confirms the diagnosis of pelvic inflammatory disease-
a. White blood cell count
b. Cervical gram stain
c. Culdocentesis
d. Laparoscopy
Your Ans. d Correct Ans. d

Solution. Ans-203: (c) Lower segment caesarean section


Ref: Dutta DC. Textbook of Obstetrics- 400

Q.203 During the first stage of labor, the membranes ruptured and the umbilical cord prolapsed. Fetal heart rate is 120-
130 per minute; cervix is about 5 cm dilated. Which one of the following will be the appropriate treatment ?
a. Repose cord and raise foot end of bed
b. Repose cord and apply ventouse
c. Lower segment caesarean section
d. Forceps delivery under general anaesthesia.
Your Ans. c Correct Ans. c

Solution. Ans-204: (b) 60 gm


Ref: Dutta - 145
Sol:
Uterus Weight
· Non-pregnant state About 50 mg (50-80)
· At term 900-1000 gm
· At 6 wk after delivery 60 gm (similar to non-pregnant state)
Q.204 At the end of the normal puerperium, the uterine weight is :
a. 40 gm
b. 60 gm
c. 80 gm
d. 100 gm
Your Ans. b Correct Ans. b

Solution. Ans-204: (b) 60 gm


Ref: Dutta - 145
Sol:
Uterus Weight
· Non-pregnant state About 50 mg (50-80)
· At term 900-1000 gm
· At 6 wk after delivery 60 gm (similar to non-pregnant state)

Q.205 The contraindication of injection PGF2α is -


a. Cardiovascular disease
b. Uterine scar
c. Peptic ulcer
d. Hypertension
Your Ans. b Correct Ans. b

Solution. Ans-205: (b) Uterine scar


Ref: Read the text below
Sol:
· PGF2a is synthesized by decidua in vivo, it promotes myometrial contractility. The main obstetric uses of this
compound are induction of abortion (MTP and missed abortion) and management of atonic PPH.
· The contraindications are hypersensitivity to the compound and uterine scar. PGF2a acts predominantly on
myometrium.
· The unpleasant side effects are nausea, vomiting, diarrhea, pyrexia or bronchospasm but the symptoms subside promptly
due to rapid metabolism of PGs.
Q.206 Preconception supplement of folic acid to woman :-
a. Minimize foetal neural tube defect
b. Help to conceive faster
c. Minimize iron deficiency anemia
d. Prevents ectopic pregnancy
Your Ans. a Correct Ans. a

Solution. Ans-206: (a) Minimize foetal neural tube defect


Ref: Read the text below
Sol:
· Preconception supplement requiresin elderly women (above 29 years), women with history of congenital birth defects.
· Folic acid (5 mg BD) supports ovum and improves quality of egg.
Q.207 The drug of choice for ovulation induction in a 28 years old infertile patient with polycystic ovarian syndrome
(PCOS) is -
a. GnRH analogues
b. Gonadotrophins
c. Clomiphene citrate
d. Metformin
Your Ans. c Correct Ans. c

Solution. Ans-207: (c) Clomiphene citrate


Ref: Read the text below
Sol:
· Clomiphene citrate is the drug of choice in PCOS patient for ovulation induction.
· Gonadotrophins are 2nd line drugsbecause chances of ovarian hyperstimulation syndrome are more with
gonadotrophins particularly in PCOS.
· Metformin is an adjuvant drug particularly in patients with features of insulin resistance.
Q.208 All of the following are associated with breech presentation at normal full term pregnancy, except:
a. Placenta accreta
b. Fetal malformation
c. Uterine anomaly
d. Cornual implantation of placenta
Your Ans. a Correct Ans. a

Solution. Ans-208: (a) Placenta accreta


Ref: Read the text below
Sol:
Causes of breech presentation :
1.Prematurity is the commonest cause of breech presentation.
2. Factors preventing spontaneous version.
(a) Twins
(b) Oligohydramnios
(c) Breech with extended legs
(d) Congential malformation of uterus such as septate or bicornuate uterus
(e) Short cord
(f) Intrauterine death of the fetus/fetal malformations.
3. Favourable adaptation :
(a) Hydrocephalus
(b) Placenta praevia
(c) Cornufundal attachment of the placenta
(d) Contracted pelvis
4. Undue mobility of the fetus :
(a) Hydramnios
(b) Multiparae with lax abdominal wall
Q.209 The fetal blood is separated from maternal blood with all except:
a. Extra embryonic mesoderm
b. Syncytio trophoblast
c. Cyto trophoblast
d. Decidua parietalis
Your Ans. d Correct Ans. d

Solution. Ans 209: (d) Decidua parietalis


Reference: Read the text below
Sol:
The mucous membrane lining the main cavity of the pregnant uterus elsewhere than at the site of attachment of the
chorionic vesicle. Also called decidua vera

Q.210 The foetal circulation is first intact and functional, separated from maternal circulation at the age of:
a. 8 days
b. 12 days
c. 17 days
d. 21 days
Your Ans. d Correct Ans. d

Solution. Ans 210: (d) 21 days


Reference: Read the text below
Sol:
The foetal circulation is first intact and functional, separated from maternal circulation at the age of 3 wks.
Q.211 Zavenelli’s maneuver done in :
a. Shoulder dystocia
b. Deep transverse arrest
c. Retained placenta
d. Face presentation.
Your Ans. a Correct Ans. a

Solution. Ans 211: (a) Shoulder dystocia


Reference: Read the text below
Sol:
Zavanelli maneuver is an obstetric maneuver that involves pushing back the delivered fetal head into the birth canal in
anticipation of performing a cesarean section in case of shoulder dystocia.
Shoulder dystocia
· Mc Roberts maneuver (most commonly performed/first performed maneuver)
· Wood’s maneuver
· Aavanelli maneuver.

Breech extraction
· Burns Marshall technique (for after coming head of Breech).
· Modified Mauriceau Smellie – Veit technique (Malar flexion an shoulder traction)
· Lovset’s maneuver (for extended arms.
Pinard’s maneuver (for frank breech extraction.)
Q.212 Early cord clamping is done is all except
a. Rh isoimmunization
b. Fetal asphyxia
c. Postmaturity
d. Prematurity
Your Ans. d Correct Ans. c

Solution. Ans 212: (c) Postmaturity


Reference: Read the text below
Sol:
Early cord clamplingis done in :
· Rh incompatibility.
· Asphyxiated baby
· Diabetic mother.
· Preterm labour/grown restricted fetus due to risk of hypervolemia.
HIV positive mother.
Q.213 Fundal height is more than period of gestation in all except
a. Hydramnios
b. IUD
c. Twin pregnancy
d. Hydatidiform mole
Your Ans. b Correct Ans. b

Solution. Ans 213: (b) IUD


Reference: Read the text below
Sol:
Condition where the height of uterus is more than the period of amenorrhea.
· Mistaken dates
· Twins
· Polyhydramnios
· Big baby
· Pelvic tumours – Ovarian/fibroid
· Concealed accidental hemorrhage.

Conditions where the Height of Uterus is less than the Period of Amenorrhea.
· Mistaken dates.
· Scanty liquor
· Fetal growth restriction
· Intrauterine fetal death.
Q.214 In a female with appendicitis in pregnancy treatment of choice is:
a. Surgery at earliest
b. Abortion with appendectomy
c. Surgery after delivery
d. Continue pregnancy with medical Rx.
Your Ans. a Correct Ans. a

Solution. Ans 214: (a) Surgery at earliest


Reference: Read the text below
Sol:
· Diagnosis of appendicitisduring pregnancy is difficult as symptoms of appendicitis viz nausea, vomiting anorexia are
normally present in pregnancy.
· But once the diagnosis is made immediate surgery should be done.
Q.215 Patau syndrome is trisomy of ?
a. Chromosome 21
b. Chromosome 13
c. Chromosome 18
d. Chromosome 45
Your Ans. b Correct Ans. b

Solution. Ans 215: (b) Chromosome 13


Reference: Read the text below
Sol:
· Down syndromeis a trisomy of chromosome 21. It is the most common nonlethal trisomy. Patau syndrome is trisomy 13
and Edwards syndrome is trisomy 18.
· Turner syndrome is 45 X, a monosomy.
· Klinefelter syndromeis the presence of an extra X chromosome in a male resulting in 47 X-X-Y.
Q.216 Which of the following Is the biggest risk factor for the development Down syndrome?
a. Cigarette smoking
b. Lack of folic acid supplementation
c. Maternal age at conception
d. Family history
Your Ans. c Correct Ans. c

Solution. Ans 216: (c) Maternal age at conception


Reference: Read the text below
Sol:
· The risk of trisomy 21 is directly related to maternal age as a result of maternal nondisjunction.
· The risk of having a child with Down syndrome increases in a gradual, linear fashion until about age 30 and increases
exponentially thereafter. Women who will be 35 years or older at the time of delivery should be offered chorionic villus
sampling or secondtrimester amniocentesis.
· Women younger than 35 years should be offered maternal serum screening at 16–18 weeks of gestation.
· The maternal serum markers used to screen for trisomy 21 are alpha-fetoprotein, unconjugated estriol, and hCG.
· The use of ultrasound to estimate gestational age improves the sensitivity and specificity of maternal serum
screening.
Q.217 "Decapitation" is an indication for-
a. Shoulder dystocia
b. Neglected shoulder presentation with dead fetus
c. Large hydrocephalus
d. Cephalic presentation with obstructed labour with dead fetus
Your Ans. d Correct Ans. b

Solution. Ans-217: (b) Neglected shoulder presentation with dead fetus


Ref: Read the text below
Sol:
· Decapitation is a destructive operation whereby the fetal head is severed from the trunk and delivery is completed with
the extraction of trunk and that of decapitated head per vaginum.
· It is indicated in neglected shoulder presentation with dead fetus where neck is easily accessible.
· For shoulder dystocia – Cleidotomy may be done when baby is dead.
· For hydrocephalus –Craniotomy can be performed which is also an indication for cephalic presentation with obstructed
labour when fetus is dead.
Q.218 For delivery of preterm baby, to cut short the II stage of labour, one of the following may be indicated-
a. Ventouse
b. Forceps
c. Both
d. None
Your Ans. b Correct Ans. b

Solution. Ans-218: (b) Forceps


Ref: Read the text below
Sol:
· It is necessary to cut short the II stage of labour in case of delivery of preterm baby because these babies are prone to
intraventricular haemorrhage.
· Forceps act as protective cage for preterm head.
Ventouse is contraindicated in these because chance of scalp avulsion and subaponeurotic haemorrhage is more.
Q.219 Battledore insertion of the umbilical cord means:-
a. Insertion of the cord at the periphery of the placenta
b. Cord insertion in the membranes away from the placenta
c. Cord insertion half-way between the centre and periphery of the placenta
d. None of the above
Your Ans. a Correct Ans. a

Solution. Ans-219: (a) Insertion of the cord at the periphery of the placenta
Ref: Read the text below
Sol:
· Battledore insertionof the umbilical cord means Insertion of the cord at the periphery of the placenta
Cord insertion in the membranes away from the placenta is associated with velamentous insertion of the cord, and vasa
praevia.
Q.220 Following is a class A FDA drug used in pregnancy:-
a. Methyl dopa
b. Aspirin
c. Heparin
d. All of the above
Your Ans. d Correct Ans. d

Solution. Ans-220: (d) All of the above


Ref: Read the text below
Sol:
· Other class A FDA drugs are penicillin - cephalosporin, Erythromycin - clindamycin. They have no teratogenicity.
· FDA B drugsare antihistamines, Decongestants. They showed no evidence of risk in humans. FDA C :- Risk can not be ruled
out. These drugs are metronidazole, Ciprofloxacin, Amino glycosides.
FDA D:-Positive evidence of risk like Carbamazepine, Valproic acid, alcohol, Warfarin. FDA X:- Contraindicated in pregnancy
like Tetracycline, Doxycycline, and Minocycline.
Q.221 Which one of the following is not a component of APGAR scoring in the newborn ?
a. Heart rate
b. Muscle tone
c. Respiratory effort
d. Body temperature
Your Ans. d Correct Ans. d

Solution. Ans-221: (d) Body temperature


Ref: Ghai - 144
Sol:
Score 0 1 2
Respiratory effort None Slow, irregular Good crying
Heart rate/min Absent < 100 > 100
Colour of the body Blue or pale Body pink, extremities blue Complete pink
Muscle tone Flaccid Flexion of extremities Active body movements
Reflex stimulation No, response Grimace Cry, cough or sneezes
Q.222 Ideal time for blood sugar screening for diabetes in a pregnant female is :-
a. 24-28 weeks
b. 28-36 weeks
c. 36 weeks
d. 1st trimester
Your Ans. a Correct Ans. a

Solution. Ans-222: (a) 24-28 weeks


Ref: Read the text below
Sol:
The guidelines which are usually followed regarding screening for diabetes :
For high risk cases → Screening should be done as soon as possible.
For average risk cases → Screening done at 24-28 weeks.
For low risk cases → blood glucose testing not routinely required.
– "Screening for gestational diabetes should be performed between 24-28 weeks in those women not known to have glucose
intolerance in their earlier pregnancy".
Q.223 All predispose to isoimmunisation in a Rh -ve female except :-
a. Advanced maternal age
b. Antepartum haemorrhage
c. Caesarian section
d. Post dated pregnancy
Your Ans. a Correct Ans. a

Solution. Ans-223: (a) Advanced maternal age


Ref: Read the text below
Sol:
Conditions predisposing to isoimmuni-zation in Rh –ve female :
· Abortion
· Chorionic villous sampling
· Amniocentesis
· Antepartum haemorrhage
· Attempted version
· Caesarian section
· Manual removal of placenta
Recent studies state that there is continuous feto-maternal bleed occurring throughout normal pregnancies. So it is not advisable
to go beyond the expected date of pregnancy. Hence, postdated pregnancy is another risk-factor for isoimmunization.
Q.224 In a case of endometrial cancer, if metastasis is seen in the vagina, what FIGO stage it would be :
a. Stage IIIa
b. Stage IIIb
c. Stage IIIc
d. Stage IVa
Your Ans. b Correct Ans. b

Solution. Ans-224: (b) Stage IIIb


Ref.:Shaw’s Textbook of Gynaecology, (14th Ed.) Pg. 375
Sol :
FIGO staging of endometrial carcinoma :
Stage 0 — Carcinoma is situ
I – Confined to corpus
Ia – Limited to endometrium
Stage
Ib — < ½ of myometrium or < 8 cm
Ic – > ½ of myometrium or > 8 cm
II – Involved corpus and cervix
Stage II A – Endocervical glandular involvement
IIB – Endocervical glandular involvement
III – Extension outside uterus (within pelvis, not outside true pelvis)
IIIa – Invade serosa or adnexa or positive peritoneal cytology.
Stage
IIIb – Vaginal metastasis
IIIc – Pelvic or para-aortic node metastasis

IV – Distant metastasis (out side the true pelvis).


Stage IVa – Invade bladder or bowel mucosa involvement.
IVb – Distant metastasis/extra-abdominal or inguinal lymph node
Q.225 Which one of the following tumours is not malignant?
a. Brenner’s tumour
b. Dysgerminoma
c. Imature teratoma
d. Krukenburg’s tumour
Your Ans. a Correct Ans. a

Solution. Ans-225: (a) Brenner’s tumour


Ref.:BRS Gyn and Obs, (2nd Ed.), Pg. 335
Sol :
§ Transitional cell (Brenner) tumours –are almost always benign (only 2% being malignant).
§ Among germ cell tumours, only teratoma mature and gonadoblastoma have 0% malignant.
Rest all teratoma immature and dysgerminoma, endodermal sinus, embryonal carcinoma, non-gestational choriocarcinoma
are 100% malignant
Q.226 In normal neonates,ossification centers are usually radiographically visible at birth in which of the following sites?
a. Patella
b. Lunate (carpal)
c. Proximal tibia
d. Head of the femur
Your Ans. c Correct Ans. c

Solution. Ans 226: (c) Proximal tibia


Ref– Read the text below
Sol:
· Ossification centersusually present at birth include the distal femur and the proximal tibia.
· Reference standards for bone maturation facilitate estimation of bone age. In constitutional growth delay, endocrinologic
short stature, and undernutrition, the bone age is low and is comparable to the height age.
· In familial short stature, the bone age is normal (compared to chronological age).
Q.227 Which of the following statements regarding temper tantrums is correct?
a. Most often indicate a serious psychosocial problem
b. Usually appear at the end of the first year
c. Peak prevalence is between 4 and 6 years
d. Routinely occur 8–10 times per day
Your Ans. c Correct Ans. b

Solution. Ans 227: (b) Usually appear at the end of the first year
Ref– Read the text below
Sol:
· As children mature, they learn what behaviors are acceptable and how much power they are able to wield by
testing limits.
· Control is a central issue. Inability to control some aspect of the external world, such as how to make a certain toy work or
when to leave, often results in a loss of internal control, that is, a temper tantrum. Fear, overtiredness, or physical
discomfort can also evoke tantrums.
· When they are reinforced by intermittent rewards, as when the parent occasionally gives in to the child’s demands,
tantrums can also become an entrenched strategy for exerting control by the toddler.
· Tantrums lasting more than 15 minutes or happening regularlymore than three times daily can reflect underlying
medical, emotional, or social problems. Tantrums normally appear at the end of the first year of life and peak at 2–4 years.
Frequent tantrums after 5 years of age usually persist throughout childhood. Clearly, this is an undesirable outcome
which, in nearly all children, is avoidable
Q.228 Thalassemia occurs due to which mutation
a. Missence
b. Splicing
c. Transition
d. Truncation
Your Ans. d Correct Ans. b

Solution. Ans 228: (b) Splicing


Ref– Read the text below
Sol:
· Mutation causing thalassemiacan affect any step in the pathway of globin gene expression including deletion,
nonsence and frameshift mutation.
The most common forms arise from mutations that derange splicing of M-RNA precursor or prematurely terminate
translation of the M-RNA.
Q.229 A child has repeated vomiting and developed metabolic alkalosis. The treatment given is
a. Ringer lactate
b. I.V. normal saline and potassium
c. ORS
d. I.V. Normal saline.
Your Ans. b Correct Ans. b

Solution. Ans 229: (b) I.V. normal saline and potassium


Ref– Read the text below
Sol:
Potassium supplementation should not be given to all patient of hypovolemia until hypokalemia is proved. But in following
conditions Potassium supplementation should be given as these are associated with hypokalemia
· Prolonged diarrhea
· Hypokalemic Metabolic alkalosis.
Q.230 The requirement of potassium in child is
a. 1-2 mEq/kg
b. 4-7 mEq/kg
c. 10-12 mEq/kg
d. 13-14 MEq/kg
Your Ans. b Correct Ans. a

Solution. Ans 230: (a) 1-2 mEq/kg


Ref– Read the text below
Sol:
· Potassium is plentiful in food.
· Dietary consumption varies considerably even though 1-2 mEq/kg is the recommended intake.
· Most absorption occurs in the small intestine whereas the colon exchange body potassium for luminal sodium.
Q.231 The recommended agent for treatment of Rocky Mountain spotted fever in a child younger than 8 yr is:
a. Ceftriaxone
b. Chloramphenicol
c. Doxycycline
d. Clindamycin
Your Ans. d Correct Ans. c

Solution. Ans 231: (c) Doxycycline


Ref– Read the text below
Sol:
· Chloramphenicol and tetracyclineshave proven efficacy against Rocky Mountain spotted fever, but chloramphenicol may
be associated with higher mortality.
· Dental stainingis unlikely with a single course of a tetracycline.
· Doxycycline is recommendedbecause the risk of dental staining is less than with other tetracyclines.
Q.232 What is the most worrisome complication of Kawasaki disease ?
a. Encephalitis
b. Coronary artery aneurysm
c. Cardiac valve dysfunction
d. Intracerebral hemorrhage
Your Ans. b Correct Ans. b

Solution. Ans 232: (b) Coronary artery aneurysm


Ref– Read the text below
Sol:
· Nearly a quarter of untreated children will develop coronary artery dilatation.
· This is most common cause of acquired heart disease in children younger than 5 years of age.
· The coronary artery dilatation can result in aneurysm formation and myocardial infarction.
Q.233 A baby can follows an object with 1800 ,can hold neck, but can sit without support. The age of the baby is:
a. 1 month
b. 3 month
c. 6 month
d. 9 month
Your Ans. d Correct Ans. b
Solution. Ans 233: (b) 3 month
Ref– Read the text below
Sol:
A baby can follow an object with 180 degrees at 8 wks,can hold the neck at 3 months and Can sit with support in 5
months.
Q.234 A newborn has been diagnosed with aniridia.Which of the following tests should be performed on this patient?
a. Chest radiograph
b. Alpha-fetoprotein measurement
c. Renal function testing
d. Renal ultrasound
Your Ans. d Correct Ans. d

Solution. Ans 234: (d) Renal ultrasound


Reference: Read the text below
Sol:
· Aniridia involvesthe whole eye and should not be thought of as an isolated iris defect. Iris tissue usually is present but
is hypoplastic.
· It usually occurs bilaterally and vision is severely impaired. Aniridia can either be a sporadic occurrence or can have an
autosomal dominant inheritance pattern. One-fifth of the patients with sporadic anirida will develop Wilms tumor.
· These patients will need surveillance with renal ultrasound every 4–6 months.
There is also an association between Wilms tumor, aniridia, genitourinary anomalies, and mental retardation (WAGR
acronym).
Q.235 A 4-yr-old boy is evaluated for his first generalized tonic-clonic seizure, which lasted 10 min. There is no history of
illness or fever, and findings on examination an hour after the seizure are completely normal. The most
appropriate management is:
a. Begin therapy with carbamazepine
b. Order an EEG
c. Order a CT scan of the brain
d. Order an MRI study of the brain
Your Ans. b Correct Ans. b

Solution. Ans 235: (b) Order an EEG


Reference: Read the text below
Sol:
· An EEG will help define the presence of a seizure focus and if there is a specific seizure type.
· A normal EEG may help the pediatric neurologist determine if anticonvulsant therapy is needed.
Q.236 A non ventilated preterm baby in incubator is under observation. Which is the best way to monitor the baby’s
breathing and detect apnea?
a. Infrared thoracic movement study
b. Capnography
c. Nasal digital temperature monitoring
d. Impedence technique.
Your Ans. b Correct Ans. d

Solution. Ans 236: (d) Impedence technique


Reference: Read the text below
Sol:
· The respiratory monitor based on impedance technique measures changes in the electrical resistance during
breathing.
· Impedance pneumography is based on the relationship between respired volume (ΔV) and changes of transthoracic
impedance (Δ/Z/) during breathing.
· The electrode is fixed on the chest wall to pick up signals which are displayed as respiratory rate.
Capnography –It is a simple non invasive method to assess arterial Co2 It is used to asses the placement of ET tube in
esophagus or trachea.
Q.237 All of the following statements regarding Hirschsprung disease are true except:
a. It is uncommon in preterm infants
b. Males are affected more commonly than females
c. It is associated with VATER syndrome
d. It is associated with trisomy 21
Your Ans. a Correct Ans. c
Solution. Ans-237: (c) It is associated with VATER syndrome
Ref: Read the text below
Sol :
· Imperforate anus is associated with VATER syndrome.
· Hirschsprung disease is the most common anatomic cause of lower intestinal obstruction in neonates.
Q.238 A 9-yr-old boy develops local pain and swelling of the left lateral aspect of the skull. Physical examination reveals
marked tenderness and warmth. Plain film shows a well-defined radiolucent skull lesion. The skeletal survey is
otherwise normal. The most likely diagnosis is:
a. Eosinophilic granuloma
b. Hand-Schüller-Christian disease
c. Osteomyelitis of the skull
d. Brain abscess
Your Ans. b Correct Ans. d

Solution. Ans-238: (d) Brain abscess


Ref: Read the text below
Sol :
· Eosinophilic granuloma is a monostotic or polyostotic disease with no extraskeletal involvement. This differentiates
eosinophilic granuloma from other forms of Langerhans cell histiocytosis (Letterer-Siwe or Hand-Schüller- Christian
variants).
Q.239 Which of the following benign bone processes may be associated with visceral involvement?
a. Osteoid osteoma
b. Aneurysmal bone cyst
c. Osteochondroma
d. Langerhans cell histiocytosis
Your Ans. d Correct Ans. d

Solution. Ans-239: (d) Langerhans cell histiocytosis


Ref: Read the text below
Sol :
Children with Langerhans cell histiocytosis andbone lesions should be evaluated for visceral involvement because
treatment of Hand-Schüller- Christian disease and Letterer-Siwe disease is more complex and often requires systemic
therapy.
Q.240 All are signs of hydrocephalus in a neonate except
a. Enlarged head
b. Sunset sign
c. Crack pot sign
d. Depressed fontanelle
Your Ans. d Correct Ans. d

Solution. Ans-240: (d) Depressed fontanelle


Ref: Nelson -1990
Sol :
Clinical presentation of Hydrocephalous in Neonates :
o Accelerated rate of enlargement of the head is most prominent sign.
o Anterior fontanelle is wide open and bulging.
o Scalp veins are dilated
o Forehead is broad
o Eye may deviate downward-setting sun eye sign
o Long tract signs including brisk tendon reflex, spasticity, clonus and Babinski sign are common.
o Percussion of the skull may produce a “Cracked pot” or Macewen’s sign, indicating separation of the sutures.
Q.241 True statement regarding Brain tumor in children is :
a. Mostly is infra-tentorial
b. Papilledema is rare
c. Is the most common tumor in children
d. Hydrocephalus is rare
Your Ans. d Correct Ans. a
Solution. Ans-241: (a) Mostly is infra-tentorial
Ref: Nelson -1702
Sol :
§ In a survey of children with sampled brain tumors, a slight predominance of infratentorial tumor location (43.2%)
followed by supratentorial location (40.9%) is reported.
§ There is age related difference in primary location of tumor.
§ Within 1st year of life supratentorial tumours predominate and include most commonly choroid plexus complex tumors.
§ For 1-10 years of age, infratentorial tumours predominate and includes juvenile pilocytic astrocytoma and
Medulloblastoma.
§ After 10 year of age, once again, supratentorial tumours predominatewith the diffuse astrocytomas most common.
§ Juvenile piolocytic astrocytoma is the most common childhood brain tumour.
Q.242 10 years old Ramu has increasing muscle weakness and raised CPK levels. The most likely defect is in plasma
membrane of :
a. Nerves
b. Muscle fibers
c. Basement membrane
d. All body cells
Your Ans. b Correct Ans. b

Solution. Ans-242: (b) Muscle fibers


Ref: Harrison 16th edition/2527
Sol :
§ CK is preferred muscle enzyme to measure in the evaluation of myopathies. Damage to muscle causes the CK to leak from the
muscle fiber to the serum.
§ The CK-MM isoenzyme predominates in skeletal muscle while CK-MB is the marker for cardiac muscles
§ It is clear from above that increase in CPK level suggest muscle cell damage.
Q.243 All of the following are features of Juvenile CML except
a. Thrombocytopenia
b. Fetal Hb is increased
c. Philadelphia chromosome is positive
d. Lymphadenopathy
Your Ans. a Correct Ans. c

Solution. Ans-243: (c) Philadelphia chromosome is positive


Ref: O.P. Ghai -465
Sol :
§ Juvenile CML is mostly seen in infancy and early childhoodbelow the age of 5 years.
§ Philadelphia chromosome is always negative.
Feature Adult CML Juvenile CML
1. Age 10-12 years < 2 years
2. Bleeding Manifestation Absent Frequent
3. Thrombocytopenia Uncommon Frequently present
4. Lymphadenopathy Rare Frequent
5. Splenomegaly Marked Variable
6. WBC count at diagnosis > 10,000/CC < 10,000/CC
7. HbF Normal Increased
8. Immunoglobulin Normal Increased
9. Response to Busulphan Good Poor
10.Survival 2.5-3 years 9 months

Q.244 Single gene defect causing multiple unrelated problem :


a. Pleotropism
b. Pseudo dominance
c. Penetrance
d. Anticipation
Your Ans. a Correct Ans. a
Solution. Ans-244: (a) Pleotropism
Ref: Robbin’s -150
Sol :
§ A single mutant gene may lead to many end effects, termed as – pleotropism. Example-Sickle cell anemia
§ Conversely, mutation at several genetic loci may produce the same trait called – gentetic heterogeneity. Example-
Profound childhood deafness.
§ Penetrance – Some individuals inherit the disease but are phenotypically normal. This is referred to as reduced
penetrance. Penetrance is expressed in Mathematical terms, 50% penetrance means that 50% of those who carry
the gene express the trait.
Q.245 Cataracts are noted in all of the following except:
a. Rubella (congenital)
b. Galactosemia
c. Galactokinase deficiency
d. Hyperoxygenation
Your Ans. d Correct Ans. d

Solution. Ans-245: (d) Hyperoxygenation


Ref: Read the text below
Sol :
· Hyperoxia in preterm neonates usually causes retinopathy of prematurity, which occasionally is associated with cataracts.
Oxygen does not directly cause cataracts.
Q.246 An 18-mo-old manifests pendular nystagmus, head nodding, and torticollis. Findings on a cranial MRI scan are
normal. This child is most likely to have:
a. Epilepsy
b. Congenital blindness
c. Neuroblastoma
d. Spasmus nutans
Your Ans. d Correct Ans. d

Solution. Ans-246: (d) Spasmus nutans


Ref: Read the text below
Sol :
· The acquired triad of nystagmus, head nodding, and torticollis, in its classic form, is self-limited and benign.
· Nonetheless, children with brain tumors may have signs resembling components of spasmus nutans.
Q.247 A 1-yr-old child presents with increased size of the cornea. Review of systems reveals history of increased tearing
and apparent sensitivity to light. The cornea appears cloudy. The most likely diagnosis is:
a. Retinoblastoma
b. Glaucoma
c. Chorioretinitis
d. Aniridia
Your Ans. b Correct Ans. b

Solution. Ans-247: (b) Glaucoma


Ref: Read the text below
Sol :
· Symptoms of infantile glaucoma(glaucoma that begins in the first 3 years of life) include the classic triad of epiphora
(tearing), photophobia (sensitivity to light), and blepharospasm (eyelid squeezing), which are usually attributed to
corneal irritation.
· An increase in intraocular pressure leads to expansion of the globe, including the cornea.
Q.248 Metatarsus adductus is associated with all of the following except:
a. Out-toeing
b. Bilaterality in 50%
c. More common in firstborn
d. Nonoperative treatment
Your Ans. d Correct Ans. a

Solution. Ans-248: (a) Out-toeing


Ref: Read the text below
Sol :
· This condition gives the appearance of intoeing.
· It may be due to deformational forces in utero hence the association with being firstborn and with hip dysplasias.
Q.249 An infant develops cough and fever. The X-ray examination is suggestive of broncho-pneumonia. All of the
following viruses can be the causative agent except
a. Parainfluenza viruses
b. Influenza virus A
c. Respiratory synchytial virus
d. Mumps virus.
Your Ans. b Correct Ans. d

Solution. Ans 249: (d) Mumps virus


Reference: Read the text below
Sol:
· The peak attack rate for viral pneumonia is between the ages of 2 and 3 years and decrease there after.
· For viral pneumonia, RSV is the major pathogen especially in children younger than 3 years of age.
Other common viruses causing pneumonia include–
· Parainfluenza viruses.
· Influenza viruses.
Adenoviruses.
Q.250 The use of intravenous lipids predisposes most significantly to catheterrelated infections and fungemia due to
which of the following organisms?
a. Candida albicans
b. Cryptococcus
c. Histoplasma
d. Malassezia furfur
Your Ans. a Correct Ans. d

Solution. Ans 250: (d) Malassezia furfur


Ref– Read the text below
Sol:
· Catheter-related infections and fungemiawith Malassezia furfur occur almost exclusively in patients receiving
intravenous lipids.
· The use of lipid emulsions containing mediumchain triglycerides inhibits the growth of Malassezia.
Q.251 Condyloma acuminate can be treated with which of the following ?
a. Podophyllin
b. Trichloroacetic acid
c. Interferon alpha
d. All of the above
Your Ans. d Correct Ans. d

Solution. Ans-251: (D) All of the above


Ref: Read the text below
Sol :
§ Condyloma acuminate can be treated with any one or combination of the following: topical podophyllin preparations,
topical trichloroacetic acid, cryotherapy, electrocautery, surgical or laser excision, oral cimetidine, intralesional
interferon alpha, topical 5-fluorouracil.
Q.252 Tinea versicolor is caused by which of the following organism ?
a. Pityrosporum
b. Microsporum
c. Trichophyton
d. Epidermophyton
Your Ans. a Correct Ans. a

Solution. Ans-252: (a) Pityrosporum


Ref: Read the text below
Sol :
§ Tinea versicolor is theonly tinea infection caused by a non-dermatophyte, pityrosporum.
§ The other tinea infections (pedis, corporis, capitis, cruris, unguium) are dermatophyte infections caused by organisms that
favor the keratinized cells of skin, hair, and nails.
Q.253 A 20-year old young female presented with a relatively painless ulcer of 3 cm on the labia majora with raised
margins. Which of the following organisms would have most likely caused this ulcer?
a. Neisseria gonorrheae infection
b. Herpes simplex virus infection
c. Chlaymdia trachomatis cervicitis
d. Treponema pallidum infection
Your Ans. d Correct Ans. d

Solution. Ans 253: (d) Treponema pallidum infection


Ref– Read the text below
Sol:
Painless ulcer with raised margins is suggestive of a chancre. Salient features of all genital ulcer disease is given
below.

Chancre (1o Herpes


Features Chancroid LGV GI (Donovanosis)
Syphilis) genitalis
Incubation period 9-90 d (3 wk) 3-10 days 2-20 d (6 d) 3-30 d 8-80 d
Primary:mostly
asymptomatic,
others have
fever, malaise
associated with
Multiplesmall
extremely
inflammatory
painful,
papules which
-Usually Single, tender,
become pustular
painless, well- grouped
and rupture to form
demarcated, vesicles over
soft, painful, Lesion starts as a
round ulcer with erythematous
tender, well- painless papuleà
firm-indurated base which on
defined ulcers ulceratesàvelvety
clean base and rupturing Transient,asymptomatic
with ragged ulcerating
a raised margin. produce herpes-like vesicle or
undermined granulomatous
Lesional features -Lesions are erosions (deep papule usually single
edges and bleed masses with rolled
non-tender in some that is hardly noticed
easily on up edges, beefy-
-Extragenital on àpunched- out by patient.
manipulation and is red, friable
finger tip. ulcers.
more superficial surface which
Manipulation Recurrent: HG
without any bleeds easily
leads to serous lesions are
induration.
exudates & no usually
Auto-inoculation
bleeding. asymptomatic or
from primary
small, superficial
lesions may occur
erosions and
in adjacent skin
heals in 1-2
weeks unlike
primary GH
which takes 2-4
weeks.
Main presenting
Bilateral, tender,
featurepresenting
“Inflammatory firm in primary Granulomatous
aspainful & tender
Usually bubo”:usually in primary nodules in
lymph nodes (unilateral
bilateral,non- unilateral, cases,but subcutaneous
in 2/3rd of cases)which
Lymph-adenopathy tender, firm, tender, matted usually minimal tissue forming
get matted together
shotty, discrete & and forming in recurrent “pseudobuboes”
and for multilocular
non-suppurative. “unilocular episodes unless not true
abscessesàrupture
abscesses” secondarily lymphadenopaty.
through multiple sinuses
infected.
“Sign of groove” +

Q.254 A young male presents with painful ulcers on the mouth and glans penis with blurred vision and history of
recurrent epididymitis. Which of the following is the most probable diagnosis?
a. Behcet syndrome
b. Oculocutaneous aphthous ulcer syndrome
c. Fabry’s disease
d. Epidermolysis bullosa
Your Ans. a Correct Ans. a
Solution. Ans 254: (a) Behcet syndrome
Ref– Read the text below
Sol:
Diagnostic criteria for Behçet's disease are:
Major:must have oral aphthous ulcers (any shape, size, or number at least 3 times in any 12 months period)
along with 2 out of the following 4 Minor symptoms:
Minor:
· Genital ulcers (including anal ulcers and spots in the genital region and swollen testicles or epididymitis in men)
· Skin lesions (papulo-pustules, folliculitis, erythema nodosum, acne in post-adolescents not on corticosteroids)
· Eye inflammation (iritis, uveitis, retinal vasculitis, cells in the vitreous)
· Pathergy reaction (papule >2 mm diameter, 24-48 hrs or more after needle-prick). The pathergy test has a specificity of 95%
to 100%.
Q.255 A middle aged male presents with multiple painful blisters on a erythematous base along the T3 dermatome on
the trunk. Which of the following etiological agents is most likely to be implicated?
a. Pox virus
b. Varicella zoster virus
c. Herpes simplex virus
d. Human papilloma virus
Your Ans. b Correct Ans. b

Solution. Ans 255: (b) Varicella zoster virus


Ref– Read the text below
Sol:
· Explanation: Varicella(chicken pox)àprimary lesion, which is kept latent in dorsal root ganglion. Reactivation of the virus
from the dorsal root ganglion lead to àherpes zoster (shingles)àusually affects the elderly and the
immunocompromisedàConstitutional symptoms followed by tingling and pain, erythema, and vesicle formation in a unilateral
dermatomaldistribution (Thoracic > cranial > cervical > lumbar). Ocular involvement-Herpes ophthalmitis (Hutchinson’s
sign =presence of vesicles on the tip and side of nose heralds eye involvement). Post-herpetic neuralgia: common
complication of HZ especially in elderly patients.
· Treatment: Oral acyclovir, (Famciclovir, Valacyclovir). In immunocompromised patient, IV acyclovir treatment is indicated.
Q.256 A male presents with sudden onset painful tense blisters with urticarial plaques. Which of the following would be
the best test for establishing a diagnosis in this case?
a. Direct immunoflourescence
b. Histopathology
c. Cytopathology
d. Indirect immunofluorescence
Your Ans. a Correct Ans. a

Solution. Ans 256: (a) Direct immunoflourescence


Ref– Read the text below
Sol:
Explanation: diagnosis is “Bullous Pemphigoid”.
It is commoner than pemphigus vulgaris & occurs in old age (i.e. >60).
Pathogenesis: Production of autoantibody reacting with basement membrane zone which in turns leads to separation of
epidermis from the dermis. The target sites are major & minor bullous pemphigoid antigens (BP antigen-1& 2 respectively) which
are lying in the hemidesmosomes (BP 1) and lamina lucida (BP 2). Complement, eosinophils and neutrophils also appear to be
required to form blisters.

Clinical Features
§ Characterized by tense blisters, often with a preceding or concurrent urticarial plaque by 1-3 weeks.
§ Presents as large, tense, intact blisters at lower abdomen, inner thighs, groin, axillae and flexures. Face and scalp are less
involved.
§ Mucosal lesions are less common
§ Nikolsky's (-), Bulla spread sign (-/+).
Diagnosis: skin biopsy: subepiderma cleft with eosinophils>> neutrophils in blister cavity.
Confirmatory test: DIF (direct immunoflourescence): which shows linear deposition of C3 and IgG at dermo-epidermal junction.
Q.257 The Ridley-Jopling classification for leprosy is based on which of the following parameters?
a. Histopathological, clinical, bacteriological, immunological
b. Histopathological, clinical, bacteriological, therapeutic
c. Histopathological, bacteriological, epidemiological, therapeutic
d. Histopathological, epidemiological, bacteriological, therapeutic
Your Ans. a Correct Ans. a
Solution.
Ans 257: (a) Histopathological, clinical, bacteriological, immunological
Ref– Read the text below
Sol:
The most widely accepted scheme of classification is Ridley and Jopling classificationbased on 4 parameters:
1. Clinical
2. Bacteriological (slit smear)
3. Histological (skin biopsy)
4. Immunological (Lepromin testing)
Features TT BT BB BL LL
Skin
lesions
Single or few Numerous, Numerous,
Number 1-3 Several (10-30)
(3-10) uncountable uncountable
B/L B/L
Symmetry Localized Localized Asymmetrical
Asymmetrical Symmetrical
Size May be large Variable Variable Usually small Small
àBoth well
defined and ill
defined lesions àUsually ill-
coexist defined
àWell-defined
(polymorphic) macules with
but with areas
Well-defined àBizarre classically
Clinical of poor
raised border geographical central
definition
Borders and depressed lesion infiltration Ill-defined
(Serrated);
center (saucer àAnnular àInverted
àSatellite
right side up) lesions saucer
lesionsnear
àSwiss lesionsmay
margins
cheeseor appear
punched out
lesionsare
characteristic
Moderately Mildly Mildly
Sensation Anesthetic Normoesthetic
anesthetic anesthetic anesthetic
Markedly Moderately Slightly
Hair Lost Normal
decreased decreased decreased
Surface Dry, scaly Dry ± scaly Dry/shiny Shiny Shiny
Multiple, Multiple,
Enlargement of
asymmetrical, symmetrical
1-2 peripheral, Few but usually
glove and (late)
Peripheral feeding nerves asymmetrical Variable,
stocking glove and
Nerves near the skin and near usually many
anesthesia stocking
lesions usually lesions
(later than in anesthesia
early.
LL) (Early)
(-) ve may be
Bacteriological AFB (-) ve 1-3+ 3-5+ 5-6+ (Globi)
rarely 1-2+
Immunological Lepromin +++ ++/+ - - -
Well-defined
Ill-defined
epitheloid cell
Epitheloid granulomas Histiocytic
granulomas
granulomas are containing few Granuloma
with giant cells Elongated
less well epitheloid cells, (Foamy
abutting the granulomas of
defined and few histiocytes, and macrophages =
epidermis highly foamy
giant cells only rare lepra or virchow
without any macrophages
(foreign body > lymphocytes cell), plenty of
free Clear grenz
langhans) and (naked lymphocytes.
supepidermal zone
many granulomas) Clear grenz
Histo Histology zone (Grenz Dermal nerves
lymphocytes. Clear grenz zone
zone) are usually
Subepidermal zone Dermal nerves
The dermal normal or
zone is Dermal nerves show marked
nerves are mildly
inconstant are infiltrated perineurial
greatly proliferative
Dermal nerves with mild proliferation
infiltrated or perineurium
are greatly reactive (‘onion-skin’
completely AFB+++++
swollen perineural appearance)
destroyed
AFB ± proliferation AFB ++++
AFB are usually
AFB+ +
absent
Q.258 A psoriatic patient presents with complaint of his usual topical steroid preparation becoming less and less
effective. Most likely, this is a phenomenon called
a. Woronoff’s
b. Tachyphylaxis
c. Auspitz’s
d. Munro’s
Your Ans. b Correct Ans. b

Solution. Ans-258: (b) Tachyphylaxis


Ref: Read the text below
Sol :
§ Stopping the steroid for a while or switching to other agents (e.g. tar preparations) for a time can help combat
tachyphylaxis.
§ Woronoff’s sign is the term for the ring of hypopigmentation that sometimes surrounds the periphery of psoriasis
plaques.
§ Auspitz’s sign is the pinpoint bleeding that occurs when deep psoriatic scale is removed, for example, with a fingernail.
§ It has been found to be nonspecific for psoriasis, but can be helpful in diagnosis.
§ Munro’s micro abscesses are pockets of neutrophils characteristically seen in histologic sections of psoriatic lesions,
and are not related to treatment.
Q.259 Treatment for verruca plana includes which of the following ?
a. Retin A
b. Efudex
c. Cryotherapy
d. All of the above
Your Ans. d Correct Ans. d

Solution. Ans-259: (d) All of the above


Ref: Read the text below
Sol :
§ Verruca plana can be treated withany one or combination of the following : topical tretinoin (Retin A), topical 5-
fluorouracil, and cryotherapy.
Q.260 Which of the following is known to cause hand-foot-and-month disease ?
a. Poxvirus
b. Treponema pallidum
c. Coxsackie virus
d. Human papillomavirus
Your Ans. c Correct Ans. c

Solution. Ans-260: (c) Coxsackie virus


Ref: Read the text below
Sol :
§ Hand-foot-and-mouth diseaseis associated with infection by subtype A 16 of the coxsackievirus family.
§ Poxvirus causesmolluscum contagiosum.
§ Treponema pallidumis a spirochete that causes syphilis.
§ Human papillomaviruscauses condylomata acuminate (genital warts).
Q.261 The neuromuscular blocker that does not need reversal of action by neostigmine at the end of the operation is
a. d-Tubocurarine
b. Doxacurium
c. Pipecuronium
d. Mivacurium
Your Ans. d Correct Ans. d

Solution. Ans-261: (d) Mivacurium


Ref.:KDT’s - 345
Sol :
§ Long acting non-depolarizing (competitive) NM blocking agentsrequire reversal with neostigmine.
§ Mivacurium is the shortest acting NDMR.
§ It does not require reversal due to its short duration of action.
§ Mivacurium can be used in day care surgery.
Q.262 Which of the following anesthetic agents cause hepatitis on repeated usage ?
a. Isoflurane
b. Desflurane
c. Halothane
d. None of the above
Your Ans. c Correct Ans. c

Solution. Ans-262: (c) Halothane


Ref.:KD Tripathi- 371
Sol :
§ Repeated doses of halothane cause hepatotoxicity (‘halothanehepatitis’ – do not use halothane twice within a 6-month
period) and it can trigger malignant hyperpyrexia.
Q.263 Prilocaine
a. Is an amino ester
b. Is used in eutectic mixtures
c. Is not metabolized by plasma cholinesterase
d. Is the LA of choice in labour analgesia
Your Ans. c Correct Ans. b

Solution. Ans-263: (b) Is used in eutectic mixtures


Ref.:KD Tripathi, 6th edn. Pg. 351, 357
Sol :

§ Prilocaine has been used mainly for infiltration, nerve block and IV regional anesthesia.
§ EMLA (eutectic mixture of LA – lignocaine and prilocaine) can produce anesthesia on the skin-used in the form of a skin
cream that is applied under occlusive dressing for 1 hour before IV cannulation, split skin graft harvesting, and other superficial
procedures.
§ One of the metabolites of prilocaine is ortho-toluidine, which can produce methemoglobinemia.
Q.264 In spinal anaesthesia the segmental level of
a. Sympathetic block is lower than the sensory block
b. Sympathetic block is higher than the sensory block
c. Motor block is higher than the sensory block
d. Sympathetic, motor and sensory block has the same level
Your Ans. b Correct Ans. b

Solution. Ans-264: (b) Sympathetic block is higher than the sensory block
Ref.:KDT’s - 360
Sol :
§ Spinal anesthesia createsa zone of differential blockade in which sympathetic fibres are blocked two segments higher
and motor fibres are blocked two segments lower than the level of sensory block.
Q.265 Intravenous regional anaesthesia is suitable for
a. Orthopedic manipulation on the upper limb
b. Vascular surgery on the lower limb
c. Head and neck surgery
d. Caesarian section
Your Ans. a Correct Ans. a
Solution. Ans-265: (a) Orthopedic manipulation on the upper limb
Ref.:KDT’s - 361
Sol :
§ IVRA is indicated for procedures on upper limb or lower limb of less than one hour duration.
Q.266 Which of the following is a Co-axial breathing circuit ?
a. Magill’s Circuit
b. Bain’s Circuit
c. JRMATP circuit
d. Circle system
Your Ans. a Correct Ans. b

Solution. Ans-266: (b) Bain’s Circuit


Ref.:Lee’s anesthesia, 13th edn. Pg. 91, 620
Sol :
§ The Bain circuit is a “coaxial” Mapleson D-the same components, but the fresh gas flow tubing is directed within the
inspiratory limb, with fresh gas entering the circuit near the mask.
§ The Bain has been shown to add more heat and humidity to inhaled gases than other Mapleson circuits.
Q.267 The most rapidly acting nondepolarizing neuromuscular blocking agent which can be used as an alternative to
succinylcholine for tracheal intubation is
a. Rocuronium
b. Pancuronium
c. Doxacurium
d. Pipecuronium
Your Ans. a Correct Ans. a

Solution. Ans-267: (a) Rocuronium


Ref.:KDT’s - 345
Sol :
§ Rocuronium is the fastest acting non-depolarizing muscle relaxant(NDMR).
§ It can be used for the rapid sequence endotracheal intubation in patients having contra-indications to the use of
SCh.
§ Mivacurium is the shortest acting NDMR.
Q.268 The following antibiotic accentuates the neuromuscular blockade produced by pancuronium
a. Streptomycin
b. Erythromycin
c. Penicillin G
d. Chloramphenicol
Your Ans. a Correct Ans. a

Solution. Ans-268: (a) Streptomycin


Ref.:KDT’s - 722
Sol :
Major adverse effects of aminoglycosides are :
§ Nephrotoxicity
§ Ototoxicity
§ Neuromuscular blockade
Q.269 Dantrolene sodium reduces skeletal muscle tone by
a. Reducing acetylcholine release from motor nerve endings
b. Suppressing spinal polysynaptic reflexes
c. Inhibiting the generation of muscle action potential
d. Reducing Ca2+ release from sarcoplasmic reticulum in the muscle fiber
Your Ans. d Correct Ans. d

Solution. Ans-269: (d) Reducing Ca2+ release from sarcoplasmic reticulum in the muscle fibre
Ref.:KDT’s - 347
Sol :
§ Dantrolene is the drug of choice forthe treatment of malignant hyperthermia and neuroleptic malignant syndrome.
§ It acts as an antagonist of ryanodine receptors (present on smooth endoplasmic reticulum).
It inhibits the release of Ca2+ from sarcoplasmic reticulum in the muscle fibre.
Q.270 Which of the following statements is not true of local anesthetics ?
a. The local anesthetic is required in the unionized form for penetrating the neuronal membrane
b. The local anesthetic approaches its receptor only from the intraneuronal face of the Na+ channel
c. The local anesthetic binds to its receptor mainly when the Na+ channel is in the resting stat
d. The local anesthetic combines with its receptor in the ionized cationic form
Your Ans. c Correct Ans. c

Solution. Ans-270: (c) The local anesthetic binds to its receptor mainly when the Na+ channel is in the resting state
Ref.:KDT’s - 353
Sol :
§ All local anesthetics are weak bases.
§ Las act by blocking Na+ channels from inside the neuron (intraneuronal face).
§ These can cross the membrane only in unionized (lipid soluble) form. Sodium bicarbonate is therefore added to make
the LA rapid acting.
§ Once inside the neuron, Las again gets ionized and bind to Na+ channels.
Binding to Na+ channels is more in repeticitively firing neurons than in resting neurons.
Q.271 A scintillation crystal widely used for gamma-ray detection is
a. LSO
b. BGO
c. YSO
d. Nal(TI)
Your Ans. d Correct Ans. d

Solution. Ans-271: (d) Nal(TI)


Ref:Read the text below
Sol :
· NaI doped with Thallium is usedas a detector in a gamma camera. All the other three are detector crystals used in PET
scanners.
Q.272 Parasites that may show calcification on radiographs include
a. Cysticercosis
b. Guinea worm
c. Amoebiasis
d. Loa Loa
Your Ans. a Correct Ans. a

Solution. Ans-272: (a) Cysticercosis


Ref.:Read the text below
Sol :
§ Parasitic calcification is described ina wide number of infestations including cysticercosis & guinea worm when the
appearances are characteristic.
Q.273 ‘ Scalloping’ of the edge of sigmoid colon on barium enema is seen in : -
a. Divericulosis
b. Pneumatosis Intestinalis
c. Ulcerative colitis
d. Carcinoma colon
Your Ans. d Correct Ans. a

Solution. Ans 273: (a) Divericulosis


Reference – Read the text below.
Sol:
· ‘ Scalloping’ of the edge of sigmoid colon on barium enema is seen indivericulosis
Q.274 Primary radiation is
a. Which goes from cathode to anode of X-ray tube
b. Which goes from X-ray tube to patient
c. Which goes from patient
d. Which goes from film to patient
Your Ans. b Correct Ans. a
Solution. Ans-274: (a) Which goes from cathode to anode of X-ray tube
Ref: Christensen’s Physics of Diagnostic Radiology 4th Ed-64
Sol:
§ Primary radiation : It goes from cathode to anode of X-ray tube. Except for the useful beam, the bulk of this radiation is
absorbed in the tube housing.
§ Radiation arriving directly from its source without interaction with matter.
§ Secondary radiation : It is radiation other than primary and is emitted by any matter irradiated with X-rays, which are often
loosely called scattered radiation.
Q.275 The potential difference applied to an X-ray tube is increased. As a result, in the emitted radiation
a. The intensity increases
b. The minimum wavelength increases
c. The intensity remains unchanged
d. The minimum wavelength decreases
Your Ans. a Correct Ans. a

Solution. Ans-275: (a) The intensity increases


Ref: Christensen’s Physics of Diagnostic Radiology 4th Ed., 34
Sol:
§ The wavelength of the characteristic radiations produced by the target of X ray tube is not changed by the potential
difference (kVp) applied.
§ The overall intensity is proportional to kVp2. The tube current (mA) determines the number of electrons hitting the anode,
and therefore the total amount of X-rays.
§ The quantity of X-rays produced depends on atomic number of target material, kVp and mA, while the quality depends on
kVp only.
Q.276 Classical x-ray appearance of “bronchiectasis” is
a. Blunting of diaphragmatic costal angles
b. Complete opacification of one lobe with no additional findings
c. Patchy consolidation centered on bronchi
d. Prominent bronchi that can be followed far out into the lung fields
Your Ans. d Correct Ans. d

Solution. Ans 276: - (d) Prominent bronchi that can be followed far out into the lung fields
Ref- Read the text below
Sol.
· X-ray findings suggestive of bronchiectasis include scattered irregular opacities caused by mucous plugs,
honeycombing, and rings and “tram lines” caused by thickened, dilated airways located perpendicular to the x-ray beam and
prominent bronchi that can be followed far out into the lung fields.
· Radiographic patterns may differ by underlying disease: Bronchiectasis due to cystic fibrosis develops predominantly in
upper lobes, whereas that due to other causes is more diffuse or predominates in the lower lobes.
· High-resolution CT is the test of choice for defining the extent of bronchiectasis.
Q.277 A 79 year old man presents with a gradual deterioration in mental state, he also has had difficulty in walking and
has had frequent falls. Here is his CT scan:

With this scan appearance, what other piece of clinical information will help to establish the diagnosis?
a. Urinary incontinence
b. Sexually inappropriate
c. Auditory hallucinations
d. Resting tremor
Your Ans. a Correct Ans. a
Solution. Ans 277: - (a) Urinary incontinence
Ref- Read the text below
Sol.
· This case represents one of normal pressure hydrocephalus. Clinically described as a triad of - urinary incontinence,
falls (gait apraxia) and dementia.
· CT scan or MRIwill demonstrate ventricular dilation.
· This is a potentially treatable cause of dementia. The treatment option in the first instance is lumbar puncture. If a clinical
improvement is shown with lumbar puncture then the patient may benefit from VP shunt.
Q.278 X-rays are modified
a. Photons
b. Neutrons
c. Positrons
d. Protons
Your Ans. a Correct Ans. a

Solution. Ans-278: (a) Photons


Ref.:Read the text below
Sol :
X-radiation(composed of X-rays) is a form of electromagnetic radiation. X-rays have a wavelengthin the range of 0.01 to 10
nanometers, corresponding to frequenciesin the range 30 petahertzto 30 exahertz(3×1016 Hz to 3×1019 Hz) and energies in the
range 100 eVto 100 keV. Penetrating electromagnetic radiation with wavelengths shorter than those of visible light, usually
produced by bombardment of a metallic target with fast electrons in a high vacuum.
Q.279 Left lateral view with barium on oesophagus helps to assess chamber enlargement in all except :
a. Fallot’s tetralogy
b. Mitral incompetence
c. Pericardial effusion
d. Aortic stenosis
Your Ans. c Correct Ans. c

Solution. Ans-279: (c) Pericardial effusion


Ref.:Read the text below
Sol :
§ Position of CXR for left pleural effusion is left lateral decubitus.
§ The enlarged right ventricle is best seen in the lateral view where it fills in the normal retrosternal space, but is also
identifiable when gross in the PA view where it straightens the left border and elevates the apex of the heart.
§ In mitral disease the combination of the left auricular and right ventricular enlargement leads to the ‘cottage loaf’
appearance.
Q.280 The quenching gas in a Geiger-Muller Counter is generally:
a. Argon
b. Helium
c. Ethyl alcohol
d. Monosodium glutamate
Your Ans. a Correct Ans. c

Solution. Ans-280: (c) Ethyl alcohol


Ref:Read the text below
Sol :
· GM counters are used as radiation measuring devices.
· They work at a high voltage where high number of secondary ionizations occur and a quenching gas is required to
absorb the avalanche of electrons.
Q.281 Amphetamine psychois (stimulant induced psychotic disorder) is characterized by all of the following except
a. Depression
b. Loose associations
c. Clear sensorium
d. Paranoia
Your Ans. c Correct Ans. a
Solution. Ans 281: (a) Depression
Reference – Read the text below
Sol:
· Stimulant psychosisoften is associated with paranoia and loose associations in a clear sensorium, which may make it
indistinguishable from schizophrenia.
· Tactile hallucinations and delusions of infestationwith parasites also may occur.
Depression is usually associatedwith stimulant withdrawal, not stimulant intoxication.
Q.282 Common side effects of tricyclic antidepressants include all of the following except
a. Orthostatic hypotension
b. Dry mouth
c. Diarrhea
d. Blurred vision
Your Ans. c Correct Ans. c

Solution. Ans-282: (c) Diarrhea


Ref: Read the text below
Sol:
§ Tricyclic antidepressantshave a variety of side effects, many of which are related to postsynaptic receptor-blocking
effects.
§ Blockade of muscarinic receptors cause anticholinergic symptomssuch as dry mouth, constipation, blurred vision, and
tachycardia. Blockade of histamine receptors produces sedation.
§ Orthostatic hypotensionis caused by adrenergic (α)-blockade.
§ Diarrhea is not a common side effect of this drug class.
Q.283 Effective treatments for dysthymic disorder include all of the following except
a. Selective serotonin reuptake inhibitors (SSRIs)
b. Monoamine oxidase inhibitors (MAOIs)
c. Electroconvulsive therapy (ECT)
d. Cognitive therapy
Your Ans. c Correct Ans. c

Solution. Ans-283: (c) Electroconvulsive therapy (ECT)


Ref: Read the text below
Sol:
§ Although dysthymia was long thought to be refractory to pharmacotherapy, it is now thought that many patients may
respond to medications, particularly selective serotonin reuptake inhibitors (SSRIs) and monoamine oxidase inhibitors
(MAOIs).
§ Of the psychotherapies, the best evidence exists for the effectiveness of cognitive and behavioral approaches.
Dysthymiais not an indication for electroconvulsive therapy (ECT
Q.284 Opioid overdoses produce all of the following signs except
a. Dilated pupils
b. Hypotension
c. Depressed reflexes
d. Coma
Your Ans. a Correct Ans. a

Solution. Ans-284: (a) Dilated pupils


Ref: Read the text below
Sol:
§ Like alcohol, barbiturates, and benzodiazepines, narcotics can produce depression of consciousness, reflexes, blood
pressure, and respiration.
§ Narcotics cause constricted rather than dilated pupils.
§ Opioid intoxication can be diagnosed with intravenous naloxone.
Q.285 A 47-year-old woman presents with pain during intercourse. The pain has become increasingly problematic over
the last 6 months. The first item in the differential diagnosis should be
a. Vaginismus
b. Atrophic vaginitis
c. Dyspareunia
d. Major depression
Your Ans. c Correct Ans. b
Solution. Ans-285: (b) Atrophic vaginitis
Ref: Read the text below
Sol:
§ On a statistical basis and on the basis of the patients age, the physician may suspect that menopause may have decreased
this woman’s circulating estrogen levels, resulting in atrophic vaginitis.
§ However, the physician must perform a pelvic examination before making a diagnosis because of the possibility that a
neoplasm or other disease may be causing the pain.
§ Diagnoses such as dyspareunia and vaginismus repuire that general medical conditions be excluded before making these
diagnoses.
§ Although dyspareunia is a symptom of somatization disorder, a late onset of the symptom is unusual in a patient
suffereing from somatization disorder.
Q.286 Factors thought to be involved in the etiology of an antisocial personality disorder include.
a. Closed head injury
b. Encephalitis
c. An alcoholic father
d. Loss of a parent
Your Ans. d Correct Ans. c

Solution. Ans-286: (c) An alcoholic father


Ref: Read the text below
Sol:
§ Although the etiology of antisocial disorder is not clear, it is thought that one powerful predictor is a sociopathic or
alcoholic father.
§ Antisocial behaviorhas been associated with closed head injury and encephalitis; however, the proper diagnosis for these
behaviors is an organic personality syndrome.
§ Loss of a parenthas not been associated with the development of antisocial personality, but inconsistent and compulsive
parenting has.
Although antisocial personality is associated with low socioeconomic status, this is not thought to be a cause but rather the
result of this kind of behavior.
Q.287 Which of the following disorders is thought by many to be strongly influenced by cultural roles?
a. Avoidant personality
b. Antisocial personality
c. Dependent personality
d. Borderline personality.
Your Ans. b Correct Ans. d

Solution. Ans 287: (d) Borderline personality


Reference – Read the text below
Sol:
· Some researchers in psychiatry feel thatdependent personality disorder depends to a large degree on cultural roles
(i.e., certain individuals or groups are expected to assume dependent roles in the culture based on their gender or ethnic
background).
To some extent, this may be changing, but it can still be observed.
Q.288 A patient with which one of the following disorders is most likely to handle the stress of hospitalization relatively
well?
a. Schizotypal
b. Dependent
c. Antisocial
d. Avoidant
Your Ans. b Correct Ans. d

Solution. Ans 288: (d) Avoidant


Reference – Read the text below
Sol:
· Patients with avoidant personality disorder are undermanding and generally cooperative.
· A medical illness allows them to be taken care of, which may help their illness.
· Patients with schizotypal, dependent, antisocial, and borderline personality disorders all tend to have more
problematic interactions with staff while hospitalized.
Q.289 Atypical depression describes a subset of major depressive disorder with which of the following symptoms?
a. Delusions and hallucinations
b. Manic as well as depressive symptoms
c. Overeating and oversleeping
d. Initial insomnia.
Your Ans. a Correct Ans. c

Solution. Ans 289: (c) Overeating and oversleeping


Reference – Read the text below
Sol:
· Atypical depression, which may respond more successfully to monoamine oxidase (MAO) inhibitors or selective serotonin
serotonin reuptake inhibitiors (SSRIs) than to other drugs, is so named because patterns of eating and sleeping are the opposite
of those seen in most depressed patients. Rather than exhibiting insomnia and poor appetite, those patients sleep
excessively and gain weight.
· Delusions and hallucinationsoccur in patients with major depressive disorder with psychotic features.
· When both manic and depressive symptoms are seen concurrently, the patient is said to exhibit a mixed episode.
· Worthlessness, guilt, and initial insomnia are all typical symptoms of major depression.
Q.290 Preoccupation with fear or belief of having serious diseases in body
a. Conversion disorder
b. Somatization disorder
c. Hypochondriasis
d. Obsessive compulsive disorder.
Your Ans. c Correct Ans. c

Solution. Ans 290: (c) Hypochondriasis


Reference – Read the text below
Sol:
· Hypochondriasis is defined as persistent preoccupationwith fear or belief of having one or more serious disease,
based on person’s own interpretation of normal body function or minor hysical abnormality.
· The fear or belief in not a delusion patient may agree, regarding the possibility of his exaggerating the graveness of
situation at the time.
Pre-ocupation with medical terms and syndromes in common.
Q.291 Biomechanics of fracture of the neck of the femur showing stress patterns of different angles of inclination has
been described by
a. Watson Jones
b. Charnley
c. Pauwel
d. Smith Peterson.
Your Ans. c Correct Ans. c

Solution. Ans-291: (c) Pauwel


Ref: Read the text below
Sol:
· Pauwel described the significance of the angle of inclination(of the fracture) on fracture healing.
When the angle of inclination was small the compressive force is greater than shearing force hence the chances of union
are more, whereas when the angle was more the shearing forces are more than the compressive force hence the chances of
healing were reduced.
Q.292 X-ray signs of acute osteomyelitis are seen after
a. 48 hours
b. 10 days
c. 3 weeks
d. 5days
Your Ans. b Correct Ans. b

Solution. Ans-292: (b) 10 days


Ref:Read the text below
Sol:
§ Films are negative in the initial 10 days subsequentlya localized area of bone destruction is observed in the metaphysic
surrounded by a wide zone of decalcified bone.
§ Later within the next few weeks the periosteal shadow is elevated at the same level and multiple lamination of bone
parallel with the shaft are seen.
Q.293 Tenderness at the anatomical snuff box is characteristic of
a. Scaphoid fracture
b. Radial styloid fracture
c. De quervain’s tensoynovitis
d. All of the above
Your Ans. d Correct Ans. d

Solution. Ans-293: (d) All of the above


Ref:Read the text below
Sol:
· The radius and scaphoid articulate deep to the snuffbox to form the basis of the wrist joint. In the event of a fall onto
an outstretched hand, this is the area through which the brunt of the force will focus.
· This results in these two bones being the most often fractured of the wrist. In a case where there is
localized tenderness within the snuffbox, knowledge of wrist anatomy leads to the speedy conclusion that the fracture is likely to
be of the scaphoid.
· This is understandable as the scaphoid is a small, oddly shaped bone whose purpose is to facilitate mobility rather than
confer stability to the wrist joint.
· In the event of inordinate application of force over the wrist, this small scaphoid is clearly likely to be the weak link.
Interestingly, scaphoid fracture is one of the most frequent causes of medico-legal issues.
Q.294 Mallet finger occur due to tear of the
a. Plantar aponeuosis
b. Terminal flexors
c. Terminal extensors
d. Dorsal venous arch
Your Ans. c Correct Ans. c

Solution. Ans-294: (c) Terminal extensors


Ref:Read the text below
Sol:
§ Mallet fingeris also called base ball finger. This result form injury to the extensor tendon of the terminal phalanx.
§ It may be due to direct trauma but more often following tendon rupture when the finger tip is forcibly bent during active
extension perhaps while tucking the blankets under the mattress or trying to catch a ball
Q.295 All the following are radiological features of scurvy except
a. White line of Fraenkel
b. Pelkan spur
c. Wimberger’s line
d. Schmorl’s node
Your Ans. d Correct Ans. d

Solution. Ans-295: (d) Schmorl’s node.


Ref: Read the text below
Sol:
· The white line of Fraenkel – a broadened irregular radioopaque line caused by calcified cartilage between the epiphyseal
line and the metaphysis.
· The pelkan spur-a small bony spur protruding from the lateral and occasionally the medial border of the metaphysic.
· The scurvy line-the zone of translucency in the metaphysic adjacent to the white line of Fraenkel.
The winberger line- the dense line encircling the epiphysis.
Q.296 A pilon fracture of tibia is
a. Intraarticular fracture of distal tibia
b. Compound fracture of tibia
c. Comminuted fracture of upper tibia
d. Intraarticular fracture of upper tibia.
Your Ans. a Correct Ans. a

Solution. Ans-296: (a) Intraarticular fracture of distal tibia


Ref: Read the text below
Sol:
· A pilon fracture of tibia is intraarticular fracture of distal tibia
This fracture is also called the plafond fracture.
Q.297 Dislocation of the inferior radioulnar joint and fracture head of radius is known as
a. Monteggia fracture dislocation
b. Galeazi fracture dislocation
c. Essex lopresti dislocation
d. Soprani’s dislocation
Your Ans. c Correct Ans. c

Solution. Ans-297: (c) Essex lopresti dislocation


Ref: Read the text below
Sol:
· Comminuted fracture of the head of radiuswith dislocation of inferior radio ulnar joint is known as Essex Lopresti
fracture dislocation.
Q.298 Most common cause of Tardy Ulnar Nerve palsy is
a. Fracture lateral condyle humerus
b. Anterior dislocation of shoulder
c. Fracture head of radius
d. Fracture shaft of humerus.
Your Ans. a Correct Ans. a

Solution. Ans-298: (a) Fracture lateral condyle humerus


Ref: Read the text below
Sol:
· Non Union lateral condyle humerusleads to cubitus valgus, and its importance is the liability of delayed ulnar nerve
palsy.
· Years after the injury the patientnotices weakness of the hand with numbness and tingling of the medial 2
fingers,this is because of attrition damage of the ulnar nerve.
Treatment is by anterior transposition of the ulnar nerve.
Q.299 An enchondroma rarely exhibits aggressive behavior but which of the following is an ominous symptom in a
patient with en-chondroma signaling malignant change.
a. Constitutional symptoms like fever
b. Weight loss
c. Pain in the absence of fracture
d. Breathlessness
Your Ans. c Correct Ans. c

Solution. Ans-299: (c) Pain in the absence of fracture


Ref: Read the text below
Sol:
· In the absence of fracture, pain is a ominous feature and suggests that the tumor is growing and may be malignant.
Q.300 All the following are complications of multiple myeloma except
a. Pathological fracture
b. Polycythemia
c. Amyloidosis
d. Ampaired immune function
Your Ans. b Correct Ans. b

Solution. Ans-300: (b) Polycythemia


Ref: Read the text below
Sol:
· Serum microglobulin estimationmay provide an useful assessment of prognosis.
· Poor prognostic featuresof multiple myeloma are Hb less than 70 g/l, severe hypo albuminemia, intractable renal failure,
thrombocytopenia, plasma cell leukemia.
· Anemia and pancytopeniabecause of bone marrow involvement with malignant plasma cells, polycythemia is virtually
unheard of.

Vous aimerez peut-être aussi